Translate

MEDICAL LAB TECHNICIAN EXAM MCQS PART 3 Q.NO. 2001 TO 3000

 

MEDICAL LAB TECHNICIAN EXAM MCQS PART 3

Q.NO. 2001 TO 3000

PDF DOWNLOAD  CLICK HERE

Ø PASSWORD BELLOWS SIDE IN LAST QUESTIONS

                PART- BLOOD BANK 50 MCQS

 

1. Question: What is the universal blood donor type?

A. A+

B. O-

C. AB+

D. B-

Answer: B. O-

 

2. Question: Which blood type is considered the universal recipient?

A. AB+

B. O-

C. A-

D. B+

Answer: A. AB+

 

3. Question: What is the most common blood type in the world?

A. AB+

B. A-

C. O+

D. B-

Answer: C. O+

4. Question: Which component of blood is responsible for clotting?

 

A. Platelets

B. Red Blood Cells

C. Plasma

D. White Blood Cells

Answer: A. Platelets

5. Question: What is the term for the process of separating blood into its individual components?

A. Hemolysis

B. Coagulation

C. Centrifugation

D. Agglutination

Answer: C. Centrifugation

 

6. Question: Which blood type is considered the "universal plasma donor"?

 

A. AB+

B. O-

C. A-

D. B+

Answer: A. AB+

 

7. Question: What is the Rh factor also known as?

A. Rhodium

B. Rhesus

C. Rhinovirus

D. Rhubarb

Answer: B. Rhesus

 

8. Question: Which blood group is known as the "Bombay blood group"?

A. A+

B. O-

C. AB+

D. O(h)

Answer: D. O(h)

9. Question: Which blood component is responsible for carrying oxygen to the body tissues?

 

A. Platelets

B. Red Blood Cells

C. Plasma

D. White Blood Cells

Answer: B. Red Blood Cells

 

10. Question: In the ABO blood group system, which antigens are present in type B blood?

- A. A antigens

- B. B antigens

- C. Both A and B antigens

- D. No antigens

**Answer: B. B antigens**

11. Question: Which blood type can a person with AB- receive in a blood transfusion?

- A. AB+

- B. O-

- C. A-

- D. B+

 

**Answer: B. O-**

12. Question: What is the term for an abnormal immune response to blood transfusion, leading to the destruction of transfused red blood cells?

- A. Hemolysis

- B. Agglutination

- C. Coagulation

- D. Transfusion Reaction

 

**Answer: D. Transfusion Reaction**

13. Question: What is the primary purpose of crossmatching in blood banking?

- A. Determine blood type

- B. Identify infectious agents

- C. Assess donor-recipient compatibility

- D. Measure clotting factors

 

**Answer: C. Assess donor-recipient compatibility**

14. Question: Which blood type is considered the rarest among the ABO groups?

- A. AB+

- B. O-

- C. A-

- D. B-

 

**Answer: A. AB-**

15. Question: What is the significance of the term "antibody screen" in blood banking?

- A. Screening for infectious agents in donated blood

- B. Detecting antibodies in the recipient's plasma

- C. Testing clotting factors in the blood

- D. Assessing red blood cell morphology

 

**Answer: B. Detecting antibodies in the recipient's plasma**

16. Question: Which laboratory test is performed to identify the presence of antibodies in a person's blood?

- A. Hemoglobin test

- B. Blood typing

- C. Direct Coombs test

- D. Complete Blood Count (CBC)

 

**Answer: C. Direct Coombs test**

17. Question: What is the purpose of cryoprecipitate in blood transfusion?

- A. Increase platelet count

- B. Provide clotting factors

- C. Enhance oxygen-carrying capacity

- D. Prevent allergic reactions

 

**Answer: B. Provide clotting factors**

18. Question: Which condition is characterized by an inadequate number of red blood cells or a deficiency of hemoglobin in the blood?

- A. Hemophilia

- B. Anemia

- C. Leukemia

- D. Thrombocytopenia

 

**Answer: B. Anemia**

19. Question: What is the primary purpose of storing blood in a blood bank?

- A. Prolong shelf life

- B. Prevent clotting

- C. Maintain temperature

- D. Ensure availability for transfusions

 

**Answer: D. Ensure availability for transfusions**

20. Question: In the Rh system, if a person is Rh-negative, what antibodies may be naturally present in their plasma?

- A. Anti-A

- B. Anti-B

- C. Anti-Rh

- D. No antibodies

 

**Answer: D. No antibodies**

21. Question: Which blood component is responsible for immune defense and fighting infections?

- A. Platelets

- B. Red Blood Cells

- C. Plasma

- D. White Blood Cells

 

**Answer: D. White Blood Cells**

22. Question: What is the primary antibody present in the plasma of a person with blood type B?

- A. Anti-A

- B. Anti-B

- C. Anti-AB

- D. Anti-Rh

 

**Answer: A. Anti-A**

23. Question: What is the purpose of the term "autologous donation" in blood banking?

- A. Donating blood for oneself

- B. Donating blood for a family member

- C. Donating blood for research purposes

- D. Donating blood for emergency use

 

**Answer: A. Donating blood for oneself**

24. Question: Which blood type is considered the "universal red cell donor" in emergency situations?

- A. AB+

- B. O-

- C. A-

- D. B+

 

**Answer: B. O-**

25. Question: What is the maximum storage time for packed red blood cells in a blood bank refrigerator?

- A. 14 days

- B. 21 days

- C. 30 days

- D. 45 days

 

**Answer: B. 21 days**

26. Question: In blood banking, what does the term "apheresis" refer to?

- A. Separation of blood components using centrifugation

- B. Blood transfusion from one individual to another

- C. Clotting of blood during storage

- D. Testing for infectious diseases in donated blood

 

**Answer: A. Separation of blood components using centrifugation**

27. Question: Which anticoagulant is commonly used in blood collection tubes for routine blood tests?

- A. Heparin

- B. EDTA

- C. Sodium citrate

- D. Sodium fluoride

 

**Answer: B. EDTA**

28. Question: What is the primary function of the Rh factor in blood typing?

- A. Determining blood type

- B. Controlling clotting factors

- C. Identifying donor compatibility

- D. Predicting Rh incompatibility during pregnancy

 

**Answer: D. Predicting Rh incompatibility during pregnancy**

29. Question: Which blood component is responsible for maintaining blood pressure and electrolyte balance?

- A. Platelets

- B. Red Blood Cells

- C. Plasma

- D. Albumin

 

**Answer: D. Albumin**

30. Question: In the direct Coombs test, what does a positive result indicate?

- A. Presence of antibodies on red blood cells

- B. Absence of antibodies in the plasma

- C. Clumping of red blood cells

- D. Incompatibility between donor and recipient blood

 

**Answer: A. Presence of antibodies on red blood cells**

31. Question: What is the purpose of a tourniquet during the blood donation process?

- A. To minimize blood clotting

- B. To identify veins for needle insertion

- C. To prevent infection

- D. To measure blood pressure

 

**Answer: B. To identify veins for needle insertion**

32. Question: Which blood type is considered the "universal plasma donor" in the ABO system?

- A. AB+

- B. O-

- C. A-

- D. B+

 

**Answer: A. AB+**

33. Question: What is the primary function of the Rh factor in the blood?

- A. Clotting

- B. Oxygen transport

- C. Antibody production

- D. Immune response

 

**Answer: B. Oxygen transport**

34. Question: In the ABO blood group system, which blood types can a person with blood type O receive in a transfusion?

- A. O only

- B. A and B

- C. O and AB

- D. O and B

 

35. Question: What is the term for a condition where the recipient's antibodies attack and destroy transfused red blood cells?

- A. Hemolysis

- B. Agglutination

- C. Coagulation

- D. Transfusion reaction

 

**Answer: A. Hemolysis**

36. Question: Which blood component is responsible for maintaining blood pH and electrolyte balance?

- A. Platelets

- B. Red Blood Cells

- C. Plasma

- D. Buffers

 

**Answer: D. Buffers**

37. Question: What is the primary screening test used to determine blood type?

- A. Crossmatching

- B. Hemoglobin test

- C. Blood typing

- D. Coagulation profile

 

**Answer: C. Blood typing**

38. Question: Which blood type is considered the "universal donor" for plasma in the ABO system?

- A. AB+

- B. O-

- C. A-

- D. B+

 

**Answer: A. AB+**

39. Question: In blood transfusion, what does the term "irradiated blood" refer to?

- A. Blood exposed to UV light for sterilization

- B. Blood treated with anticoagulants

- C. Blood stored at low temperatures

- D. Blood exposed to ionizing radiation to prevent graft-versus-host disease

 

**Answer: D. Blood exposed to ionizing radiation to prevent graft-versus-host disease**

40. Question: Which blood type is the least common in the ABO system worldwide?

- A. AB+

- B. O-

- C. A-

- D. B+

 

**Answer: A. AB+**

41. Question: What is the primary storage temperature for fresh-frozen plasma in a blood bank?

- A. -20°C

- B. -30°C

- C. -40°C

- D. -80°C

 

**Answer: A. -20°C**

42. Question: Which blood type is considered the "universal donor" for whole blood in the ABO system?

- A. AB+

- B. O-

- C. A-

- D. B+

 

**Answer: B. O-**

43. Question: What is the purpose of a blood bank's quality control program?

- A. To ensure proper documentation

- B. To monitor temperature conditions

- C. To maintain the accuracy of test results

- D. To manage donor recruitment

 

**Answer: C. To maintain the accuracy of test results**

44. Question: Which blood component is responsible for transporting hormones and nutrients throughout the body?

- A. Platelets

- B. Red Blood Cells

- C. Plasma

- D. White Blood Cells

 

**Answer: C. Plasma**

45. Question: What is the primary screening test for infectious diseases in donated blood?

- A. Blood typing

- B. Hemoglobin test

- C. Crossmatching

- D. Nucleic acid testing (NAT)

 

**Answer: D. Nucleic acid testing (NAT)**

46. Question: What is the purpose of an antiglobulin (Coombs) test in blood banking?

- A. To detect antibodies on red blood cells

- B. To determine blood type

- C. To assess platelet count

- D. To measure clotting factors

 

**Answer: A. To detect antibodies on red blood cells**

47. Question: Which blood component plays a crucial role in maintaining blood pressure and volume?

- A. Platelets

- B. Red Blood Cells

- C. Plasma

- D. Albumin

 

**Answer: D. Albumin**

48. Question: What is the primary function of the RhoGAM injection during pregnancy?

- A. To prevent Rh incompatibility

- B. To induce labor

- C. To enhance fetal development

- D. To increase hemoglobin levels

 

**Answer: A. To prevent Rh incompatibility**

49. Question: In the ABO system, which blood type has both A and B antigens on the red blood cells?

- A. AB+

- B. O-

- C. A-

- D. B+

 

**Answer: A. AB+**

50. Question: What is the term for a blood clot that forms and remains in a blood vessel?

- A. Embolus

- B. Thrombus

- C. Hematoma

- D. Petechia

 

**Answer: B. Thrombus**

 

Ø PART- SICKLE CELL 40 MCQS

1. Question: What is the primary cause of Sickle Cell Disease (SCD)?

A. Bacterial infection

B. Genetic mutation

C. Environmental factors

D. Autoimmune response

Answer: B. Genetic mutation

 

2. Question: In Sickle Cell Disease, the mutation affects the production of which protein in red blood cells?

A. Hemoglobin A

B. Hemoglobin B

C. Hemoglobin C

D. Hemoglobin S

Answer: D. Hemoglobin S

 

3. Question: What is the typical shape of red blood cells in individuals with Sickle Cell Disease?

A. Biconcave discs

B. Spherical

C. Crescent or sickle-shaped

D. Irregular polygons

Answer: C. Crescent or sickle-shaped

 

4. Question: Which population is most commonly affected by Sickle Cell Disease?

A. Asian

B. European

C. African

D. South American

Answer: C. African

 

5. Question: What is the major clinical manifestation of Sickle Cell Disease?

A. Chronic cough

B. Joint pain

C. Visual impairment

D. Insulin resistance

Answer: B. Joint pain

 

6. Question: Which organ is particularly vulnerable to damage in individuals with Sickle Cell Disease?

A. Liver

B. Kidney

C. Heart

D. Spleen

Answer: D. Spleen

 

7. Question: What triggers the sickling of red blood cells in Sickle Cell Disease?

A. High oxygen levels

B. Low oxygen levels

C. Elevated pH

D. Normal blood pH

Answer: B. Low oxygen levels

 

8. Question: Which of the following conditions can result from the blockage of blood vessels by sickled cells in Sickle Cell Disease?

A. Stroke

B. Osteoporosis

C. Rheumatoid arthritis

D. Hypertension

Answer: A. Stroke

 

9. Question: What is the recommended treatment for managing pain crises in individuals with Sickle Cell Disease?

A. Antibiotics

B. Blood transfusions

C. Pain medications

D. Surgery

Answer: C. Pain medications

 

10. Question: How is Sickle Cell Disease inherited?

- A. Autosomal dominant

- B. Autosomal recessive

- C. X-linked dominant

- D. X-linked recessive

 

**Answer: B. Autosomal recessive**

11. Question: What is the term used to describe the heterozygous condition where an individual carries one normal hemoglobin gene and one sickle hemoglobin gene?

- A. Sickle cell trait

- B. Sickle cell anemia

- C. Sickle cell crisis

- D. Sickle cell pneumonia

 

 

**Answer: A. Sickle cell trait**

12. Question: Which type of hemoglobin is predominant in individuals with Sickle Cell Disease during a crisis?

- A. Hemoglobin A

- B. Hemoglobin B

- C. Hemoglobin C

- D. Hemoglobin S

 

 

**Answer: D. Hemoglobin S**

13. Question: What is the primary role of hydroxyurea in the management of Sickle Cell Disease?

- A. Pain relief

- B. Immune system boost

- C. Inducing hemoglobin A production

- D. Reducing the frequency of painful crises

 

 

**Answer: D. Reducing the frequency of painful crises**

14. Question: Which of the following is a potential complication of Sickle Cell Disease related to the breakdown of red blood cells?

- A. Hemophilia

- B. Hyperbilirubinemia

- C. Thrombocytosis

- D. Leukopenia

 

**Answer: B. Hyperbilirubinemia**

15. Question: What is the gold standard diagnostic test for confirming Sickle Cell Disease?

- A. Complete Blood Count (CBC)

- B. Blood smear

- C. Hemoglobin electrophoresis

- D. Genetic testing

 

 

**Answer: C. Hemoglobin electrophoresis**

16. Question: Which organ is primarily responsible for the production of hemoglobin in the human body?

- A. Liver

- B. Spleen

- C. Bone marrow

- D. Kidney

 

 

**Answer: C. Bone marrow**

17. Question: In addition to pain management, what other therapeutic approach is commonly used for preventing complications in Sickle Cell Disease?

- A. Anticoagulant therapy

- B. Blood thinners

- C. Stem cell transplant

- D. Oxygen therapy

 

 

**Answer: C. Stem cell transplant**

18. Question: What is the term for the condition where sickle-shaped cells block small blood vessels, causing pain and organ damage?

- A. Ischemia

- B. Infarction

- C. Hemolysis

- D. Vaso-occlusion

 

**Answer: D. Vaso-occlusion**

19. Question: What preventive measure is recommended for newborns identified with Sickle Cell Disease through newborn screening?

- A. Blood transfusion

- B. Hydroxyurea therapy

- C. Pneumococcal vaccination

- D. Early stem cell transplant

 

 

**Answer: C. Pneumococcal vaccination**

20. Question: Which blood component is affected by the abnormal hemoglobin in Sickle Cell Disease, leading to its characteristic shape?

- A. Platelets

- B. Red Blood Cells

- C. Plasma

- D. White Blood Cells

 

 

**Answer: B. Red Blood Cells**

 

 

21. Question: Which of the following symptoms is NOT commonly associated with a sickle cell crisis?

- A. Abdominal pain

- B. Joint pain

- C. Chest pain

- D. Migraine headaches

 

 

**Answer: D. Migraine headaches**

22. Question: What is the term for the condition where the spleen is damaged and becomes non-functional in individuals with Sickle Cell Disease?

- A. Splenomegaly

- B. Sickle splenic syndrome

- C. Asplenia

- D. Hypersplenism

 

 

**Answer: C. Asplenia**

23. Question: What is the primary goal of hematopoietic stem cell transplantation in Sickle Cell Disease?

- A. Pain relief

- B. Cure the disease

- C. Increase red blood cell production

- D. Prevent complications

 

 

**Answer: B. Cure the disease**

24. Question: Which blood type is most commonly associated with Sickle Cell Disease?

- A. A+

- B. O-

- C. B-

- D. S

 

 

**Answer: A. A+**

25. Question: What is the term for the condition where a person with Sickle Cell Disease inherits one sickle cell gene and one gene for hemoglobin C?

- A. Sickle cell trait

- B. Sickle cell anemia

- C. Hemoglobin SC disease

- D. Double heterozygosity

 

 

**Answer: C. Hemoglobin SC disease**

26. Question: What is the primary mechanism by which hydroxyurea exerts its therapeutic effects in Sickle Cell Disease?

- A. Increases oxygen affinity of hemoglobin

- B. Stimulates red blood cell production

- C. Induces fetal hemoglobin production

- D. Inhibits sickle cell gene expression

 

**Answer: C. Induces fetal hemoglobin production**

27. Question: What is the term for the inherited condition where a person has two identical sickle cell genes (HbS/HbS)?

- A. Sickle cell trait

- B. Sickle cell anemia

- C. Double heterozygosity

- D. Hemoglobin SC disease

 

 

**Answer: B. Sickle cell anemia**

28. Question: What is the primary function of sickle cell screening programs?

- A. Identify carriers of the sickle cell trait

- B. Diagnose sickle cell complications

- C. Provide pain management

- D. Prevent vaso-occlusive crises

 

 

**Answer: A. Identify carriers of the sickle cell trait**

29. Question: Which of the following organs is particularly susceptible to infections in individuals with Sickle Cell Disease due to impaired immune function?

- A. Lungs

- B. Liver

- C. Kidneys

- D. Pancreas

 

 

**Answer: A. Lungs**

30. Question: What is the term for the acute chest syndrome, a serious complication of Sickle Cell Disease characterized by chest pain and difficulty breathing?

- A. Sickle cell crisis

- B. Vaso-occlusion

- C. Pneumonia

- D. Splenic sequestration

 

**Answer: A. Sickle cell crisis**

31. Question: What is the primary reason for the increased risk of infections in individuals with Sickle Cell Disease?

- A. Impaired immune response

- B. High white blood cell count

- C. Excessive production of antibodies

- D. Presence of abnormal platelets

 

**Answer: A. Impaired immune response**

32. Question: Which of the following complications is associated with the long-term use of hydroxyurea in Sickle Cell Disease treatment?

- A. Increased risk of infection

- B. Pulmonary hypertension

- C. Renal failure

- D. Bone marrow suppression

 

**Answer: D. Bone marrow suppression**

33. Question: What is the term for the condition where sickle-shaped cells block blood flow to the spleen, leading to rapid enlargement and potential life-threatening complications?

- A. Splenic sequestration

- B. Sickle splenic syndrome

- C. Hypersplenism

- D. Sickle cell trait

 

**Answer: A. Splenic sequestration**

34. Question: Which of the following is a potential complication of Sickle Cell Disease that results from the destruction of red blood cells and increased bilirubin production?

- A. Hyperthyroidism

- B. Hemochromatosis

- C. Cholecystitis

- D. Gallstones

 

**Answer: D. Gallstones**

35. Question: What is the term for the inherited condition where a person has one sickle cell gene and one gene for beta-thalassemia?

- A. Sickle cell trait

- B. Hemoglobin SC disease

- C. Sickle beta-thalassemia

- D. Sickle cell anemia

 

**Answer: C. Sickle beta-thalassemia**

36. Question: Which of the following complications can result from a lack of oxygen delivery to the bones and joints in Sickle Cell Disease?

- A. Osteoarthritis

- B. Osteoporosis

- C. Avascular necrosis

- D. Rheumatoid arthritis

 

**Answer: C. Avascular necrosis**

37. Question: What is the term for the condition where sickle cells cause the sudden pooling of blood in the spleen, leading to a drop in hemoglobin levels and potential shock?

- A. Splenic sequestration

- B. Vaso-occlusion

- C. Hypersplenism

- D. Sickle splenic syndrome

 

**Answer: A. Splenic sequestration**

38. Question: Which immunization is crucial for preventing infections in individuals with Sickle Cell Disease, particularly those caused by Streptococcus pneumoniae?

- A. MMR vaccine

- B. Tetanus vaccine

- C. Pneumococcal vaccine

- D. Hepatitis B vaccine

 

**Answer: C. Pneumococcal vaccine**

39. Question: What is the term for the condition where sickle cells cause a sudden and severe drop in hemoglobin levels, leading to increased blood viscosity and potential stroke?

- A. Splenic sequestration

- B. Aplastic crisis

- C. Hyperhemolysis

- D. Thrombotic crisis

 

**Answer: B. Aplastic crisis**

40. Question: Which of the following is a potential complication of Sickle Cell Disease that results from the formation of blood clots in small blood vessels?

- A. Thrombocytopenia

- B. Deep vein thrombosis

- C. Pulmonary embolism

- D. Hemophilia

 

**Answer: C. Pulmonary embolism**

Ø MALARIA DISEASE 40 MCQS

1. Question: What is the primary cause of malaria?

 

A. Bacteria

B. Parasites

C. Virus

D. Fungi

Answer: B. Parasites

 

2. Question: Which species of Plasmodium is responsible for the most severe form of malaria in humans?

 

A. Plasmodium falciparum

B. Plasmodium vivax

C. Plasmodium malariae

D. Plasmodium ovale

Answer: A. Plasmodium falciparum

 

3. Question: What is the primary mode of transmission of malaria to humans?

 

A. Direct contact

B. Airborne droplets

C. Contaminated water

D. Mosquito bites

Answer: D. Mosquito bites

 

4. Question: In which part of the human body do malaria parasites undergo asexual reproduction?

 

A. Liver

B. Lungs

C. Red blood cells

D. Kidneys

Answer: C. Red blood cells

 

5. Question: Which type of malaria is known for its characteristic 48-hour fever cycle?

 

A. P. vivax malaria

B. P. falciparum malaria

C. P. malariae malaria

D. P. ovale malaria

Answer: C. P. malariae malaria

 

6. Question: Which of the following antimalarial drugs is derived from the bark of the cinchona tree?

 

A. Chloroquine

B. Artemisinin

C. Quinine

D. Mefloquine

Answer: C. Quinine

 

7. Question: What is the characteristic feature of Plasmodium vivax infection regarding relapses?

 

A. No relapses

B. Single relapse

C. Multiple relapses

D. Irregular relapses

Answer: C. Multiple relapses

 

8. Question: Which stage of the malaria parasite's life cycle is responsible for human-to-mosquito transmission?

 

A. Sporozoite

B. Trophozoite

C. Schizont

D. Gametocyte

Answer: D. Gametocyte

 

9. Question: What is the recommended preventive measure for individuals traveling to malaria-endemic areas?

 

A. Hepatitis A vaccination

B. Insect repellent

C. Oral rehydration solution

D. Tuberculosis prophylaxis

Answer: B. Insect repellent

 

10. Question: What is the common name for the mosquito species most responsible for transmitting malaria in Africa?

- A. Anopheles gambiae

- B. Aedes aegypti

- C. Culex pipiens

- D. Anopheles stephensi

 

**Answer: A. Anopheles gambiae**

 

 

11. Question: Which organ is primarily affected during the initial stages of malaria infection?

- A. Liver

- B. Lungs

- C. Spleen

- D. Kidneys

 

 

**Answer: A. Liver**

12. Question: What is the name of the diagnostic test commonly used to detect the presence of malaria parasites in a blood sample?

- A. ELISA test

- B. PCR assay

- C. Rapid diagnostic test (RDT)

- D. Western blot

 

 

**Answer: C. Rapid diagnostic test (RDT)**

13. Question: Which of the following is a common symptom of malaria?

- A. Joint pain

- B. Increased appetite

- C. Hypertension

- D. Euphoria

 

 

**Answer: A. Joint pain**

14. Question: What is the recommended first-line treatment for uncomplicated Plasmodium falciparum malaria in many regions where resistance is low?

- A. Chloroquine

- B. Artemisinin-based combination therapy (ACT)

- C. Quinine

- D. Mefloquine

 

 

**Answer: B. Artemisinin-based combination therapy (ACT)**

15. Question: In which stage of the malaria life cycle do symptoms typically occur in the human host?

- A. Sporozoite

- B. Trophozoite

- C. Schizont

- D. Gametocyte

 

 

 

**Answer: B. Trophozoite**

16. Question: Which population group is particularly vulnerable to severe malaria, including cerebral malaria?

- A. Adults

- B. Children under 5 years old

- C. Pregnant women

- D. Elderly individuals

 

 

**Answer: B. Children under 5 years old**

17. Question: Which Plasmodium species can cause relapses years after the initial infection?

- A. Plasmodium falciparum

- B. Plasmodium vivax

- C. Plasmodium ovale

- D. Plasmodium malariae

 

 

**Answer: C. Plasmodium ovale**

18. Question: What is the term for the severe form of malaria that can lead to seizures, coma, and death?

- A. Mild malaria

- B. Uncomplicated malaria

- C. Severe malaria

- D. Asymptomatic malaria

 

 

**Answer: C. Severe malaria**

19. Question: What is the primary vector control method for preventing malaria transmission?

- A. Insecticide-treated bed nets

- B. Indoor residual spraying

- C. Larviciding

- D. Genetic modification of mosquitoes

 

 

**Answer: A. Insecticide-treated bed nets**

20. Question: What is the name of the class of drugs that includes artemisinin derivatives, widely used in the treatment of malaria?

- A. Quinolines

- B. Antifolates

- C. Artemisinin-based compounds

- D. Aminoquinolines

 

 

**Answer: C. Artemisinin-based compounds**

 

 

21. Question: What is the primary role of the Anopheles mosquito in the malaria life cycle?

- A. Transmitting the malaria parasite to humans

- B. Acting as a reservoir for the malaria parasite

- C. Causing the initial infection in humans

- D. Producing gametocytes in humans

 

**Answer: A. Transmitting the malaria parasite to humans**

22. Question: What is the term for the dormant form of the malaria parasite that can persist in the liver for an extended period before causing a relapse?

- A. Sporozoite

- B. Trophozoite

- C. Hypnozoite

- D. Gametocyte

 

 

**Answer: C. Hypnozoite**

23. Question: Which continent bears the highest burden of malaria cases globally?

- A. Europe

- B. Asia

- C. Africa

- D. North America

 

 

**Answer: C. Africa**

24. Question: What is the primary mechanism by which artemisinin-based combination therapies (ACTs) work against malaria parasites?

- A. Inhibiting DNA synthesis

- B. Blocking protein synthesis

- C. Disrupting the parasite's cell membrane

- D. Inducing oxidative stress

 

 

**Answer: D. Inducing oxidative stress**

25. Question: Which of the following is NOT a common symptom of severe malaria?

- A. Respiratory distress

- B. Jaundice

- C. Hypotension

- D. Weight gain

 

 

**Answer: D. Weight gain**

26. Question: What is the recommended prophylactic drug for travelers to malaria-endemic areas?

- A. Chloroquine

- B. Doxycycline

- C. Artemether-lumefantrine

- D. Primaquine

 

 

**Answer: B. Doxycycline**

27. Question: Which of the following antimalarial drugs is known for causing vivid dreams or nightmares as a side effect?

- A. Mefloquine

- B. Artemisinin

- C. Atovaquone-proguanil

- D. Quinine

 

 

**Answer: A. Mefloquine**

28. Question: In which stage of the Plasmodium life cycle do merozoites burst out of infected red blood cells, leading to the release of new parasites?

- A. Sporozoite

- B. Trophozoite

- C. Schizont

- D. Gametocyte

 

 

**Answer: C. Schizont**

29. Question: What is the term for the phenomenon where individuals living in malaria-endemic areas develop partial immunity to severe forms of the disease over time?

- A. Acquired resistance

- B. Immune tolerance

- C. Malaria adaptation

- D. Premunition

 

 

**Answer: D. Premunition**

30. Question: What is the primary target of the malaria parasite in the human host during the asexual stage of its life cycle?

- A. Liver cells

- B. Red blood cells

- C. Lung cells

- D. Nervous system cells

 

 

**Answer: B. Red blood cells**

31. Question: What is the recommended treatment for uncomplicated malaria caused by Plasmodium vivax or Plasmodium ovale, considering their ability to form hypnozoites?

- A. Chloroquine

- B. Artemisinin-based combination therapy (ACT)

- C. Primaquine

- D. Mefloquine

 

 

**Answer: C. Primaquine**

32. Question: Which symptom distinguishes malaria from other febrile illnesses such as dengue and typhoid fever?

- A. Joint pain

- B. Headache

- C. Jaundice

- D. Cyclical fever spikes

 

 

**Answer: D. Cyclical fever spikes**

33. Question: What is the role of the female Anopheles mosquito in the transmission of the malaria parasite?

- A. It injects sporozoites into the human bloodstream during a blood meal.

- B. It lays eggs in stagnant water.

- C. It harbors the dormant form of the parasite.

- D. It produces gametocytes.

 

 

**Answer: A. It injects sporozoites into the human bloodstream during a blood meal.**

34. Question: In which geographic region is malaria caused by Plasmodium vivax most commonly found?

- A. Sub-Saharan Africa

- B. Southeast Asia

- C. South America

- D. Central Asia

 

 

**Answer: B. Southeast Asia**

35. Question: Which antimalarial drug is known for its use in the prevention of malaria during pregnancy due to its safety for the fetus?

- A. Doxycycline

- B. Artemisinin-based combination therapy (ACT)

- C. Mefloquine

- D. Chloroquine

 

 

**Answer: D. Chloroquine**

36. Question: What is the term for the condition where individuals carry the malaria parasite without showing symptoms and serve as a potential reservoir for transmission?

- A. Symptomatic carriers

- B. Asymptomatic carriers

- C. Hyperparasitemia

- D. Resistant carriers

 

 

**Answer: B. Asymptomatic carriers**

37. Question: Which type of malaria is known for causing a more prolonged and chronic infection in the human host?

- A. Plasmodium falciparum malaria

- B. Plasmodium vivax malaria

- C. Plasmodium ovale malaria

- D. Plasmodium malariae malaria

 

 

**Answer: D. Plasmodium malariae malaria**

38. Question: What is the primary mechanism of action of quinoline antimalarial drugs like chloroquine and hydroxychloroquine?

- A. Inhibition of parasite enzymes

- B. Disruption of parasite cell membrane

- C. Inhibition of DNA synthesis

- D. Induction of oxidative stress

 

 

**Answer: C. Inhibition of DNA synthesis**

39. Question: Which organ is targeted by Plasmodium falciparum during the severe form of malaria known as cerebral malaria?

- A. Liver

- B. Lungs

- C. Brain

- D. Spleen

 

 

**Answer: C. Brain**

40. Question: What is the most effective method for preventing the spread of drug-resistant malaria parasites?

- A. Developing new antimalarial drugs

- B. Early diagnosis and prompt treatment

- C. Vector control measures

- D. Mass drug administration

 

 

**Answer: B. Early diagnosis and prompt treatment**

 

Ø BACTERIAL DISEASE 50 MCQS

1. Question: Which bacterium is responsible for causing tuberculosis in humans?

 

A. Escherichia coli

B. Mycobacterium tuberculosis

C. Streptococcus pneumoniae

D. Staphylococcus aureus

Answer: B. Mycobacterium tuberculosis

 

2. Question: What is the primary mode of transmission for Escherichia coli (E. coli) infections?

 

A. Airborne droplets

B. Contaminated food and water

C. Sexual contact

D. Blood transfusions

Answer: B. Contaminated food and water

 

3. Question: Which bacterial pathogen is responsible for causing Lyme disease?

 

A. Borrelia burgdorferi

B. Clostridium tetani

C. Salmonella enterica

D. Haemophilus influenzae

Answer: A. Borrelia burgdorferi

 

4. Question: What is the primary symptom of infection with Helicobacter pylori?

 

A. Diarrhoea

B. Abdominal pain

C. Respiratory distress

D. Skin rash

Answer: B. Abdominal pain

 

5. Question: Which bacterial species is commonly associated with urinary tract infections (UTIs)?

 

A. Streptococcus pyogenes

B. Escherichia coli

C. Mycobacterium tuberculosis

D. Clostridium difficile

Answer: B. Escherichia coli

 

6. Question: What bacterial genus includes species that cause food poisoning, such as Salmonella and E. coli?

 

A. Clostridium

B. Listeria

C. Enterococcus

D. Salmonella

Answer: D. Salmonella

 

7. Question: Which bacterium is responsible for causing dental caries (tooth decay)?

 

A. Streptococcus mutans

B. Staphylococcus aureus

C. Neisseria gonorrhoeae

D. Pseudomonas aeruginosa

Answer: A. Streptococcus mutans

 

8. Question: What bacterial pathogen is associated with the development of peptic ulcers in the stomach?

 

A. Helicobacter pylori

B. Clostridium difficile

C. Escherichia coli

D. Staphylococcus aureus

Answer: A. Helicobacter pylori

 

9. Question: Which bacterial disease is transmitted through the bite of infected fleas and can cause severe pneumonia and septicaemia?

 

A. Tetanus

B. Plague

C. Diphtheria

D. Anthrax

Answer: B. Plague

 

10. Question: What bacterial genus includes species that can cause both strep throat and necrotizing fasciitis (flesh-eating disease)?

- A. Streptococcus

- B. Staphylococcus

- C. Neisseria

- D. Clostridium

 

 

**Answer: A. Streptococcus**

 

 

11. Question: Which bacterium is associated with the formation of pseudo membranes in the throat and respiratory tract, leading to a condition known as diphtheria?

- A. Streptococcus pneumoniae

- B. Corynebacterium diphtheriae

- C. Neisseria meningitidis

- D. Clostridium botulinum

 

**Answer: B. Corynebacterium diphtheriae**

12. Question: What is the causative agent of urinary tract infections that can ascend to the kidneys, causing pyelonephritis?

- A. Escherichia coli

- B. Pseudomonas aeruginosa

- C. Klebsiella pneumoniae

- D. Enterococcus faecalis

 

 

**Answer: A. Escherichia coli**

13. Question: Which bacterial pathogen is responsible for causing syphilis, a sexually transmitted infection?

- A. Neisseria gonorrhoeae

- B. Treponema pallidum

- C. Chlamydia trachomatis

- D. Mycoplasma genitalium

 

 

**Answer: B. Treponema pallidum**

14. Question: What bacterial species is commonly associated with the development of pneumonia in individuals with weakened immune systems, such as those with HIV/AIDS?

- A. Mycobacterium tuberculosis

- B. Legionella pneumophila

- C. Pseudomonas aeruginosa

- D. Streptococcus pneumoniae

 

 

**Answer: C. Pseudomonas aeruginosa**

15. Question: Which bacterial infection is characterized by the formation of a toxin that can cause severe muscle spasms and respiratory failure, known as "lockjaw"?

- A. Botulism

- B. Tetanus

- C. Anthrax

- D. Pertussis

 

 

**Answer: B. Tetanus**

16. Question: What bacterial pathogen is responsible for causing bubonic, septicaemic, and pneumonic forms of disease, collectively known as the Black Death in medieval Europe?

- A. Yersinia pestis

- B. Francisella tularensis

- C. Bacillus anthracis

- D. Mycobacterium leprae

 

 

**Answer: A. Yersinia pestis**

17. Question: Which bacterial species is commonly associated with foodborne intoxications and is responsible for causing symptoms such as vomiting and diarrhoea?

- A. Listeria monocytogenes

- B. Clostridium difficile

- C. Bacillus cereus

- D. Shigella flexneri

 

 

**Answer: C. Bacillus cereus**

18. Question: What is the primary mode of transmission for the bacterium Chlamydia trachomatis, causing sexually transmitted infections as well as eye infections in newborns?

- A. Airborne droplets

- B. Sexual contact

- C. Contaminated water

- D. Vector bites

 

**Answer: B. Sexual contact**

19. Question: Which bacterial species is responsible for causing Legionnaires' disease, a severe form of pneumonia often associated with contaminated water sources?

- A. Mycobacterium avium

- B. Legionella pneumophila

- C. Streptococcus agalactiae

- D. Haemophilus influenzae

 

**Answer: B. Legionella pneumophila**

20. Question: What is the common name for the bacterial infection caused by Yersinia enterocolitica, leading to symptoms such as abdominal pain and diarrhoea?

- A. Campylobacteriosis

- B. Yersiniosis

- C. Shigellosis

- D. E. coli infection

 

 

**Answer: B. Yersiniosis**

 

 

21. Question: What bacterium is responsible for causing whooping cough, a highly contagious respiratory infection that primarily affects infants and young children?

- A. Bordetella pertussis

- B. Streptococcus pyogenes

- C. Mycoplasma pneumoniae

- D. Haemophilus influenzae

 

 

**Answer: A. Bordetella pertussis**

22. Question: Which bacterial species is commonly associated with community-acquired pneumonia and is a leading cause of respiratory tract infections in adults?

- A. Legionella pneumophila

- B. Streptococcus pneumoniae

- C. Chlamydia pneumoniae

- D. Mycoplasma pneumoniae

 

**Answer: B. Streptococcus pneumoniae**

23. Question: What bacterial pathogen is responsible for causing gas gangrene, a condition characterized by the rapid destruction of muscle tissue?

- A. Clostridium difficile

- B. Clostridium perfringens

- C. Clostridium botulinum

- D. Clostridium tetani

 

 

**Answer: B. Clostridium perfringens**

24. Question: Which bacterial species is a common cause of food poisoning associated with improperly cooked poultry and eggs?

- A. Clostridium difficile

- B. Salmonella enterica

- C. Escherichia coli

- D. Campylobacter jejune

 

**Answer: B. Salmonella enterica**

25. Question: What bacterium is associated with the development of stomach ulcers and is known for its ability to survive in the acidic environment of the stomach?

- A. Escherichia coli

- B. Helicobacter pylori

- C. Campylobacter jejuni

- D. Clostridium difficile

 

**Answer: B. Helicobacter pylori**

26. Question: Which bacterial infection is characterized by the development of a characteristic skin lesion called a chancre at the site of initial infection?

- A. Syphilis

- B. Gonorrhoea

- C. Chlamydia

- D. Trichomoniasis

 

**Answer: A. Syphilis**

27. Question: What bacterial pathogen is responsible for causing Q fever, a zoonotic disease transmitted to humans from infected animals?

- A. Coxiella burnetii

- B. Francisella tularensis

- C. Rickettsia rickettsii

- D. Bartonella henselae

 

**Answer: A. Coxiella burnetii**

28. Question: Which bacterial species is associated with the development of respiratory tract infections and is a common cause of otitis media in children?

- A. Moraxella catarrhalis

- B. Staphylococcus aureus

- C. Haemophilus influenzae

- D. Streptococcus pyogenes

 

**Answer: C. Haemophilus influenzae**

29. Question: What is the primary mode of transmission for the bacterium Neisseria gonorrhoeae, causing the sexually transmitted infection gonorrhea?

- A. Airborne droplets

- B. Sexual contact

- C. Contaminated food

- D. Vector bites

 

**Answer: B. Sexual contact**

30. Question: Which bacterial pathogen is responsible for causing melioidosis, a tropical disease with symptoms ranging from pneumonia to severe septicemia?

- A. Burkholderia pseudomallei

- B. Vibrio cholerae

- C. Clostridium difficile

- D. Pseudomonas aeruginosa

 

**Answer: A. Burkholderia pseudomallei**

 

   

31. Question: What bacterial species is commonly associated with urinary tract infections and is known for its ability to form biofilms on catheters and medical devices?

- A. Escherichia coli

- B. Klebsiella pneumoniae

- C. Proteus mirabilis

- D. Enterococcus faecalis

 

**Answer: C. Proteus mirabilis**

32. Question: Which bacterium is responsible for causing typhoid fever, a systemic infection characterized by prolonged fever, abdominal pain, and constipation or diarrhea?

- A. Salmonella enterica serotype Typhi

- B. Escherichia coli

- C. Shigella dysenteriae

- D. Clostridium difficile

 

**Answer: A. Salmonella enterica serotype Typhi**

33. Question: What bacterial pathogen is commonly associated with causing community-acquired pneumonia and is often implicated in outbreaks among crowded populations?

- A. Streptococcus pyogenes

- B. Mycobacterium tuberculosis

- C. Legionella pneumophila

- D. Chlamydia pneumoniae

 

**Answer: C. Legionella pneumophila**

34. Question: Which bacterial species is a common cause of nosocomial (hospital-acquired) infections, particularly in immunocompromised patients and those with indwelling catheters?

- A. Staphylococcus aureus

- B. Pseudomonas aeruginosa

- C. Clostridium difficile

- D. Streptococcus pneumoniae

 

 

**Answer: B. Pseudomonas aeruginosa**

35. Question: What bacterium is responsible for causing brucellosis, a zoonotic disease transmitted to humans from infected animals, often through the consumption of unpasteurized dairy products?

- A. Brucella spp.

- B. Yersinia enterocolitica

- C. Coxiella burnetii

- D. Leptospira interrogans

 

 

**Answer: A. Brucella spp.**

36. Question: Which bacterial infection is characterized by the presence of "rice water" stools and is associated with contaminated water and poor sanitation?

- A. Cholera

- B. Shigellosis

- C. Campylobacteriosis

- D. Typhoid fever

 

 

**Answer: A. Cholera**

37. Question: What bacterial species is commonly associated with causing respiratory tract infections, particularly in children, and is a common cause of otitis media?

- A. Haemophilus influenzae

- B. Streptococcus pneumoniae

- C. Neisseria meningitidis

- D. Moraxella catarrhalis

 

**Answer: D. Moraxella catarrhalis**

38. Question: Which bacterial pathogen is associated with causing gastroenteritis through the consumption of contaminated food, particularly poultry and eggs?

- A. Salmonella enterica

- B. Escherichia coli

- C. Campylobacter jejuni

- D. Clostridium perfringens

 

 

**Answer: C. Campylobacter jejuni**

39. Question: What bacterial species is responsible for causing urinary tract infections and is known for its ability to produce urease, leading to the formation of urinary stones?

- A. Escherichia coli

- B. Proteus mirabilis

- C. Klebsiella pneumoniae

- D. Enterococcus faecalis

 

 

**Answer: B. Proteus mirabilis**

40. Question: Which bacterium is associated with causing melioidosis, a disease characterized by abscess formation in various organs, particularly in regions with tropical climates?

- A. Burkholderia pseudomallei

- B. Vibrio cholerae

- C. Francisella tularensis

- D. Clostridium perfringens

 

 

**Answer: A. Burkholderia pseudomallei**

41. Question: What bacterium is responsible for causing Legionnaires' disease, a severe form of pneumonia often associated with contaminated water sources, air conditioning systems, and hot tubs?

- A. Legionella pneumophila

- B. Mycobacterium avium

- C. Chlamydia pneumoniae

- D. Streptococcus pyogenes

 

 

**Answer: A. Legionella pneumophila**

42. Question: Which bacterial species is commonly associated with causing gastroenteritis through the consumption of raw or undercooked seafood, particularly oysters?

- A. Vibrio cholerae

- B. Campylobacter jejuni

- C. Escherichia coli

- D. Vibrio parahaemolyticus

 

 

**Answer: D. Vibrio parahaemolyticus**

43. Question: What bacterium is responsible for causing pertussis, also known as whooping cough, a highly contagious respiratory disease characterized by severe coughing fits?

- A. Bordetella pertussis

- B. Streptococcus pneumoniae

- C. Mycoplasma pneumoniae

- D. Haemophilus influenzae

 

 

**Answer: A. Bordetella pertussis**

44. Question: Which bacterial pathogen is associated with causing peptic ulcers and gastric cancer and is capable of surviving in the acidic environment of the stomach?

- A. Helicobacter pylori

- B. Clostridium difficile

- C. Escherichia coli

- D. Listeria monocytogenes

 

 

**Answer: A. Helicobacter pylori**

45. Question: What bacterium is responsible for causing tularemia, a zoonotic disease that can be transmitted to humans through contact with infected animals or bites from certain arthropods?

- A. Francisella tularensis

- B. Yersinia pestis

- C. Bacillus anthracis

- D. Coxiella burnetii

 

 

**Answer: A. Francisella tularensis**

46. Question: Which bacterial species is commonly associated with causing urinary tract infections in individuals with anatomical abnormalities of the urinary tract, such as vesicoureteral reflux?

- A. Escherichia coli

- B. Klebsiella pneumoniae

- C. Proteus mirabilis

- D. Enterococcus faecalis

 

 

**Answer: C. Proteus mirabilis**

47. Question: What bacterium is responsible for causing melioidosis, a potentially severe disease endemic in Southeast Asia and Northern Australia?

- A. Burkholderia pseudomallei

- B. Vibrio cholerae

- C. Clostridium perfringens

- D. Mycobacterium avium

 

 

**Answer: A. Burkholderia pseudomallei**

48. Question: Which bacterial pathogen is associated with causing cat scratch fever, a condition transmitted to humans through scratches or bites from infected cats?

- A. Bartonella henselae

- B. Coxiella burnetii

- C. Yersinia enterocolitica

- D. Rickettsia rickettsii

 

 

**Answer: A. Bartonella henselae**

49. Question: What bacterial species is responsible for causing whooping cough, especially in infants, and can lead to severe respiratory distress?

- A. Bordetella pertussis

- B. Streptococcus pneumoniae

- C. Mycoplasma pneumoniae

- D. Haemophilus influenzae

 

 

**Answer: A. Bordetella pertussis**

50. Question: Which bacterium is associated with causing leprosy, a chronic infectious disease that primarily affects the skin, peripheral nerves, and mucous membranes of the respiratory tract?

- A. Mycobacterium leprae

- B. Mycobacterium tuberculosis

- C. Mycobacterium avium

- D. Mycobacterium bovis

 

 

**Answer: A. Mycobacterium leprae**

Ø VIROLOGY MCQS 80 MCQS

1. Question: What is the genetic material of a virus?

 

A. DNA

B. RNA

C. Both DNA and RNA

D. Neither DNA nor RNA

Answer: C. Both DNA and RNA

 

2. Question: Which enzyme is responsible for replicating viral RNA into DNA in retroviruses?

 

A. Reverse transcriptase

B. RNA polymerase

C. DNA ligase

D. Helicase

Answer: A. Reverse transcriptase

 

3. Question: What is the outer protein coat of a virus called?

 

A. Capsule

B. Envelope

C. Capsid

D. Membrane

Answer: C. Capsid

 

4. Question: In which host cell organelle does the replication of many viruses take place?

 

A. Nucleus

B. Endoplasmic reticulum

C. Mitochondria

D. Golgi apparatus

Answer: A. Nucleus

 

5. Question: What is the primary function of the viral envelope?

 

A. Protection of the viral genome

B. Facilitating viral entry into host cells

C. Providing structural support

D. Enabling viral replication

Answer: B. Facilitating viral entry into host cells

 

6. Question: Which type of virus utilizes reverse transcriptase to convert its RNA genome into DNA inside the host cell?

 

A. Retrovirus

B. Orthomyxovirus

C. Picornavirus

D. Adenovirus

Answer: A. Retrovirus

 

7. Question: What is the term for a virus that infects bacteria?

 

A. Bacteriophage

B. Retrovirus

C. Viroid

D. Prion

Answer: A. Bacteriophage

 

8. Question: Which viral family does the human immunodeficiency virus (HIV) belong to?

 

A. Herpesviridae

B. Retroviridae

C. Papillomaviridae

D. Adenoviridae

Answer: B. Retroviridae

 

9. Question: What is the name of the protein coat surrounding the nucleic acid of a virus?

 

A. Capsid

B. Enzyme

C. Matrix

D. Spikes

Answer: A. Capsid

 

10. Question: What is the process by which a virus enters a host cell and releases its genetic material?

- A. Replication

- B. Transcription

- C. Translation

- D. Entry or penetration

 

makefile

Copy code

**Answer: D. Entry or penetration**

 

 

11. Question: What is the term for a virus that contains both RNA and DNA in its genome?

- A. Retrovirus

- B. Picornavirus

- C. Flavivirus

- D. Orthomyxovirus

 

**Answer: A. Retrovirus**

12. Question: Which viral enzyme is responsible for the synthesis of complementary RNA strands during the replication of RNA viruses?

- A. Reverse transcriptase

- B. RNA polymerase

- C. DNA ligase

- D. Helicase

 

**Answer: B. RNA polymerase**

13. Question: What is the name of the process by which a virus acquires an envelope from the host cell membrane during budding?

- A. Lysis

- B. Exocytosis

- C. Endocytosis

- D. Maturation

 

**Answer: B. Exocytosis**

14. Question: Which virus is associated with the development of the common cold?

- A. Influenza virus

- B. Rhinovirus

- C. Coronavirus

- D. Adenovirus

 

**Answer: B. Rhinovirus**

15. Question: What is the name of the viral enzyme responsible for cleaving and processing viral polyproteins into functional proteins during replication?

- A. Protease

- B. Ligase

- C. Polymerase

- D. Helicase

 

**Answer: A. Protease**

16. Question: Which type of virus is known for causing persistent infections and may lead to chronic diseases, such as liver cirrhosis and hepatocellular carcinoma?

- A. Retrovirus

- B. Hepadnavirus

- C. Flavivirus

- D. Herpesvirus

 

**Answer: B. Hepadnavirus**

17. Question: What is the primary target of the human papillomavirus (HPV)?

- A. Skin cells

- B. Liver cells

- C. Nervous system cells

- D. Epithelial cells

 

**Answer: D. Epithelial cells**

18. Question: Which viral family includes viruses that cause diseases such as measles, mumps, and rubella?

- A. Picornaviridae

- B. Togaviridae

- C. Paramyxoviridae

- D. Herpesviridae

 

**Answer: C. Paramyxoviridae**

19. Question: What is the primary mode of transmission for the human immunodeficiency virus (HIV)?

- A. Airborne droplets

- B. Sexual contact

- C. Contaminated food

- D. Vector bites

 

**Answer: B. Sexual contact**

20. Question: Which viral family does the influenza virus belong to?

- A. Retroviridae

- B. Orthomyxoviridae

- C. Picornaviridae

- D. Flaviviridae

 

**Answer: B. Orthomyxoviridae**

 

 

21. Question: What is the primary target of the human immunodeficiency virus (HIV) in the human body?

- A. T cells

- B. Liver cells

- C. Neurons

- D. Epithelial cells

 

 

**Answer: A. T cells**

22. Question: Which virus is responsible for causing chickenpox during primary infection and may later reappear as shingles?

- A. Herpes simplex virus type 1 (HSV-1)

- B. Herpes simplex virus type 2 (HSV-2)

- C. Varicella-zoster virus (VZV)

- D. Cytomegalovirus (CMV)

 

**Answer: C. Varicella-zoster virus (VZV)**

23. Question: What is the name of the viral enzyme responsible for unwinding the double-stranded DNA during replication?

- A. Reverse transcriptase

- B. DNA polymerase

- C. Helicase

- D. RNA polymerase

 

**Answer: C. Helicase**

24. Question: Which hepatitis virus is primarily transmitted through the fecal-oral route, causing symptoms such as jaundice and liver inflammation?

- A. Hepatitis A virus (HAV)

- B. Hepatitis B virus (HBV)

- C. Hepatitis C virus (HCV)

- D. Hepatitis D virus (HDV)

 

**Answer: A. Hepatitis A virus (HAV)**

25. Question: What is the primary mode of transmission for the Zika virus?

- A. Mosquito bites

- B. Sexual contact

- C. Blood transfusions

- D. Airborne droplets

 

**Answer: A. Mosquito bites**

26. Question: Which viral family includes the human herpesviruses, such as herpes simplex virus, varicella-zoster virus, and cytomegalovirus?

- A. Herpesviridae

- B. Adenoviridae

- C. Papillomaviridae

- D. Paramyxoviridae

 

**Answer: A. Herpesviridae**

27. Question: What is the name of the protein coat surrounding the nucleic acid of a virus, excluding the envelope?

- A. Capsid

- B. Enzyme

- C. Matrix

- D. Spikes

 

**Answer: A. Capsid**

28. Question: Which viral family includes the human immunodeficiency virus (HIV)?

- A. Retroviridae

- B. Togaviridae

- C. Flaviviridae

- D. Orthomyxoviridae

 

**Answer: A. Retroviridae**

29. Question: What is the primary mode of transmission for the Ebola virus?

- A. Mosquito bites

- B. Sexual contact

- C. Airborne droplets

- D. Contact with infected bodily fluids

 

**Answer: D. Contact with infected bodily fluids**

30. Question: Which virus is responsible for causing infectious mononucleosis (mono)?

- A. Epstein-Barr virus (EBV)

- B. Cytomegalovirus (CMV)

- C. Varicella-zoster virus (VZV)

- D. Human herpesvirus 6 (HHV-6)

 

**Answer: A. Epstein-Barr virus (EBV)**

 

 

31. Question: What is the primary mode of transmission for the rabies virus?

- A. Mosquito bites

- B. Airborne droplets

- C. Contact with infected blood

- D. Animal bites, particularly from mammals

 

**Answer: D. Animal bites, particularly from mammals**

32. Question: Which viral family includes the viruses causing dengue fever, Zika virus, and yellow fever?

- A. Flaviviridae

- B. Togaviridae

- C. Coronaviridae

- D. Picornaviridae

 

**Answer: A. Flaviviridae**

33. Question: What is the name of the phenomenon where a virus integrates its genetic material into the host cell's genome and remains latent?

- A. Lytic cycle

- B. Budding

- C. Lysogeny

- D. Transduction

 

**Answer: C. Lysogeny**

34. Question: Which virus is responsible for causing the majority of cervical cancer cases?

- A. Human papillomavirus (HPV)

- B. Herpes simplex virus (HSV)

- C. Human immunodeficiency virus (HIV)

- D. Hepatitis B virus (HBV)

 

**Answer: A. Human papillomavirus (HPV)**

35. Question: What is the name of the protein spikes on the surface of the influenza virus that determine its subtype?

- A. Capsid

- B. Hemagglutinin (HA)

- C. Neuraminidase (NA)

- D. Matrix protein

 

**Answer: B. Hemagglutinin (HA)**

36. Question: Which virus is responsible for causing hand, foot, and mouth disease in children?

- A. Coxsackievirus

- B. Respiratory syncytial virus (RSV)

- C. Rotavirus

- D. Norovirus

 

 

**Answer: A. Coxsackievirus**

37. Question: What is the primary target of the measles virus in the human body?

- A. Liver cells

- B. Respiratory epithelial cells

- C. Immune system cells

- D. Nervous system cells

 

 

**Answer: B. Respiratory epithelial cells**

38. Question: Which viral enzyme is responsible for cleaving and processing viral polyproteins into individual functional proteins during the replication of picornaviruses?

- A. Protease

- B. Reverse transcriptase

- C. RNA polymerase

- D. Helicase

 

**Answer: A. Protease**

39. Question: What is the term for the process by which a virus acquires its envelope as it exits the host cell?

- A. Exocytosis

- B. Budding

- C. Maturation

- D. Fusion

 

**Answer: B. Budding**

40. Question: Which virus is associated with causing hemorrhagic fevers such as Ebola virus disease and Marburg virus disease?

- A. Filovirus

- B. Arenavirus

- C. Flavivirus

- D. Bunyavirus

 

**Answer: A. Filovirus**

 

 

41. Question: Which virus is responsible for causing infectious hepatitis and is commonly transmitted through contaminated food and water?

- A. Hepatitis A virus (HAV)

- B. Hepatitis B virus (HBV)

- C. Hepatitis C virus (HCV)

- D. Hepatitis D virus (HDV)

 

 

**Answer: A. Hepatitis A virus (HAV)**

42. Question: What is the name of the antiviral drug commonly used to treat influenza infections by inhibiting neuraminidase?

- A. Oseltamivir (Tamiflu)

- B. Acyclovir

- C. Ribavirin

- D. Interferon

 

**Answer: A. Oseltamivir (Tamiflu)**

43. Question: Which viral family does the human cytomegalovirus (CMV) belong to?

- A. Herpesviridae

- B. Flaviviridae

- C. Coronaviridae

- D. Togaviridae

 

**Answer: A. Herpesviridae**

44. Question: What is the name of the process by which a virus transfers its genetic material from one host cell to another using a bacteriophage as a vector?

- A. Transduction

- B. Conjugation

- C. Transformation

- D. Replication

 

**Answer: A. Transduction**

45. Question: Which virus is responsible for causing the majority of cases of the common cold?

- A. Rhinovirus

- B. Influenza virus

- C. Coronavirus

- D. Adenovirus

 

**Answer: A. Rhinovirus**

46. Question: What is the name of the viral enzyme responsible for synthesizing DNA from an RNA template during retrovirus replication?

- A. RNA polymerase

- B. Reverse transcriptase

- C. DNA ligase

- D. Helicase

 

**Answer: B. Reverse transcriptase**

47. Question: Which virus is associated with causing roseola, a childhood illness characterized by a high fever and a rash that appears after the fever breaks?

- A. Varicella-zoster virus (VZV)

- B. Human herpesvirus 6 (HHV-6)

- C. Cytomegalovirus (CMV)

- D. Epstein-Barr virus (EBV)

 

**Answer: B. Human herpesvirus 6 (HHV-6)**

48. Question: Which viral family includes the viruses responsible for causing the common cold, bronchitis, and pneumonia?

- A. Picornaviridae

- B. Coronaviridae

- C. Paramyxoviridae

- D. Orthomyxoviridae

 

 

**Answer: D. Orthomyxoviridae**

49. Question: What is the name of the phenomenon where a virus enters a state of dormancy within the host cell without causing immediate harm?

- A. Latency

- B. Budding

- C. Fusion

- D. Maturation

 

**Answer: A. Latency**

50. Question: Which virus is responsible for causing severe acute respiratory syndrome (SARS)?

- A. Influenza virus

- B. Coronavirus

- C. Rhinovirus

- D. Paramyxovirus

 

**Answer: B. Coronavirus**

51. Question: Which virus is associated with causing Kaposi's sarcoma, a cancer often seen in immunocompromised individuals such as those with HIV/AIDS?

- A. Epstein-Barr virus (EBV)

- B. Human herpesvirus 8 (HHV-8)

- C. Cytomegalovirus (CMV)

- D. Varicella-zoster virus (VZV)

 

**Answer: B. Human herpesvirus 8 (HHV-8)**

52. Question: What is the primary mode of transmission for the hepatitis B virus (HBV)?

- A. Airborne droplets

- B. Sexual contact

- C. Contaminated food

- D. Contact with infected blood and body fluids

 

**Answer: D. Contact with infected blood and body fluids**

53. Question: Which virus is responsible for causing infectious gastroenteritis and is often transmitted through contaminated food and water?

- A. Norovirus

- B. Rotavirus

- C. Adenovirus

- D. Astrovirus

 

**Answer: A. Norovirus**

54. Question: What is the term for the outermost protein layer surrounding the viral genome in some viruses, contributing to protection and attachment?

- A. Capsid

- B. Matrix

- C. Envelope

- D. Spikes

 

 

**Answer: C. Envelope**

55. Question: Which virus is associated with causing hand, foot, and mouth disease, particularly in young children?

- A. Coxsackievirus

- B. Rotavirus

- C. Norovirus

- D. Adenovirus

 

makefile

Copy code

**Answer: A. Coxsackievirus**

56. Question: What is the name of the viral enzyme responsible for synthesizing DNA from an RNA template during reverse transcription in retroviruses?

- A. DNA polymerase

- B. Reverse transcriptase

- C. RNA polymerase

- D. Helicase

 

**Answer: B. Reverse transcriptase**

57. Question: Which viral family includes the viruses causing mumps and parainfluenza?

- A. Picornaviridae

- B. Togaviridae

- C. Paramyxoviridae

- D. Orthomyxoviridae

 

**Answer: C. Paramyxoviridae**

58. Question: What is the name of the glycoprotein spikes on the surface of the HIV virus that facilitate entry into host cells?

- A. Capsid

- B. Hemagglutinin (HA)

- C. Neuraminidase (NA)

- D. gp120 and gp41

 

 

**Answer: D. gp120 and gp41**

59. Question: Which virus is responsible for causing yellow fever, a disease transmitted by mosquitoes in tropical and subtropical regions?

- A. Flavivirus

- B. Togavirus

- C. Bunyavirus

- D. Arenavirus

 

 

**Answer: A. Flavivirus**

60. Question: What is the primary target of the poliovirus in the human body?

- A. Liver cells

- B. Nervous system cells

- C. Epithelial cells

- D. T cells

 

**Answer: B. Nervous system cells**

 

 

61. Question: Which virus is responsible for causing severe liver disease, including cirrhosis and hepatocellular carcinoma?

- A. Hepatitis A virus (HAV)

- B. Hepatitis B virus (HBV)

- C. Hepatitis C virus (HCV)

- D. Hepatitis E virus (HEV)

 

**Answer: C. Hepatitis C virus (HCV)**

62. Question: What is the term for the protective protein coat surrounding the nucleic acid of a virus, including the capsid and envelope?

- A. Matrix

- B. Nucleocapsid

- C. Spikes

- D. Envelope

 

**Answer: B. Nucleocapsid**

63. Question: Which viral family includes the viruses causing Hantavirus pulmonary syndrome (HPS) and hemorrhagic fever with renal syndrome (HFRS)?

- A. Flaviviridae

- B. Arenaviridae

- C. Bunyaviridae

- D. Togaviridae

 

**Answer: C. Bunyaviridae**

64. Question: What is the name of the protein responsible for the attachment of a virus to host cells during the initial stages of infection?

- A. Capsid

- B. Matrix

- C. Envelope

- D. Receptor-binding protein

 

**Answer: D. Receptor-binding protein**

65. Question: Which virus is responsible for causing respiratory syncytial virus (RSV) infections, particularly in infants and young children?

- A. Influenza virus

- B. Parainfluenza virus

- C. Respiratory syncytial virus (RSV)

- D. Rhinovirus

 

 

**Answer: C. Respiratory syncytial virus (RSV)**

66. Question: What is the name of the viral enzyme responsible for cutting host cell mRNA during the process of cap-snatching in influenza virus replication?

- A. Protease

- B. Neuraminidase

- C. RNA polymerase

- D. Endonuclease

 

 

**Answer: D. Endonuclease**

67. Question: Which virus is associated with causing roseola infantum, a common childhood illness characterized by a high fever and a rash that appears after the fever subsides?

- A. Varicella-zoster virus (VZV)

- B. Human herpesvirus 6 (HHV-6)

- C. Cytomegalovirus (CMV)

- D. Epstein-Barr virus (EBV)

 

**Answer: B. Human herpesvirus 6 (HHV-6)**

68. Question: What is the primary mode of transmission for the mumps virus?

- A. Airborne droplets

- B. Sexual contact

- C. Contaminated food

- D. Respiratory secretions

 

 

**Answer: D. Respiratory secretions**

69. Question: Which viral family includes the viruses causing SARS-CoV-2 (COVID-19)?

- A. Coronaviridae

- B. Orthomyxoviridae

- C. Paramyxoviridae

- D. Filoviridae

 

 

**Answer: A. Coronaviridae**

70. Question: What is the name of the process by which enveloped viruses enter host cells by merging with the host cell membrane?

- A. Endocytosis

- B. Budding

- C. Fusion

- D. Lysis

 

 

**Answer: C. Fusion**

 

 

71. Question: Which virus is responsible for causing mononucleosis-like symptoms and is associated with the development of chronic fatigue syndrome (CFS)?

- A. Epstein-Barr virus (EBV)

- B. Cytomegalovirus (CMV)

- C. Human herpesvirus 8 (HHV-8)

- D. Varicella-zoster virus (VZV)

 

 

**Answer: B. Cytomegalovirus (CMV)**

72. Question: What is the primary mode of transmission for the West Nile virus?

- A. Mosquito bites

- B. Sexual contact

- C. Airborne droplets

- D. Contaminated food

 

 

**Answer: A. Mosquito bites**

73. Question: Which viral family includes the viruses causing the common cold and bronchiolitis in infants?

- A. Coronaviridae

- B. Adenoviridae

- C. Picornaviridae

- D. Paramyxoviridae

 

 

**Answer: B. Adenoviridae**

74. Question: What is the term for the process by which viral RNA is translated directly into protein without the need for reverse transcription?

- A. Transcription

- B. Translation

- C. Replication

- D. Reverse transcription

 

 

**Answer: B. Translation**

75. Question: Which virus is associated with causing a severe respiratory illness known as Middle East Respiratory Syndrome (MERS)?

- A. Influenza virus

- B. Coronavirus

- C. Rhinovirus

- D. Paramyxovirus

 

 

**Answer: B. Coronavirus**

76. Question: What is the name of the viral enzyme responsible for cleaving and processing viral polyproteins into individual functional proteins during the replication of coronaviruses?

- A. Protease

- B. Polymerase

- C. Helicase

- D. Reverse transcriptase

 

 

**Answer: A. Protease**

77. Question: Which virus is responsible for causing German measles or rubella?

- A. Rubi virus

- B. Parvovirus

- C. Rubula virus

- D. Togavirus

 

**Answer: A. Rubi virus**

78. Question: What is the primary target of the measles virus in the human body?

- A. Liver cells

- B. Respiratory epithelial cells

- C. Immune system cells

- D. Nervous system cells

 

 

**Answer: B. Respiratory epithelial cells**

79. Question: Which viral family includes the viruses causing cold sores, genital herpes, and chickenpox?

- A. Herpesviridae

- B. Adenoviridae

- C. Togaviridae

- D. Picornaviridae

 

 

**Answer: A. Herpesviridae**

80. Question: What is the name of the viral enzyme responsible for copying RNA into DNA during reverse transcription in retroviruses?

- A. Reverse transcriptase

- B. RNA polymerase

- C. DNA ligase

- D. Helicase

 

 

**Answer: A. Reverse transcriptase**

Ø IMMUNOLOGY MCQS 100

 

1. Question: What is the primary function of antibodies in the immune system?

 

A. Phagocytosis

B. Cytokine production

C. Antigen presentation

D. Binding to antigens and neutralization

Answer: D. Binding to antigens and neutralization

 

2. Question: Which cells are responsible for presenting antigens to T cells in the immune system?

 

A. B cells

B. Macrophages

C. Neutrophils

D. Natural killer cells

Answer: B. Macrophages

 

3. Question: What is the primary role of T cells in the immune response?

 

A. Antibody production

B. Phagocytosis

C. Cell-mediated immunity

D. Complement activation

Answer: C. Cell-mediated immunity

 

4. Question: Which immunoglobulin is involved in allergic reactions and is found on the surface of mast cells and basophils?

 

A. IgA

B. IgD

C. IgE

D. IgM

Answer: C. IgE

 

5. Question: What is the main function of cytotoxic T cells?

 

A. Antibody production

B. Phagocytosis

C. Killing infected cells

D. Cytokine release

Answer: C. Killing infected cells

 

6. Question: Which type of immunity is acquired through vaccination or exposure to an infectious agent?

 

A. Innate immunity

B. Passive immunity

C. Adaptive immunity

D. Natural immunity

Answer: C. Adaptive immunity

 

7. Question: What is the role of the thymus in the immune system?

 

A. Antibody production

B. Maturation of T cells

C. Maturation of B cells

D. Phagocytosis

Answer: B. Maturation of T cells

 

8. Question: Which cells are responsible for producing antibodies in the immune system?

 

A. T cells

B. B cells

C. Natural killer cells

D. Macrophages

Answer: B. B cells

 

9. Question: What is the process by which immune cells engulf and digest foreign particles or cells?

 

A. Phagocytosis

B. Exocytosis

C. Endocytosis

D. Pinocytosis

Answer: A. Phagocytosis

 

10. Question: Which type of immunity is provided by the transfer of antibodies from mother to infant through breast milk or placenta?

- A. Innate immunity

- B. Passive immunity

- C. Adaptive immunity

- D. Acquired immunity

 

 

**Answer: B. Passive immunity**

 

11. Question: What is the primary function of helper T cells in the immune system?

 

A. Phagocytosis

B. Killing infected cells

C. Coordinating immune responses

D. Producing antibodies

Answer: C. Coordinating immune responses

 

12. Question: Which molecule is responsible for antigen recognition by T cells?

 

A. Immunoglobulin

B. Major histocompatibility complex (MHC)

C. Complement

D. Cytokine

Answer: B. Major histocompatibility complex (MHC)

 

13. Question: Which cell type is involved in the regulation of immune responses and the prevention of autoimmune reactions?

 

A. Helper T cells

B. Regulatory T cells

C. Cytotoxic T cells

D. Memory B cells

Answer: B. Regulatory T cells

 

14. Question: What is the role of dendritic cells in the immune system?

 

A. Antibody production

B. Phagocytosis

C. Killing infected cells

D. Antigen presentation

Answer: D. Antigen presentation

 

15. Question: Which immunoglobulin is the most abundant in blood and is the first to appear during an immune response?

 

A. IgA

B. IgD

C. IgG

D. IgM

Answer: D. IgM

 

16. Question: What is the function of memory cells in the immune system?

 

A. Immediate response to infections

B. Long-term protection against specific pathogens

C. Phagocytosis of foreign particles

D. Activation of complement system

Answer: B. Long-term protection against specific pathogens

 

17. Question: Which type of immunity is conferred by the transfer of sensitized T cells from a donor to a recipient?

 

A. Passive immunity

B. Cell-mediated immunity

C. Adaptive immunity

D. Innate immunity

Answer: B. Cell-mediated immunity

 

18. Question: Which cells are responsible for releasing histamine and mediating immediate hypersensitivity reactions?

 

A. T cells

B. B cells

C. Mast cells

D. Macrophages

Answer: C. Mast cells

 

19. Question: What is the primary function of natural killer (NK) cells in the immune system?

 

A. Phagocytosis

B. Antibody production

C. Killing infected and cancerous cells

D. Antigen presentation

Answer: C. Killing infected and cancerous cells

 

20. Question: Which cell type is responsible for producing cytokines that modulate the activity of other immune cells?

 

A. B cells

B. T cells

C. Natural killer cells

D. Macrophages

Answer: B. T cells

 

21. Question: Which cytokine is involved in promoting inflammation and is often elevated during allergic reactions?

 

A. Interleukin-10 (IL-10)

B. Tumour necrosis factor-alpha (TNF-alpha)

C. Interferon-gamma (IFN-gamma)

D. Interleukin-4 (IL-4)

Answer: B. Tumour necrosis factor-alpha (TNF-alpha)

 

22. Question: What is the term for the process by which antibodies and complement proteins coat microorganisms, making them more susceptible to phagocytosis?

 

A. Opsonization

B. Complementation

C. Agglutination

D. Neutralization

Answer: A. Opsonization

 

23. Question: Which type of hypersensitivity reaction involves IgE antibodies and mast cells, leading to immediate allergic responses?

 

A. Type I

B. Type II

C. Type III

D. Type IV

Answer: A. Type I

 

24. Question: What is the primary function of the complement system in the immune response?

 

A. Phagocytosis

B. Cytolysis

C. Antibody production

D. T cell activation

Answer: B. Cytolysis

 

25. Question: Which cell type plays a crucial role in the surveillance for and destruction of virus-infected cells in a non-specific manner?

 

A. T cells

B. B cells

C. Natural killer (NK) cells

D. Macrophages

Answer: C. Natural killer (NK) cells

 

26. Question: What is the name of the process by which T cells and B cells undergo a maturation and selection process to ensure self-tolerance?

 

A. Clonal expansion

B. Positive selection

C. Negative selection

D. Somatic recombination

Answer: C. Negative selection

 

27. Question: Which immunoglobulin is primarily found in mucosal secretions, such as saliva and tears?

 

A. IgA

B. IgD

C. IgE

D. IgG

Answer: A. IgA

 

28. Question: What is the function of the major histocompatibility complex (MHC) in the immune system?

 

A. Antibody production

B. Antigen presentation

C. Phagocytosis

D. Cytokine release

Answer: B. Antigen presentation

 

29. Question: Which type of T cell is responsible for suppressing immune responses to prevent autoimmunity?

 

A. Helper T cell

B. Cytotoxic T cell

C. Memory T cell

D. Regulatory T cell

Answer: D. Regulatory T cell

 

30. Question: What is the name of the process by which immune cells move toward a site of infection or injury guided by chemical signals?

 

A. Phagocytosis

B. Chemotaxis

C. Diapedesis

D. Apoptosis

Answer: B. Chemotaxis

 

31. Question: Which immunoglobulin class is the first to be produced during an immune response and is often associated with primary infections?

 

A. IgA

B. IgD

C. IgM

D. IgG

Answer: C. IgM

 

32. Question: What is the function of the thymus in the context of T cell development?

 

A. Maturation of B cells

B. Production of antibodies

C. Maturation of T cells

D. Activation of complement system

Answer: C. Maturation of T cells

 

33. Question: Which type of T cell is primarily responsible for activating B cells and promoting antibody production?

 

A. Helper T cell

B. Cytotoxic T cell

C. Memory T cell

D. Regulatory T cell

Answer: A. Helper T cell

 

34. Question: What is the primary function of antibodies in the opsonization process?

 

A. Inhibition of viral replication

B. Neutralization of toxins

C. Facilitation of phagocytosis

D. Activation of complement

Answer: C. Facilitation of phagocytosis

 

35. Question: Which cell type is responsible for presenting antigens to B cells and initiating the antibody response?

 

A. Macrophages

B. Dendritic cells

C. T cells

D. Natural killer cells

Answer: B. Dendritic cells

 

36. Question: What is the role of memory B cells in the immune system?

 

A. Immediate response to infections

B. Long-term protection against specific pathogens

C. Phagocytosis of foreign particles

D. Activation of complement system

Answer: B. Long-term protection against specific pathogens

 

37. Question: Which cytokine is involved in the differentiation of T cells into cytotoxic T cells during the immune response?

 

A. Interleukin-2 (IL-2)

B. Interleukin-4 (IL-4)

C. Interferon-gamma (IFN-gamma)

D. Tumor necrosis factor-alpha (TNF-alpha)

Answer: A. Interleukin-2 (IL-2)

 

38. Question: What is the function of the spleen in the immune system?

 

A. Maturation of T cells

B. Production of antibodies

C. Filtering and monitoring blood for pathogens

D. Activation of complement system

Answer: C. Filtering and monitoring blood for pathogens

 

39. Question: Which type of hypersensitivity reaction involves the formation of immune complexes that deposit in tissues and cause inflammation?

 

A. Type I

B. Type II

C. Type III

D. Type IV

Answer: C. Type III

 

40. Question: What is the primary function of cytotoxic T cells in the immune response?

- A. Antibody production

- B. Phagocytosis

- C. Killing infected cells

- D. Cytokine release

 

**Answer: C. Killing infected cells**

 

41. Question: What is the term for the phenomenon where a small initial exposure to an antigen leads to a more robust immune response upon subsequent exposures?

 

A. Primary response

B. Tolerance

C. Memory response

D. Secondary response

Answer: D. Secondary response

 

42. Question: Which cells are responsible for producing antibodies with high specificity for a particular antigen?

 

A. Plasma cells

B. Memory B cells

C. Regulatory T cells

D. Helper T cells

Answer: A. Plasma cells

 

43. Question: Which class of antibodies can cross the placenta and provide passive immunity to the fetus?

 

A. IgA

B. IgD

C. IgG

D. IgM

Answer: C. IgG

 

44. Question: What is the primary function of complement proteins in the immune system?

 

A. Antibody production

B. Phagocytosis

C. Opsonization

D. Cytolysis

Answer: D. Cytolysis

 

45. Question: Which cell type plays a central role in the rejection of transplanted organs and tissues?

 

A. Helper T cells

B. Regulatory T cells

C. Memory B cells

D. Cytotoxic T cells

Answer: D. Cytotoxic T cells

 

46. Question: What is the term for the ability of the immune system to distinguish self from non-self?

 

A. Immune tolerance

B. Autoimmunity

C. Immune surveillance

D. Immunodeficiency

Answer: A. Immune tolerance

 

47. Question: Which cytokine is primarily responsible for promoting inflammation and attracting immune cells to the site of infection or injury?

 

A. Interleukin-10 (IL-10)

B. Tumor necrosis factor-alpha (TNF-alpha)

C. Interferon-gamma (IFN-gamma)

D. Interleukin-4 (IL-4)

Answer: C. Interferon-gamma (IFN-gamma)

 

48. Question: What is the main function of the complement cascade in the immune response?

 

A. Activation of B cells

B. Cytolysis of pathogens

C. Phagocytosis

D. Antigen presentation

Answer: B. Cytolysis of pathogens

 

49. Question: Which cells are responsible for the production of antibodies with diverse specificities during an immune response?

 

A. Plasma cells

B. Memory T cells

C. Helper T cells

D. Regulatory T cells

Answer: A. Plasma cells

 

50. Question: What is the term for the ability of an antigen to stimulate an immune response?

 

A. Immunogenicity

B. Antigenicity

C. Tolerance

D. Specificity

Answer: A. Immunogenicity

  

 

51. Question: Which immunoglobulin class is found on the surface of B cells and serves as a receptor for antigen recognition?

 

A. IgA

B. IgD

C. IgE

D. IgM

Answer: B. IgD

 

52. Question: What is the primary function of the spleen in the immune system?

 

A. Antibody production

B. T cell maturation

C. Filtering and monitoring blood for pathogens

D. Cytokine release

Answer: C. Filtering and monitoring blood for pathogens

 

53. Question: Which cells are responsible for presenting antigens to CD8+ cytotoxic T cells during the immune response?

 

A. Helper T cells

B. Macrophages

C. Dendritic cells

D. B cells

Answer: C. Dendritic cells

 

54. Question: What is the role of toll-like receptors (TLRs) in the innate immune response?

 

A. Phagocytosis

B. Activation of complement

C. Antigen presentation

D. Recognition of pathogen-associated molecular patterns (PAMPs)

Answer: D. Recognition of pathogen-associated molecular patterns (PAMPs)

 

55. Question: Which immunoglobulin class is mainly involved in mucosal immunity and is present in tears, saliva, and breast milk?

 

A. IgA

B. IgD

C. IgE

D. IgM

Answer: A. IgA

 

56. Question: What is the process by which immune cells undergo programmed cell death, contributing to the regulation of immune responses?

 

A. Apoptosis

B. Necrosis

C. Phagocytosis

D. Autophagy

Answer: A. Apoptosis

 

57. Question: Which immunoglobulin class is associated with allergic reactions and plays a role in defending against parasites?

 

A. IgA

B. IgD

C. IgE

D. IgM

Answer: C. IgE

 

58. Question: What is the primary function of regulatory T cells in the immune system?

 

A. Activation of cytotoxic T cells

B. Suppression of immune responses to prevent autoimmunity

C. Production of antibodies

D. Phagocytosis

Answer: B. Suppression of immune responses to prevent autoimmunity

 

59. Question: Which type of hypersensitivity reaction involves delayed responses mediated by T cells, causing tissue damage?

 

A. Type I

B. Type II

C. Type III

D. Type IV

Answer: D. Type IV

 

60. Question: What is the term for the process by which antibodies and complement proteins coat pathogens, preventing their attachment to host cells?

- A. Opsonization

- B. Neutralization

- C. Agglutination

- D. Complementation

 

**Answer: B. Neutralization**

 

61. Question: What is the primary function of memory T cells in the immune system?

 

A. Immediate response to infections

B. Long-term protection against specific pathogens

C. Phagocytosis of foreign particles

D. Activation of complement system

Answer: B. Long-term protection against specific pathogens

 

62. Question: Which type of immunity is characterized by the transfer of antibodies from one individual to another, providing immediate but temporary protection?

 

A. Innate immunity

B. Passive immunity

C. Adaptive immunity

D. Active immunity

Answer: B. Passive immunity

 

63. Question: What is the primary function of interleukins in the immune system?

 

A. Phagocytosis

B. Antibody production

C. Communication between immune cells

D. Cytolysis

Answer: C. Communication between immune cells

 

64. Question: Which cell type is responsible for presenting antigens to CD4+ helper T cells during the immune response?

 

A. Macrophages

B. B cells

C. Dendritic cells

D. Natural killer cells

Answer: C. Dendritic cells

 

65. Question: What is the role of the complement system in inflammation?

 

A. Inhibition of inflammation

B. Phagocytosis

C. Cytolysis

D. Suppression of cytokine release

Answer: B. Phagocytosis

 

66. Question: Which type of cells produce antibodies with a high affinity for specific antigens during an immune response?

 

A. Plasma cells

B. Memory B cells

C. Regulatory T cells

D. Helper T cells

Answer: A. Plasma cells

 

67. Question: What is the term for the process by which T cells recognize antigens presented by major histocompatibility complex (MHC) molecules?

 

A. Clonal expansion

B. Antigen presentation

C. Somatic recombination

D. T cell activation

Answer: D. T cell activation

 

68. Question: Which class of antibodies is involved in the neutralization of toxins produced by bacteria?

 

A. IgA

B. IgD

C. IgG

D. IgM

Answer: C. IgG

 

69. Question: What is the role of macrophages in the immune system?

 

A. Antibody production

B. Phagocytosis

C. Activation of T cells

D. Mast cell degranulation

Answer: B. Phagocytosis

 

70. Question: Which cytokine is responsible for promoting the differentiation of B cells into plasma cells during the immune response?

- A. Interleukin-2 (IL-2)

- B. Interleukin-4 (IL-4)

- C. Interleukin-10 (IL-10)

- D. Tumor necrosis factor-alpha (TNF-alpha)

 

**Answer: B. Interleukin-4 (IL-4)**

71. Question: What is the primary function of perforin in the immune response?

 

A. Inhibition of viral replication

B. Neutralization of toxins

C. Facilitation of phagocytosis

D. Induction of cytolysis in target cells

Answer: D. Induction of cytolysis in target cells

 

72. Question: Which immunoglobulin class is involved in the mucosal immune response and is found in high concentrations in saliva and respiratory secretions?

 

A. IgA

B. IgD

C. IgE

D. IgG

Answer: A. IgA

 

73. Question: What is the term for the process by which immune cells move from the bloodstream into tissues in response to inflammatory signals?

 

A. Phagocytosis

B. Chemotaxis

C. Diapedesis

D. Apoptosis

Answer: C. Diapedesis

 

74. Question: Which type of T cell plays a role in suppressing immune responses and preventing excessive reactions?

 

A. Helper T cell

B. Cytotoxic T cell

C. Memory T cell

D. Regulatory T cell

Answer: D. Regulatory T cell

 

75. Question: What is the primary function of the complement component C3b in the immune system?

 

A. Opsonization

B. Neutralization

C. Cytolysis

D. Antibody production

Answer: A. Opsonization

 

76. Question: Which cells are responsible for the synthesis and secretion of antibodies?

 

A. T cells

B. B cells

C. Natural killer cells

D. Macrophages

Answer: B. B cells

 

77. Question: What is the role of the immunoglobulin IgD in the immune system?

 

A. Opsonization

B. Antigen presentation

C. Mast cell degranulation

D. Receptor on the surface of B cells

Answer: D. Receptor on the surface of B cells

 

78. Question: Which cytokine is involved in the differentiation of B cells into plasma cells during the immune response?

 

A. Interleukin-2 (IL-2)

B. Interleukin-4 (IL-4)

C. Interleukin-10 (IL-10)

D. Tumor necrosis factor-alpha (TNF-alpha)

Answer: B. Interleukin-4 (IL-4)

 

79. Question: What is the main function of the natural killer (NK) cells in the immune system?

 

A. Phagocytosis

B. Antibody production

C. Killing infected and cancerous cells

D. Antigen presentation

Answer: C. Killing infected and cancerous cells

 

80. Question: Which immunoglobulin class is primarily responsible for providing passive immunity to newborns through breastfeeding?

- A. IgA

- B. IgD

- C. IgE

- D. IgG

 

**Answer: D. IgG**

 

81. Question: What is the primary function of the CD4 co-receptor on T cells?

 

A. Antigen recognition

B. Activation of cytotoxic T cells

C. Binding to MHC class I molecules

D. Interaction with MHC class II molecules

Answer: D. Interaction with MHC class II molecules

 

82. Question: Which type of immunity is provided by the transfer of sensitized T cells from a donor to a recipient?

 

A. Passive immunity

B. Cell-mediated immunity

C. Innate immunity

D. Adaptive immunity

Answer: B. Cell-mediated immunity

 

83. Question: What is the role of the complement component C5a in the immune response?

 

A. Opsonization

B. Chemotaxis

C. Cytolysis

D. Neutralization

Answer: B. Chemotaxis

 

84. Question: Which type of T cell is primarily responsible for directly killing infected host cells?

 

A. Helper T cell

B. Cytotoxic T cell

C. Regulatory T cell

D. Memory T cell

Answer: B. Cytotoxic T cell

 

85. Question: What is the function of antibodies in the process of agglutination?

 

A. Facilitation of phagocytosis

B. Neutralization of toxins

C. Binding to antigens and forming clumps

D. Activation of complement

Answer: C. Binding to antigens and forming clumps

 

86. Question: Which cells are responsible for presenting antigens to CD8+ cytotoxic T cells during the immune response?

 

A. Helper T cells

B. Macrophages

C. Dendritic cells

D. B cells

Answer: C. Dendritic cells

 

87. Question: What is the term for the process by which antibodies enhance the phagocytosis of microorganisms by immune cells?

 

A. Opsonization

B. Neutralization

C. Agglutination

D. Complementation

Answer: A. Opsonization

 

88. Question: Which immunoglobulin class is the most abundant in the mucous membranes and provides localized defense against pathogens?

 

A. IgA

B. IgD

C. IgE

D. IgM

Answer: A. IgA

 

89. Question: What is the primary function of Tumor Necrosis Factor-alpha (TNF-alpha) in the immune response?

 

A. Activation of B cells

B. Chemotaxis of immune cells

C. Cytolysis of pathogens

D. Suppression of inflammation

Answer: B. Chemotaxis of immune cells

 

90. Question: Which cell type is responsible for the secretion of antibodies in response to a secondary exposure to an antigen?

- A. Plasma cells

- B. Memory B cells

- C. Regulatory T cells

- D. Helper T cells

 

 

**Answer: A. Plasma cells**

 

91. Question: What is the main function of IgE antibodies in the immune response?

 

A. Opsonization

B. Neutralization of toxins

C. Induction of phagocytosis

D. Mediation of allergic reactions

Answer: D. Mediation of allergic reactions

 

92. Question: Which cell type is primarily responsible for presenting antigens to CD4+ helper T cells during the immune response?

 

A. Macrophages

B. B cells

C. Dendritic cells

D. Natural killer cells

Answer: A. Macrophages

 

93. Question: What is the term for the phenomenon where antibodies and antigens form insoluble complexes that precipitate out of solution?

 

A. Opsonization

B. Agglutination

C. Complementation

D. Neutralization

Answer: B. Agglutination

 

94. Question: Which immunoglobulin class is involved in the defense against parasitic infections and allergic responses?

 

A. IgA

B. IgD

C. IgE

D. IgM

Answer: C. IgE

 

95. Question: What is the primary function of toll-like receptors (TLRs) in the innate immune response?

 

A. Phagocytosis

B. Activation of complement

C. Antigen presentation

D. Recognition of pathogen-associated molecular patterns (PAMPs)

Answer: D. Recognition of pathogen-associated molecular patterns (PAMPs)

 

96. Question: Which cytokine is involved in promoting the differentiation of B cells into memory B cells during the immune response?

 

A. Interleukin-2 (IL-2)

B. Interleukin-4 (IL-4)

C. Interleukin-10 (IL-10)

D. Tumor necrosis factor-alpha (TNF-alpha)

Answer: B. Interleukin-4 (IL-4)

 

97. Question: What is the term for the process by which antibodies prevent the attachment of pathogens to host cells?

 

A. Opsonization

B. Neutralization

C. Agglutination

D. Complementation

Answer: B. Neutralization

 

98. Question: Which immunoglobulin class is found on the surface of mast cells and basophils and is involved in allergic reactions?

 

A. IgA

B. IgD

C. IgE

D. IgM

Answer: C. IgE

 

99. Question: What is the primary function of the Major Histocompatibility Complex (MHC) in the immune system?

 

A. Phagocytosis

B. Antigen presentation

C. Complement activation

D. Cytokine release

Answer: B. Antigen presentation

 

100. Question: Which cell type is responsible for the production of antibodies with high specificity for a particular antigen during the secondary immune response?

- A. Plasma cells

- B. Memory B cells

- C. Regulatory T cells

- D. Helper T cells

 

**Answer: B. Memory B cells**

Ø CYTOLOGY 80 MCQS

 

1. Question: What is the primary function of the endoplasmic reticulum (ER) in a cell?

 

A. Synthesis of ribosomal RNA

B. Lipid synthesis and detoxification

C. Protein synthesis

D. ATP production

Answer: C. Protein synthesis

 

2. Question: Which cellular organelle is responsible for the breakdown of cellular waste and damaged organelles?

 

A. Golgi apparatus

B. Lysosome

C. Endoplasmic reticulum

D. Nucleus

Answer: B. Lysosome

 

3. Question: What is the main function of the nucleus in a eukaryotic cell?

 

A. ATP synthesis

B. Cellular respiration

C. Storage of genetic information

D. Protein synthesis

Answer: C. Storage of genetic information

 

4. Question: Which structure is responsible for the production of ribosomal RNA (rRNA) in a cell?

 

A. Nucleolus

B. Mitochondria

C. Endoplasmic reticulum

D. Golgi apparatus

Answer: A. Nucleolus

 

5. Question: In which phase of the cell cycle does DNA replication occur?

 

A. G1 phase

B. S phase

C. G2 phase

D. M phase

Answer: B. S phase

 

6. Question: What is the primary function of the mitochondria in a cell?

 

A. Synthesis of proteins

B. Energy production (ATP synthesis)

C. Modifying and packaging proteins

D. Lipid synthesis

Answer: B. Energy production (ATP synthesis)

 

7. Question: Which cellular structure is responsible for maintaining cell shape, providing structural support, and facilitating cell movement?

 

A. Microtubules

B. Microfilaments

C. Intermediate filaments

D. Centrioles

Answer: A. Microtubules

 

8. Question: What is the role of the Golgi apparatus in a eukaryotic cell?

 

A. Protein synthesis

B. Lipid synthesis

C. Modification and packaging of proteins

D. Energy production

Answer: C. Modification and packaging of proteins

 

9. Question: Which cellular structure is involved in the separation of chromosomes during cell division?

 

A. Centriole

B. Lysosome

C. Nucleus

D. Endoplasmic reticulum

Answer: A. Centriole

 

10. Question: What is the function of the cell membrane (plasma membrane) in a cell?

- A. Control of substances entering and leaving the cell

- B. Synthesis of ATP

- C. Storage of genetic information

- D. Protein synthesis

 

makefile

Copy code

**Answer: A. Control of substances entering and leaving the cell**

 

11. Question: What is the primary function of the smooth endoplasmic reticulum (SER) in a cell?

 

A. Protein synthesis

B. Lipid synthesis and detoxification

C. Ribosomal RNA synthesis

D. ATP production

Answer: B. Lipid synthesis and detoxification

 

12. Question: Which cellular organelle is responsible for generating energy through cellular respiration?

 

A. Lysosome

B. Nucleus

C. Mitochondria

D. Golgi apparatus

Answer: C. Mitochondria

 

13. Question: What is the main function of peroxisomes in a eukaryotic cell?

 

A. Protein synthesis

B. Lipid synthesis

C. Detoxification and breakdown of fatty acids

D. Ribosomal RNA synthesis

Answer: C. Detoxification and breakdown of fatty acids

 

14. Question: In which phase of the cell cycle does cell division (mitosis) occur?

 

A. G1 phase

B. S phase

C. G2 phase

D. M phase

Answer: D. M phase

 

15. Question: What is the function of the cytoskeleton in a cell?

 

A. Storage of genetic information

B. Synthesis of ATP

C. Structural support, cell shape, and movement

D. Modification and packaging of proteins

Answer: C. Structural support, cell shape, and movement

 

16. Question: Which organelle is involved in the synthesis of cellular proteins?

 

A. Golgi apparatus

B. Lysosome

C. Endoplasmic reticulum

D. Nucleolus

Answer: C. Endoplasmic reticulum

 

17. Question: What is the role of centrosomes in cell division?

 

A. Synthesis of ribosomal RNA

B. Formation of microtubules and organization of spindle fibers

C. Lipid synthesis

D. Protein synthesis

Answer: B. Formation of microtubules and organization of spindle fibers

 

18. Question: Which cellular structure is involved in the transport of materials within the cell and between the cell and its environment?

 

A. Centriole

B. Nucleus

C. Endoplasmic reticulum

D. Vesicles and the endomembrane system

Answer: D. Vesicles and the endomembrane system

 

19. Question: What is the function of the nuclear envelope in a eukaryotic cell?

 

A. Control of substances entering and leaving the cell

B. Synthesis of ATP

C. Separation of chromosomes during cell division

D. Enclosing and protecting the nucleus

Answer: D. Enclosing and protecting the nucleus

 

20. Question: What is the term for the movement of water across a selectively permeable membrane from an area of lower solute concentration to an area of higher solute concentration?

- A. Osmosis

- B. Active transport

- C. Diffusion

- D. Facilitated diffusion

 

makefile

Copy code

**Answer: A. Osmosis**

 

21. Question: What is the primary function of the nucleolus in a cell?

 

A. Synthesis of ATP

B. Storage of genetic information

C. Production of ribosomal RNA (rRNA)

D. Detoxification

Answer: C. Production of ribosomal RNA (rRNA)

 

22. Question: Which cellular structure is responsible for the synthesis and packaging of secretory vesicles?

 

A. Centriole

B. Nucleus

C. Golgi apparatus

D. Smooth endoplasmic reticulum

Answer: C. Golgi apparatus

 

23. Question: What is the term for the process by which cells engulf large particles or other cells by wrapping pseudopodia around them?

 

A. Phagocytosis

B. Pinocytosis

C. Exocytosis

D. Endocytosis

Answer: A. Phagocytosis

 

24. Question: Which organelle contains enzymes involved in the breakdown of cellular debris and foreign substances, acting as the "digestive system" of the cell?

 

A. Peroxisome

B. Lysosome

C. Endoplasmic reticulum

D. Golgi apparatus

Answer: B. Lysosome

 

25. Question: In which phase of the cell cycle does the cell prepare for mitosis, and cellular organelles duplicate?

 

A. G1 phase

B. S phase

C. G2 phase

D. M phase

Answer: C. G2 phase

 

26. Question: What is the function of microfilaments in the cytoskeleton of a cell?

 

A. Structural support

B. Formation of cilia and flagella

C. Facilitating cell movement

D. Storage of genetic information

Answer: C. Facilitating cell movement

 

27. Question: Which cellular structure is involved in the synthesis of steroid hormones and detoxification of drugs and poisons?

 

A. Lysosome

B. Peroxisome

C. Smooth endoplasmic reticulum (SER)

D. Nucleus

Answer: C. Smooth endoplasmic reticulum (SER)

 

28. Question: What is the term for the semi-fluid substance that fills the interior of the cell and surrounds organelles?

 

A. Cytoskeleton

B. Cytosol

C. Cytoplasm

D. Nucleoplasm

Answer: C. Cytoplasm

 

29. Question: Which cellular structure is responsible for the synthesis of ribosomal RNA and the assembly of ribosomes?

 

A. Nucleolus

B. Endoplasmic reticulum

C. Golgi apparatus

D. Mitochondria

Answer: A. Nucleolus

 

30. Question: What is the function of the nuclear pore complex in a eukaryotic cell?

- A. Facilitating the movement of ions across the nuclear envelope

- B. Regulating the passage of molecules between the nucleus and cytoplasm

- C. Synthesizing ribosomal RNA

- D. Detoxification processes

 

makefile

Copy code

**Answer: B. Regulating the passage of molecules between the nucleus and cytoplasm**

 

31. Question: What is the function of ribosomes in a cell?

 

A. Synthesis of lipids

B. ATP production

C. Protein synthesis

D. Detoxification

Answer: C. Protein synthesis

 

32. Question: Which cellular structure is responsible for the regulation of gene expression and the synthesis of RNA?

 

A. Nucleolus

B. Endoplasmic reticulum

C. Nucleus

D. Golgi apparatus

Answer: C. Nucleus

 

33. Question: What is the primary function of the cytosol in a eukaryotic cell?

 

A. Energy production

B. Protein synthesis

C. Cellular respiration

D. Storage of genetic information

Answer: B. Protein synthesis

 

34. Question: In which organelle does cellular respiration occur, leading to the production of ATP?

 

A. Golgi apparatus

B. Lysosome

C. Mitochondria

D. Peroxisome

Answer: C. Mitochondria

 

35. Question: What is the term for the movement of molecules from an area of higher concentration to an area of lower concentration, without the use of energy?

 

A. Osmosis

B. Active transport

C. Diffusion

D. Facilitated diffusion

Answer: C. Diffusion

 

36. Question: Which cellular structure is involved in the formation of spindle fibers during cell division?

 

A. Centriole

B. Nucleus

C. Endoplasmic reticulum

D. Golgi apparatus

Answer: A. Centriole

 

37. Question: What is the role of the nuclear membrane in a eukaryotic cell?

 

A. Synthesis of ATP

B. Regulation of gene expression

C. Separation of chromosomes during cell division

D. Control of substances entering and leaving the nucleus

Answer: D. Control of substances entering and leaving the nucleus

 

38. Question: Which organelle is responsible for the synthesis of phospholipids and steroids?

 

A. Nucleolus

B. Endoplasmic reticulum

C. Golgi apparatus

D. Peroxisome

Answer: B. Endoplasmic reticulum

 

39. Question: What is the function of microtubules in the cytoskeleton of a cell?

 

A. Structural support

B. Facilitating cell movement

C. Formation of cilia and flagella

D. Detoxification processes

Answer: A. Structural support

 

40. Question: What is the term for the process by which cells engulf small particles or fluids by forming vesicles?

- A. Phagocytosis

- B. Pinocytosis

- C. Exocytosis

- D. Endocytosis

 

**Answer: B. Pinocytosis**

 

PAP SMEAR MCQS

 

1. Question: What is the primary purpose of a Pap smear?

 

A. Detection of breast cancer

B. Detection of cervical cancer and precancerous changes

C. Diagnosis of ovarian cysts

D. Monitoring thyroid function

Answer: B. Detection of cervical cancer and precancerous changes

 

2. Question: At what age is it recommended for women to start getting regular Pap smears as part of cervical cancer screening?

 

A. 18 years

B. 25 years

C. 30 years

D. 40 years

Answer: B. 25 years

 

3. Question: What is the recommended frequency for Pap smears for women with normal results?

 

A. Every year

B. Every two years

C. Every three years

D. Every five years

Answer: C. Every three years

 

4. Question: What does the Pap test involve?

 

A. Blood test

B. Urine test

C. Collection of cells from the cervix for examination

D. Imaging study of the uterus

Answer: C. Collection of cells from the cervix for examination

 

5. Question: Which virus is a major cause of cervical cancer and is often detected in Pap smear results?

 

A. Influenza virus

B. Human papillomavirus (HPV)

C. Herpes simplex virus (HSV)

D. Cytomegalovirus (CMV)

Answer: B. Human papillomavirus (HPV)

 

6. Question: What is the main goal of early detection through Pap smears in cervical cancer prevention?

 

A. Complete eradication of cancer cells

B. Identification of optimal treatment options

C. Detection and removal of precancerous lesions

D. Determination of cancer staging

Answer: C. Detection and removal of precancerous lesions

 

7. Question: How are abnormal Pap smear results classified?

 

A. Grade I

B. Grade II

C. Low-grade squamous intraepithelial lesion (LSIL) and high-grade squamous intraepithelial lesion (HSIL)

D. Stage 1, Stage 2, and Stage 3

Answer: C. Low-grade squamous intraepithelial lesion (LSIL) and high-grade squamous intraepithelial lesion (HSIL)

 

8. Question: What is the recommended follow-up for women with abnormal Pap smear results indicating high-grade changes?

 

A. Repeat Pap smear in one year

B. Colposcopy and biopsy

C. Immediate hysterectomy

D. Hormonal therapy

Answer: B. Colposcopy and biopsy

 

9. Question: How does the Pap smear contribute to the prevention of cervical cancer?

 

A. By directly treating precancerous lesions

B. By preventing the development of human papillomavirus (HPV)

C. By identifying and treating precancerous changes before they become cancerous

D. By removing the entire cervix

Answer: C. By identifying and treating precancerous changes before they become cancerous

 

10. Question: What is the significance of an "ASCUS" result on a Pap smear?

- A. Presence of normal cervical cells

- B. Indication of cancerous cells

- C. Uncertain or atypical squamous cells of undetermined significance

- D. Detection of hormonal imbalance

**Answer: C. Uncertain or atypical squamous cells of undetermined significance**

 

CERVICAL CANCER MCQ

1. Question: What is the primary cause of cervical cancer?

 

A. Smoking

B. Human papillomavirus (HPV) infection

C. Hormonal imbalance

D. Genetic factors

Answer: B. Human papillomavirus (HPV) infection

 

2. Question: Which type of HPV is most strongly associated with the development of cervical cancer?

 

A. HPV type 6

B. HPV type 11

C. High-risk HPV types, especially 16 and 18

D. Low-risk HPV types, especially 6 and 11

Answer: C. High-risk HPV types, especially 16 and 18

 

3. Question: What is the role of the HPV vaccine in cervical cancer prevention?

 

A. Treatment of existing HPV infections

B. Prevention of all HPV types

C. Prevention of high-risk HPV types, including 16 and 18

D. Cure for cervical cancer

Answer: C. Prevention of high-risk HPV types, including 16 and 18

 

4. Question: Which of the following is a known risk factor for the development of cervical cancer?

 

A. Early menopause

B. Hormone replacement therapy

C. Multiple sexual partners

D. Regular Pap smear screenings

Answer: C. Multiple sexual partners

 

5. Question: What is the purpose of a colposcopy in the evaluation of cervical abnormalities?

 

A. Removal of the cervix

B. Visualization and biopsy of the cervix

C. Hormone therapy

D. Pap smear collection

Answer: B. Visualization and biopsy of the cervix

 

6. Question: How does the FIGO (International Federation of Gynaecology and Obstetrics) staging system classify the extent of cervical cancer?

 

A. Stage 0, Stage I, Stage II, Stage III, Stage IV

B. Grade I, Grade II, Grade III

C. Localized, Regional, Distant

D. Low-risk, Intermediate-risk, High-risk

Answer: A. Stage 0, Stage I, Stage II, Stage III, Stage IV

 

7. Question: What is the most common histological type of cervical cancer?

 

A. Adenocarcinoma

B. Squamous cell carcinoma

C. Small cell carcinoma

D. Neuroendocrine carcinoma

Answer: B. Squamous cell carcinoma

 

8. Question: What is the recommended age for initiating cervical cancer screening with Pap smears in most guidelines?

 

A. 18 years

B. 25 years

C. 30 years

D. 40 years

Answer: B. 25 years

 

9. Question: What is the primary treatment for early-stage cervical cancer?

 

A. Chemotherapy

B. Hysterectomy

C. Radiation therapy

D. Hormone therapy

Answer: B. Hysterectomy

 

10. Question: What is the main purpose of lymph node dissection in cervical cancer surgery?

- A. To remove cancerous cells from lymph nodes

- B. To assess the spread of cancer to nearby lymph nodes

- C. To prevent the recurrence of cancer

- D. To improve cosmetic outcomes

**Answer: B. To assess the spread of cancer to nearby lymph nodes**

 

11. Question: Which of the following is a symptom commonly associated with advanced cervical cancer?

- A. Irregular menstrual cycles

- B. Vaginal bleeding after intercourse

- C. Breast tenderness

- D. Migraine headaches

 

**Answer: B. Vaginal bleeding after intercourse**

12. Question: What is the role of a cone biopsy (conization) in the diagnosis and treatment of cervical abnormalities?

- A. Removal of the entire uterus

- B. Removal of the ovaries

- C. Removal of a cone-shaped piece of the cervix for examination

- D. Chemotherapy administration

**Answer: C. Removal of a cone-shaped piece of the cervix for examination**

13. Question: How does cervical intraepithelial neoplasia (CIN) correlate with the development of cervical cancer?

- A. CIN is a benign condition unrelated to cancer.

- B. CIN represents precancerous changes in the cervical cells.

- C. CIN is an early stage of invasive cervical cancer.

- D. CIN is only associated with adenocarcinoma, not squamous cell carcinoma.

**Answer: B. CIN represents precancerous changes in the cervical cells.**

14. Question: What is the recommended follow-up for women with cervical intraepithelial neoplasia 1 (CIN 1) on biopsy?

- A. Immediate hysterectomy

- B. Repeat Pap smear in one year

- C. Colposcopy and biopsy

- D. Hormone replacement therapy

 

**Answer: B. Repeat Pap smear in one year**

15. Question: Which stage of cervical cancer involves invasion beyond the uterus but not reaching the pelvic wall or the lower third of the vagina?

- A. Stage I

- B. Stage II

- C. Stage III

- D. Stage IV

 

**Answer: B. Stage II**

16. Question: What is the primary purpose of adjuvant therapy in the treatment of cervical cancer?

- A. To improve cosmetic outcomes

- B. To prevent the recurrence of cancer

- C. To remove cancerous cells from lymph nodes

- D. To assess the spread of cancer to nearby lymph nodes

**Answer: B. To prevent the recurrence of cancer**

17. Question: What is the significance of the Papanicolaou (Pap) smear in cervical cancer screening?

- A. It directly detects HPV infection.

- B. It identifies specific genetic mutations in cervical cells.

- C. It assesses the hormonal status of the cervix.

- D. It detects abnormal changes in cervical cells before cancer develops.

 

**Answer: D. It detects abnormal changes in cervical cells before cancer develops.**

18. Question: Which modality is commonly used for the imaging evaluation of cervical cancer spread to distant organs?

- A. Computed tomography (CT) scan

- B. Mammography

- C. Magnetic resonance imaging (MRI)

- D. Positron emission tomography (PET) scan

 

**Answer: D. Positron emission tomography (PET) scan**

19. Question: What is the role of pelvic exenteration in the management of locally advanced cervical cancer?

- A. To remove cancerous cells from lymph nodes

- B. To assess the spread of cancer to nearby lymph nodes

- C. To remove the uterus, cervix, and surrounding structures

- D. To administer chemotherapy directly to the cervix

 

**Answer: C. To remove the uterus, cervix, and surrounding structures**

20. Question: What is the main goal of cervical cancer survivorship care?

- A. To eradicate all cancerous cells

- B. To address the physical and emotional needs of survivors

- C. To administer ongoing aggressive treatment

- D. To minimize the use of screening and follow-up procedures

**Answer: B. To address the physical and emotional needs of survivors**

 

BREAST CANCER TEST MCQ

 

1. Question: What is the primary purpose of a mammogram in breast cancer screening?

 

A. Detection of breast lumps

B. Assessment of breast density

C. Visualization of breast anatomy

D. Early detection of breast cancer

Answer: D. Early detection of breast cancer

 

2. Question: At what age is it generally recommended for women to start regular mammogram screenings for breast cancer?

 

A. 25 years

B. 30 years

C. 40 years

D. 50 years

Answer: C. 40 years

 

3. Question: What is the purpose of breast self-examinations (BSE) in breast cancer awareness?

 

A. To replace mammograms

B. To diagnose breast cancer

C. To identify changes and report them to a healthcare provider

D. To assess breast density

Answer: C. To identify changes and report them to a healthcare provider

 

4. Question: What imaging technique uses sound waves to create images of the breast and is sometimes used in addition to mammography for further evaluation?

 

A. Magnetic Resonance Imaging (MRI)

B. Computed Tomography (CT) scan

C. Positron Emission Tomography (PET) scan

D. Breast ultrasound

Answer: D. Breast ultrasound

 

5. Question: What is the primary goal of genetic testing for breast cancer susceptibility genes, such as BRCA1 and BRCA2?

 

A. To diagnose breast cancer

B. To determine the stage of breast cancer

C. To identify individuals at increased risk for hereditary breast cancer

D. To assess response to treatment

Answer: C. To identify individuals at increased risk for hereditary breast cancer

 

6. Question: In breast cancer staging, what does the "T" represent in the TNM classification system?

 

A. Tumor size

B. Treatment response

C. Total lymph nodes involved

D. Time since diagnosis

Answer: A. Tumor size

 

7. Question: Which molecular marker is commonly tested in breast cancer to determine the presence of hormone receptors and guide hormonal therapy decisions?

 

A. HER2 (Human Epidermal Growth Factor Receptor 2)

B. EGFR (Epidermal Growth Factor Receptor)

C. ER (Estrogen Receptor)

D. PD-L1 (Programmed Death-Ligand 1)

Answer: C. ER (Estrogen Receptor)

 

8. Question: What is the primary purpose of sentinel lymph node biopsy in breast cancer surgery?

 

A. To assess tumor size

B. To determine the stage of breast cancer

C. To remove cancerous lymph nodes

D. To identify the first lymph nodes draining the tumor

Answer: D. To identify the first lymph nodes draining the tumor

 

9. Question: What is the most common type of breast cancer, accounting for approximately 70-80% of cases?

 

A. Inflammatory breast cancer

B. Ductal carcinoma in situ (DCIS)

C. Invasive ductal carcinoma (IDC)

D. Lobular carcinoma in situ (LCIS)

Answer: C. Invasive ductal carcinoma (IDC)

 

10. Question: What is the purpose of neoadjuvant chemotherapy in the treatment of breast cancer?

- A. To prevent breast cancer recurrence

- B. To shrink tumors before surgery

- C. To replace surgery in some cases

- D. To enhance radiation therapy effectiveness

 

**Answer: B. To shrink tumors before surgery**

Ø HISTOPATHOLOGY 100 MCQS

 

1. Question: What is the primary purpose of histopathology?

 

A. Diagnosis of infectious diseases

B. Examination of tissues for structural abnormalities and diseases

C. Study of cell cultures

D. Assessment of blood chemistry

Answer: B. Examination of tissues for structural abnormalities and diseases

 

2. Question: Which staining technique is commonly used in histopathology to differentiate between cell nuclei and the surrounding cytoplasm?

 

A. Hematoxylin and eosin (H&E) stain

B. Giemsa stain

C. Periodic acid-Schiff (PAS) stain

D. Masson's trichrome stain

Answer: A. Hematoxylin and eosin (H&E) stain

 

3. Question: What does immunohistochemistry (IHC) involve in histopathology?

 

A. Staining for cellular structures

B. Staining for specific proteins using antibodies

C. Staining for carbohydrates

D. Staining for lipids

Answer: B. Staining for specific proteins using antibodies

 

4. Question: In histopathology, what does the term "biopsy" refer to?

 

A. Removal of an entire organ for examination

B. Collection of tissue samples for microscopic examination

C. Visualization of internal structures using imaging techniques

D. Assessment of blood parameters

Answer: B. Collection of tissue samples for microscopic examination

 

5. Question: What is the purpose of frozen section analysis in histopathology?

 

A. To stain tissues for permanent slides

B. To quickly assess tissue during surgery for immediate diagnosis

C. To preserve tissues for long-term storage

D. To analyze tissues under ultraviolet light

Answer: B. To quickly assess tissue during surgery for immediate diagnosis

 

6. Question: What is the role of a pathologist in histopathology?

 

A. Surgical removal of tumors

B. Analysis and interpretation of tissue specimens

C. Administration of chemotherapy

D. Conducting radiological imaging studies

Answer: B. Analysis and interpretation of tissue specimens

 

7. Question: Which staining method is commonly used to highlight collagen fibers in histopathological slides?

 

A. Hematoxylin and eosin (H&E) stain

B. Masson's trichrome stain

C. Giemsa stain

D. Wright's stain

Answer: B. Masson's trichrome stain

 

8. Question: What is the term for the study of cells and tissues under the microscope to diagnose diseases?

 

A. Cytology

B. Histology

C. Microbiology

D. Radiology

Answer: B. Histology

 

9. Question: What does the term "histopathological grading" refer to?

 

A. Determining the age of tissues

B. Assessing the spread of cancer

C. Evaluating the degree of differentiation of tumor cells

D. Identifying infectious agents in tissues

Answer: C. Evaluating the degree of differentiation of tumor cells

 

10. Question: What is the purpose of using special stains in histopathology?

- A. To enhance tissue preservation

- B. To differentiate between benign and malignant tumors

- C. To remove artifacts from tissue sections

- D. To improve the visibility of normal cells

 

 

**Answer: B. To differentiate between benign and malignant tumors**

11. Question: What is the term for the microscopic examination of cells shed or scraped from a tissue surface for diagnostic purposes?

- A. Histology

- B. Cytology

- C. Immunohistochemistry

- D. Biopsy

 

**Answer: B. Cytology**

12. Question: Which of the following stains is commonly used for highlighting acidic components like mucin in histopathology?

- A. Periodic acid-Schiff (PAS) stain

- B. Hematoxylin and eosin (H&E) stain

- C. Wright's stain

- D. Giemsa stain

 

**Answer: A. Periodic acid-Schiff (PAS) stain**

13. Question: What is the purpose of using immunofluorescence in histopathology?

- A. To visualize structures under ultraviolet light

- B. To stain cell nuclei

- C. To enhance tissue preservation

- D. To differentiate between benign and malignant tumors

 

**Answer: A. To visualize structures under ultraviolet light**

14. Question: In histopathology, what does the term "in situ" indicate when describing a lesion or tumor?

- A. Spread to nearby tissues

- B. Limited to the original site without invasion

- C. Presence of necrosis

- D. Metastasis to distant organs

 

**Answer: B. Limited to the original site without invasion**

15. Question: What is the purpose of a fine-needle aspiration (FNA) in histopathology?

- A. Collection of tissue samples for frozen section analysis

- B. Removal of entire tumors

- C. Removal of lymph nodes for examination

- D. Aspiration of cells for cytological evaluation

 

**Answer: D. Aspiration of cells for cytological evaluation**

16. Question: What does the term "tumor margin" refer to in histopathology?

- A. The size of the tumor

- B. The outer boundary of the excised tumor in relation to normal tissue

- C. The grade of tumor differentiation

- D. The degree of tumor invasion

 

**Answer: B. The outer boundary of the excised tumor in relation to normal tissue**

17. Question: Which type of tissue processing technique involves embedding tissues in paraffin wax for sectioning and staining?

- A. Cryosectioning

- B. Freeze-fracture

- C. Paraffin embedding

- D. Plastic embedding

 

**Answer: C. Paraffin embedding**

18. Question: What is the primary advantage of immunohistochemistry (IHC) in comparison to routine histological staining?

- A. Improved tissue preservation

- B. Rapid staining process

- C. Specific identification of proteins or antigens

- D. Visualization of cellular structures

 

**Answer: C. Specific identification of proteins or antigens**

19. Question: What is the purpose of a Hirschsprung disease biopsy in histopathology?

- A. To assess lung function

- B. To evaluate gastrointestinal motility

- C. To examine liver function

- D. To diagnose neurointestinal disorders

 

 

**Answer: B. To evaluate gastrointestinal motility**

20. Question: In histopathology, what does the term "hyperplasia" refer to?

- A. Abnormal cell growth forming a mass

- B. Increase in the size of cells

- C. Increase in the number of cells in a tissue

- D. Development of tumors

 

**Answer: C. Increase in the number of cells in a tissue**

 

 

21. Question: What is the primary purpose of a special stain like Masson's trichrome in histopathology?

- A. Staining cell nuclei

- B. Highlighting collagen fibers

- C. Identifying lipid droplets

- D. Differentiating between benign and malignant cells

 

**Answer: B. Highlighting collagen fibers**

22. Question: Which staining method is commonly used to visualize microorganisms such as bacteria and fungi in tissue sections?

- A. Hematoxylin and eosin (H&E) stain

- B. Periodic acid-Schiff (PAS) stain

- C. Gram stain

- D. Giemsa stain

 

 

**Answer: C. Gram stain**

23. Question: In histopathology, what does the term "metaplasia" refer to?

- A. Transformation of cells into cancerous cells

- B. Change in the type of differentiated cells

- C. Increase in the number of cells

- D. Replacement of one mature cell type with another

 

 

**Answer: D. Replacement of one mature cell type with another**

24. Question: What is the purpose of using the TUNEL assay in histopathology?

- A. Detection of apoptosis in tissue sections

- B. Visualization of cell nuclei

- C. Staining for connective tissue fibers

- D. Identification of infectious agents

 

 

**Answer: A. Detection of apoptosis in tissue sections**

25. Question: Which of the following is a common application of fluorescence in situ hybridization (FISH) in histopathology?

- A. Staining cell nuclei

- B. Detection of specific DNA sequences or gene abnormalities

- C. Highlighting collagen fibers

- D. Identifying cellular organelles

 

**Answer: B. Detection of specific DNA sequences or gene abnormalities**

26. Question: What is the primary disadvantage of frozen section analysis in histopathology compared to formalin-fixed, paraffin-embedded tissue sections?

- A. Inability to stain for proteins

- B. Limited tissue preservation

- C. Longer processing time

- D. Lack of cellular detail

 

**Answer: B. Limited tissue preservation**

27. Question: Which of the following stains is commonly used to visualize amyloid deposits in tissue sections?

- A. Hematoxylin and eosin (H&E) stain

- B. Congo red stain

- C. Wright's stain

- D. Giemsa stain

 

**Answer: B. Congo red stain**

28. Question: What is the role of electron microscopy in histopathology?

- A. Visualization of cellular structures at the molecular level

- B. Staining for specific proteins

- C. Rapid assessment of tissue during surgery

- D. Detection of cell nuclei

 

**Answer: A. Visualization of cellular structures at the molecular level**

29. Question: What does the term "granuloma" refer to in histopathology?

- A. A benign tumor

- B. A type of fungal infection

- C. A collection of immune cells

- D. Abnormal cell growth forming a mass

 

**Answer: C. A collection of immune cells**

30. Question: What is the primary purpose of a histochemical stain like PAS (Periodic acid-Schiff) in histopathology?

- A. Visualization of connective tissue fibers

- B. Detection of infectious agents

- C. Staining for mucin and glycogen

- D. Identification of specific DNA sequences

 

**Answer: C. Staining for mucin and glycogen**

 

 

31. Question: What is the term for a tissue or organ that appears normal under the microscope but is functionally impaired?

- A. Hyperplasia

- B. Dysplasia

- C. Neoplasia

- D. Pseudoneoplasia

 

**Answer: D. Pseudoneoplasia**

32. Question: Which of the following stains is commonly used to highlight reticular fibers in histopathology?

- A. Masson's trichrome stain

- B. Wright's stain

- C. Congo red stain

- D. Giemsa stain

 

**Answer: A. Masson's trichrome stain**

33. Question: In histopathology, what does the term "dysplasia" indicate?

- A. Increase in cell number

- B. Replacement of one cell type with another

- C. Change in the size and shape of cells

- D. Inflammation in tissues

 

**Answer: C. Change in the size and shape of cells**

34. Question: What is the primary purpose of the Ziehl-Neelsen stain in histopathology?

- A. Detection of fungal infections

- B. Identification of viral particles

- C. Visualization of acid-fast bacteria, like Mycobacterium tuberculosis

- D. Staining cell nuclei

 

**Answer: C. Visualization of acid-fast bacteria, like Mycobacterium tuberculosis**

35. Question: What does the term "cryosectioning" involve in histopathology?

- A. Preservation of tissues in liquid nitrogen

- B. Embedding tissues in paraffin wax

- C. Rapid freezing and sectioning of tissues for examination

- D. Staining for specific proteins using antibodies

 

**Answer: C. Rapid freezing and sectioning of tissues for examination**

36. Question: What is the primary purpose of a H&E stain in histopathology?

- A. Visualization of cell nuclei

- B. Highlighting collagen fibers

- C. Staining for specific proteins

- D. Differentiating between benign and malignant cells

 

**Answer: A. Visualization of cell nuclei**

37. Question: What is the significance of the Warthin-Starry stain in histopathology?

- A. Detection of Helicobacter pylori in gastric biopsies

- B. Visualization of fungi in tissue sections

- C. Identification of viral particles

- D. Staining for amyloid deposits

 

**Answer: A. Detection of Helicobacter pylori in gastric biopsies**

38. Question: In histopathology, what does the term "angiogenesis" refer to?

- A. Formation of new blood vessels

- B. Inflammation in blood vessels

- C. Degeneration of blood vessels

- D. Formation of blood clots

 

**Answer: A. Formation of new blood vessels**

39. Question: What is the primary purpose of a toluidine blue stain in histopathology?

- A. Detection of fungal infections

- B. Identification of viral particles

- C. Visualization of mast cells and basophils

- D. Staining for mucin

 

 

**Answer: C. Visualization of mast cells and basophils**

40. Question: Which of the following is a common fixative used in histopathology to preserve tissues for examination?

- A. Acetone

- B. Methanol

- C. Formaldehyde

- D. Xylene

 

 

**Answer: C. Formaldehyde**

41. Question: What is the primary purpose of a Gomori methenamine silver (GMS) stain in histopathology?

 

A. Visualization of connective tissue fibers

B. Detection of Helicobacter pylori in gastric biopsies

C. Identification of fungal elements

D. Staining for mucin

Answer: C. Identification of fungal elements

 

42. Question: In histopathology, what does the term "langerhans cells" refer to?

 

A. Immune cells in the skin and mucous membranes

B. Cells found in the liver

C. Red blood cells

D. Cells in the nervous system

Answer: A. Immune cells in the skin and mucous membranes

 

43. Question: What is the significance of the Jones silver stain in renal histopathology?

 

A. Identification of amyloid deposits

B. Visualization of glomerular basement membrane

C. Detection of viral particles

D. Staining for specific proteins

Answer: B. Visualization of glomerular basement membrane

 

44. Question: What does the term "mucinous carcinoma" refer to in histopathology?

 

A. A type of melanoma

B. A subtype of breast cancer characterized by mucin production

C. Cancer of the liver

D. Cancer of the colon

Answer: B. A subtype of breast cancer characterized by mucin production

 

45. Question: What is the purpose of a periodic acid-methenamine-silver (PAMS) stain in histopathology?

 

A. Identification of fungal elements

B. Visualization of collagen fibers

C. Detection of viral particles

D. Staining for mucin

Answer: A. Identification of fungal elements

 

46. Question: Which staining technique is commonly used to visualize reticulin fibers in histopathological sections?

 

A. Masson's trichrome stain

B. Wright's stain

C. Silver impregnation stain

D. Giemsa stain

Answer: C. Silver impregnation stain

 

47. Question: What is the primary role of a pathologist in the analysis of histopathological specimens?

 

A. Surgical removal of tumors

B. Collection of tissue samples

C. Analysis and interpretation of tissue specimens

D. Administration of chemotherapy

Answer: C. Analysis and interpretation of tissue specimens

 

48. Question: Which of the following stains is commonly used for highlighting eosinophils in tissue sections?

 

A. Hematoxylin and eosin (H&E) stain

B. Wright's stain

C. Giemsa stain

D. Toluidine blue stain

Answer: B. Wright's stain

 

49. Question: What is the significance of the Mallory trichrome stain in histopathology?

 

A. Visualization of mucin

B. Detection of viral particles

C. Highlighting collagen fibers

D. Staining for specific proteins

Answer: C. Highlighting collagen fibers

 

50. Question: In histopathology, what does the term "tumor budding" refer to?

- A. Rapid growth of tumors

- B. Clusters of tumor cells at the tumor margin

- C. Formation of new blood vessels within tumors

- D. Metastatic spread to distant organs

 

 

**Answer: B. Clusters of tumor cells at the tumor margin**

 

51. Question: What is the primary purpose of a Wright-Giemsa stain in histopathology?

 

A. Identification of viral particles

B. Visualization of connective tissue fibers

C. Detection of fungal elements

D. Staining for blood cells

Answer: D. Staining for blood cells

 

52. Question: In histopathology, what does the term "hyalinization" refer to?

 

A. Formation of new blood vessels

B. Degeneration and glassy appearance of tissues

C. Increase in the size of cells

D. Replacement of one cell type with another

Answer: B. Degeneration and glassy appearance of tissues

 

53. Question: What is the primary purpose of a Verhoeff-van Gieson stain in histopathology?

 

A. Identification of collagen fibers

B. Visualization of elastic fibers

C. Detection of viral particles

D. Staining for specific proteins

Answer: B. Visualization of elastic fibers

 

54. Question: In histopathology, what does the term "myocarditis" refer to?

 

A. Inflammation of the liver

B. Inflammation of the lung

C. Inflammation of the heart muscle

D. Inflammation of the kidney

Answer: C. Inflammation of the heart muscle

 

55. Question: What is the primary purpose of a Papanicolaou (Pap) stain in histopathology?

 

A. Detection of fungi

B. Visualization of cell nuclei in cytological smears

C. Identification of collagen fibers

D. Staining for specific proteins

Answer: B. Visualization of cell nuclei in cytological smears

 

56. Question: In histopathology, what does the term "xanthoma" refer to?

 

A. A type of skin cancer

B. Accumulation of lipid-laden macrophages

C. Benign tumor of the breast

D. Cancer of the liver

Answer: B. Accumulation of lipid-laden macrophages

 

57. Question: What is the primary purpose of a Ziehl-Neelsen stain in histopathology?

 

A. Detection of Helicobacter pylori

B. Visualization of acid-fast bacteria

C. Identification of collagen fibers

D. Staining for specific proteins

Answer: B. Visualization of acid-fast bacteria

 

58. Question: In histopathology, what does the term "osteoid" refer to?

 

A. Cartilaginous tissue

B. Hard,ony tissue

C. Connective tissue fibers

D. Fatty tissue

Answer: B. Hard,ony tissue

 

59. Question: What is the purpose of a Fontana-Masson stain in histopathology?

 

A. Visualization of nerve fibers

B. Identification of melanin

C. Detection of viral particles

D. Staining for specific proteins

Answer: B. Identification of melanin

 

60. Question: In histopathology, what does the term "spongiosis" refer to?

 

A. Accumulation of fluid in tissues

B. Sponge-like appearance in tissues due to edema

C. Neurological disorder

D. Increase in cell number

Answer: B. Sponge-like appearance in tissues due to edema

 

61. Question: What is the primary purpose of a Periodic acid-Schiff (PAS) stain in histopathology?

 

A. Visualization of elastic fibers

B. Detection of fungal elements and basement membrane material

C. Identification of collagen fibers

D. Staining for blood cells

Answer: B. Detection of fungal elements and basement membrane material

 

62. Question: In histopathology, what does the term "granulation tissue" refer to?

 

A. A type of benign tumor

B. Newly formed tissue during wound healing

C. A subtype of lung cancer

D. Accumulation of immune cells

Answer: B. Newly formed tissue during wound healing

 

63. Question: What is the primary purpose of a silver stain, such as the Steiner stain, in histopathology?

 

A. Visualization of elastic fibers

B. Detection of Helicobacter pylori

C. Identification of collagen fibers

D. Staining for specific proteins

Answer: B. Detection of Helicobacter pylori

 

64. Question: In histopathology, what does the term "hemosiderin" refer to?

 

A. Red blood cells

B. Iron-containing pigment

C. Collagen fibers

D. Connective tissue

Answer: B. Iron-containing pigment

 

65. Question: What is the primary purpose of a Giesma stain in histopathology?

 

A. Identification of fungi

B. Visualization of nerve fibers

C. Detection of viral particles

D. Staining for blood cells

Answer: A. Identification of fungi

 

66. Question: In histopathology, what does the term "cryptococcosis" refer to?

 

A. Fungal infection

B. Bacterial infection

C. Viral infection

D. Parasitic infection

Answer: A. Fungal infection

 

67. Question: What is the primary purpose of a Luxol fast blue stain in histopathology?

 

A. Identification of lipids

B. Visualization of nerve fibers

C. Detection of viral particles

D. Staining for specific proteins

Answer: B. Visualization of nerve fibers

 

68. Question: In histopathology, what does the term "syncytium" refer to?

 

A. Collection of immune cells

B. Fused cells with shared cytoplasm

C. Tumor cells

D. Necrotic tissue

Answer: B. Fused cells with shared cytoplasm

 

69. Question: What is the significance of the Wright-Giemsa stain in hematopathology?

 

A. Detection of viral particles

B. Visualization of blood cells in peripheral smears

C. Identification of fungi

D. Staining for specific proteins

Answer: B. Visualization of blood cells in peripheral smears

 

70. Question: In histopathology, what does the term "xanthelasma" refer to?

 

A. Accumulation of lipid-laden macrophages in the skin

B. A type of skin cancer

C. Benign tumor of the liver

D. Cancer of the colon

Answer: A. Accumulation of lipid-laden macrophages in the skin

 

71. Question: What is the primary purpose of a Fontana-Masson silver stain in histopathology?

 

A. Identification of collagen fibers

B. Visualization of nerve fibers

C. Detection of melanin in tissues

D. Staining for blood cells

Answer: C. Detection of melanin in tissues

 

72. Question: In histopathology, what does the term "carcinoma in situ" (CIS) refer to?

 

A. Invasive cancer

B. Cancer confined to the epithelium without invasion

C. Benign tumor

D. Metastatic cancer

Answer: B. Cancer confined to the epithelium without invasion

 

73. Question: What is the primary purpose of a Ziehl-Neelsen stain in histopathology?

 

A. Detection of Helicobacter pylori

B. Visualization of acid-fast bacteria, like Mycobacterium tuberculosis

C. Identification of fungi

D. Staining for specific proteins

Answer: B. Visualization of acid-fast bacteria, like Mycobacterium tuberculosis

 

74. Question: In histopathology, what does the term "acrochordon" refer to?

 

A. A type of skin cancer

B. Benign tumor of the breast

C. Skin tag or soft fibroma

D. Cancer of the liver

Answer: C. Skin tag or soft fibroma

 

75. Question: What is the primary purpose of a Wright-Giemsa stain in histopathology?

 

A. Detection of viral particles

B. Visualization of blood cells, including red and white blood cells

C. Identification of fungi

D. Staining for specific proteins

Answer: B. Visualization of blood cells, including red and white blood cells

 

76. Question: In histopathology, what does the term "Langerhans cell histiocytosis" refer to?

 

A. Inflammatory skin disorder

B. Type of leukemia

C. Accumulation of abnormal Langerhans cells

D. Benign tumor of the lung

Answer: C. Accumulation of abnormal Langerhans cells

 

77. Question: What is the primary purpose of a Warthin-Starry silver stain in histopathology?

 

A. Visualization of nerve fibers

B. Identification of viral particles

C. Detection of Helicobacter pylori in gastric biopsies

D. Staining for specific proteins

Answer: C. Detection of Helicobacter pylori in gastric biopsies

 

78. Question: In histopathology, what does the term "sarcoidosis" refer to?

 

A. Inflammatory disorder affecting multiple organs

B. Autoimmune disorder of the skin

C. Type of lymphoma

D. Accumulation of lipids in tissues

Answer: A. Inflammatory disorder affecting multiple organs

 

79. Question: What is the significance of a Congo red stain in histopathology?

 

A. Visualization of collagen fibers

B. Identification of fungal elements

C. Detection of amyloid deposits

D. Staining for specific proteins

Answer: C. Detection of amyloid deposits

 

80. Question: In histopathology, what does the term "squamous metaplasia" refer to?

 

A. Transformation of cells into cancerous cells

B. Replacement of one cell type with squamous epithelial cells

C. Increase in the size of cells

D. Benign tumor of squamous cells

Answer: B. Replacement of one cell type with squamous epithelial cells

 

81. Question: What is the primary purpose of a Gomori trichrome stain in histopathology?

 

A. Visualization of elastic fibers

B. Detection of fungal elements

C. Identification of collagen fibers

D. Staining for blood cells

Answer: C. Identification of collagen fibers

 

82. Question: In histopathology, what does the term "leiomyoma" refer to?

 

A. Benign tumor of smooth muscle

B. Cancer of the liver

C. Accumulation of lipid-laden macrophages

D. A type of skin cancer

Answer: A. Benign tumor of smooth muscle

 

83. Question: What is the primary purpose of a methenamine silver stain in histopathology?

 

A. Visualization of nerve fibers

B. Detection of fungal elements

C. Identification of collagen fibers

D. Staining for specific proteins

Answer: B. Detection of fungal elements

 

84. Question: In histopathology, what does the term "anaplasia" refer to?

 

A. A type of skin cancer

B. Loss of differentiation in cells, associated with malignancy

C. Accumulation of immune cells

D. Increase in the size of cells

Answer: B. Loss of differentiation in cells, associated with malignancy

 

85. Question: What is the primary purpose of a methyl green-pyronin stain in histopathology?

 

A. Identification of viral particles

B. Visualization of collagen fibers

C. Detection of melanin in tissues

D. Staining for nucleic acids in cells

Answer: D. Staining for nucleic acids in cells

 

86. Question: In histopathology, what does the term "aschoff bodies" refer to?

 

A. Accumulation of lipid-laden macrophages

B. Structures found in the liver

C. Characteristic of rheumatic fever in the heart tissue

D. Tumor cells with granular appearance

Answer: C. Characteristic of rheumatic fever in the heart tissue

 

87. Question: What is the primary purpose of a mucicarmine stain in histopathology?

 

A. Visualization of nerve fibers

B. Identification of fungal elements

C. Detection of melanin in tissues

D. Staining for mucin in cells

Answer: D. Staining for mucin in cells

 

88. Question: In histopathology, what does the term "Kaposi's sarcoma" refer to?

 

A. Skin infection

B. Type of bone tumor

C. Malignancy associated with human herpesvirus 8 (HHV-8)

D. Inflammation of the kidneys

Answer: C. Malignancy associated with human herpesvirus 8 (HHV-8)

 

89. Question: What is the primary purpose of a cresyl violet stain in histopathology?

 

A. Visualization of nerve fibers

B. Detection of viral particles

C. Identification of collagen fibers

D. Staining for specific proteins

Answer: A. Visualization of nerve fibers

 

90. Question: In histopathology, what does the term "fibroadenoma" refer to?

 

A. Benign tumor of the breast

B. Cancer of the lung

C. Accumulation of lipid-laden macrophages

D. Malignancy of connective tissue

Answer: A. Benign tumor of the breast

 

91. Question: What is the primary purpose of a reticulin stain in histopathology?

 

A. Visualization of collagen fibers

B. Detection of fungal elements

C. Identification of reticular fibers in tissues

D. Staining for blood cells

Answer: C. Identification of reticular fibers in tissues

 

92. Question: In histopathology, what does the term "giant cell" refer to?

 

A. Large cells with abundant cytoplasm

B. Fused cells with shared cytoplasm

C. Malignant cells

D. Cells with granular appearance

Answer: A. Large cells with abundant cytoplasm

 

93. Question: What is the primary purpose of a Wright's stain in histopathology?

 

A. Detection of viral particles

B. Visualization of blood cells, including red and white blood cells

C. Identification of fungi

D. Staining for specific proteins

Answer: B. Visualization of blood cells, including red and white blood cells

 

94. Question: In histopathology, what does the term "angiosarcoma" refer to?

 

A. Malignancy of blood vessels

B. Benign tumor of the skin

C. Accumulation of lipid-laden macrophages

D. Cancer of the liver

Answer: A. Malignancy of blood vessels

 

95. Question: What is the primary purpose of a PAS-diastase stain in histopathology?

 

A. Visualization of elastic fibers

B. Detection of viral particles

C. Identification of glycogen in tissues

D. Staining for blood cells

Answer: C. Identification of glycogen in tissues

 

96. Question: In histopathology, what does the term "osteosarcoma" refer to?

 

A. Malignancy of the bone

B. Benign tumor of the breast

C. Accumulation of lipid-laden macrophages in bone tissue

D. Cancer of the colon

Answer: A. Malignancy of the bone

 

97. Question: What is the primary purpose of a toluidine blue stain in histopathology?

 

A. Detection of viral particles

B. Visualization of nerve fibers and mast cells

C. Identification of fungal elements

D. Staining for blood cells

Answer: B. Visualization of nerve fibers and mast cells

 

98. Question: In histopathology, what does the term "basal cell carcinoma" refer to?

 

A. Malignancy of basal cells in the epidermis

B. Benign tumor of the lung

C. Accumulation of lipid-laden macrophages in the skin

D. Cancer of the liver

Answer: A. Malignancy of basal cells in the epidermis

 

99. Question: What is the primary purpose of a Van Gieson stain in histopathology?

 

A. Visualization of nerve fibers

B. Identification of fungal elements

C. Detection of viral particles

D. Staining for collagen fibers

Answer: D. Staining for collagen fibers

 

100. Question: In histopathology, what does the term "nephritis" refer to?

- A. Inflammation of the skin

- B. Inflammation of the lung

- C. Inflammation of the kidney

- D. Inflammation of the liver

 

**Answer: C. Inflammation of the kidney**

Ø ANATOMY & PHYSIOLOGY 100 MCQS

 

1. What is the primary function of the respiratory system?

 

A. Nutrient absorption

B. Gas exchange

C. Blood filtration

D. Hormone production

Answer: B. Gas exchange

 

2. Which type of muscle is voluntary and under conscious control?

 

A. Smooth muscle

B. Cardiac muscle

C. Skeletal muscle

D. Involuntary muscle

Answer: C. Skeletal muscle

 

3. The heart is part of which system in the human body?

 

A. Cardiovascular system

B. Nervous system

C. Respiratory system

D. Endocrine system

Answer: A. Cardiovascular system

 

4. What is the main function of the kidneys in the urinary system?

 

A. Blood circulation

B. Nutrient absorption

C. Filtration and excretion of waste products

D. Hormone production

Answer: C. Filtration and excretion of waste products

 

5. Which hormone is responsible for regulating blood sugar levels?

 

A. Insulin

B. Thyroxine

C. Cortisol

D. Oestrogen

Answer: A. Insulin

 

6. What is the role of the digestive system in the body?

 

A. Gas exchange

B. Nutrient absorption

C. Blood filtration

D. Muscle contraction

Answer: B. Nutrient absorption

 

7. Which part of the brain is responsible for coordinating muscle movements and maintaining balance?

 

A. Cerebrum

B. Cerebellum

C. Medulla oblongata

D. Thalamus

Answer: B. Cerebellum

 

8. What is the function of the endocrine system?

 

A. Regulation of body temperature

B. Production of antibodies

C. Secretion of hormones to regulate body functions

D. Transport of oxygen in the blood

Answer: C. Secretion of hormones to regulate body functions

 

9. Where does the process of digestion primarily occur in the human body?

 

A. Stomach

B. Small intestine

C. Esophagus

D. Mouth

Answer: B. Small intestine

 

10. Which of the following is a function of the integumentary system?

 

A. Blood circulation

B. Gas exchange

C. Protection against pathogens and dehydration

D. Hormone production

Answer: C. Protection against pathogens and dehydration

 

11. What is the primary function of the respiratory diaphragm?

 

A. Pump blood to the lungs

B. Facilitate breathing by contracting and relaxing

C. Aid in digestion

D. Produce hormones for metabolism

Answer: B. Facilitate breathing by contracting and relaxing

 

12. Which hormone is responsible for regulating the body's sleep-wake cycle?

 

A. Insulin

B. Melatonin

C. Adrenaline

D. Thyroxine

Answer: B. Melatonin

 

13. What is the purpose of the lymphatic system in the body?

 

A. Oxygen transport

B. Immune defence and fluid balance

C. Digestion of fats

D. Muscle coordination

Answer: B. Immune defence and fluid balance

 

14. The central nervous system consists of:

 

A. Brain and spinal cord

B. Nerves and ganglia

C. Heart and lungs

D. Kidneys and liver

Answer: A. Brain and spinal cord

 

15. Which organ is responsible for detoxifying harmful substances in the blood?

 

A. Kidneys

B. Liver

C. Lungs

D. Pancreas

Answer: B. Liver

 

16. What is the function of red blood cells (erythrocytes) in the circulatory system?

 

A. Carry oxygen to tissues

B. Fight infections

C. Clot blood

D. Produce antibodies

Answer: A. Carry oxygen to tissues

 

17. Which part of the brain is responsible for regulating body temperature and hunger?

 

A. Hypothalamus

B. Medulla oblongata

C. Cerebrum

D. Thalamus

Answer: A. Hypothalamus

 

18. What is the primary function of the large intestine?

 

A. Nutrient absorption

B. Water absorption and formation of feces

C. Protein digestion

D. Gas exchange

Answer: B. Water absorption and formation of feces

 

19. Which hormone is responsible for the fight or flight response in stressful situations?

 

A. Insulin

B. Cortisol

C. Adrenaline (epinephrine)

D. Thyroxine

Answer: C. Adrenaline (epinephrine)

 

20. Where are sweat glands primarily found in the skin?

 

A. Dermis

B. Epidermis

C. Subcutaneous tissue

D. Hair follicles

Answer: A. Dermis

 

21. What is the primary function of the spleen in the human body?

 

A. Blood clotting

B. Filtration of blood and storage of platelets

C. Digestion of fats

D. Oxygen transport

Answer: B. Filtration of blood and storage of platelets

 

22. Which hormone is responsible for regulating calcium levels in the blood?

 

A. Insulin

B. Parathyroid hormone (PTH)

C. Growth hormone

D. Estrogen

Answer: B. Parathyroid hormone (PTH)

 

23. What is the purpose of the Eustachian tube in the ear?

 

A. Equalize air pressure between the middle ear and the atmosphere

B. Transmit sound waves to the brain

C. Produce earwax for protection

D. Regulate balance and spatial orientation

Answer: A. Equalize air pressure between the middle ear and the atmosphere

 

24. The trachea is commonly known as the:

 

A. Windpipe

B. Food pipe

C. Esophagus

D. Bronchus

Answer: A. Windpipe

 

25. What is the primary function of the adrenal glands?

 

A. Regulation of blood sugar levels

B. Production of adrenaline and cortisol

C. Synthesis of insulin

D. Blood filtration

Answer: B. Production of adrenaline and cortisol

 

26. Which of the following is NOT a function of the skin?

 

A. Protection against pathogens

B. Synthesis of vitamin D

C. Production of insulin

D. Regulation of body temperature

Answer: C. Production of insulin

 

27. Which blood vessels carry oxygenated blood away from the heart?

 

A. Arteries

B. Veins

C. Capillaries

D. Venules

Answer: A. Arteries

 

28. The pituitary gland is often referred to as the:

 

A. Master gland

B. Digestive gland

C. Adrenal gland

D. Thyroid gland

Answer: A. Master gland

 

29. Where does fertilization of an egg by sperm typically occur in the female reproductive system?

 

A. Ovary

B. Uterus

C. Fallopian tube

D. Cervix

Answer: C. Fallopian tube

 

30. Which of the following bones is part of the axial skeleton?

 

A. Femur

B. Radius

C. Skull

D. Humerus

Answer: C. Skull

 

31. What is the function of the cerebrospinal fluid in the central nervous system?

 

A. Provide nutrients to the brain

B. Act as a shock absorber and protect the brain

C. Transmit electrical impulses

D. Regulate body temperature

Answer: B. Act as a shock absorber and protect the brain

 

32. The structure that separates the thoracic cavity from the abdominal cavity is called the:

 

A. Diaphragm

B. Pleura

C. Peritoneum

D. Mesentery

Answer: A. Diaphragm

 

33. What is the primary function of the pancreas in the digestive system?

 

A. Production of bile

B. Regulation of blood sugar levels

C. Digestion of proteins

D. Absorption of nutrients

Answer: B. Regulation of blood sugar levels

 

34. Which of the following statements about red bone marrow is correct?

 

A. Red bone marrow is primarily found in compact bones

B. Red bone marrow produces red and white blood cells

C. Red bone marrow is more abundant in adults than in infants

D. Red bone marrow stores minerals for bone strength

Answer: B. Red bone marrow produces red and white blood cells

 

35. What is the purpose of the vestibular system in the inner ear?

 

A. Hearing

B. Balance and spatial orientation

C. Temperature regulation

D. Synthesis of earwax

Answer: B. Balance and spatial orientation

 

36. Which of the following is a function of the lymph nodes in the lymphatic system?

 

A. Filtration of blood

B. Production of antibodies

C. Storage of platelets

D. Regulation of blood pressure

Answer: B. Production of antibodies

 

37. The process by which an egg matures and is released from the ovary is called:

 

A. Ovulation

B. Menstruation

C. Fertilization

D. Implantation

Answer: A. Ovulation

 

38. What is the primary function of the cornea in the eye?

 

A. Control the amount of light entering the eye

B. Refract light to focus it on the retina

C. Produce tears for lubrication

D. Change the shape of the lens for accommodation

Answer: B. Refract light to focus it on the retina

 

39. Which of the following bones is part of the appendicular skeleton?

 

A. Sternum

B. Vertebra

C. Humerus

D. Skull

Answer: C. Humerus

 

40. What is the function of the alveoli in the respiratory system?

 

A. Transport oxygen in the blood

B. Filter air entering the lungs

C. Exchange gases (oxygen and carbon dioxide) with the bloodstream

D. Produce mucus for lung protection

Answer: C. Exchange gases (oxygen and carbon dioxide)

 

 

41. What is the function of the thyroid gland in the endocrine system?

 

A. Regulation of blood sugar levels

B. Control of metabolic rate and energy production

C. Production of insulin

D. Maintenance of water balance

Answer: B. Control of metabolic rate and energy production

 

42. The joint between two bones that allows movement in multiple directions is called a:

 

A. Hinge joint

B. Ball and socket joint

C. Pivot joint

D. Gliding joint

Answer: B. Ball and socket joint

 

43. Which part of the brain is responsible for regulating basic life functions such as heartbeat and breathing?

 

A. Cerebrum

B. Cerebellum

C. Medulla oblongata

D. Thalamus

Answer: C. Medulla oblongata

 

44. The epiglottis prevents food from entering which structure during swallowing?

 

A. Trachea (windpipe)

B. Oesophagus

C. Stomach

D. Lungs

Answer: A. Trachea (windpipe)

 

45. What is the function of the gallbladder in the digestive system?

 

A. Production of bile

B. Storage and concentration of bile

C. Digestion of carbohydrates

D. Absorption of nutrients

Answer: B. Storage and concentration of bile

 

46. Which blood vessels carry deoxygenated blood back to the heart?

 

A. Arteries

B. Veins

C. Capillaries

D. Venules

Answer: B. Veins

 

47. The process by which cells engulf and digest foreign particles or damaged cells is called:

 

A. Phagocytosis

B. Osmosis

C. Active transport

D. Pinocytosis

Answer: A. Phagocytosis

 

48. Where does the process of digestion begin in the human digestive system?

 

A. Stomach

B. Small intestine

C. Mouth

D. Oesophagus

Answer: C. Mouth

 

49. What is the function of the semicircular canals in the inner ear?

 

A. Hearing

B. Balance and spatial orientation

C. Temperature regulation

D. Synthesis of earwax

Answer: B. Balance and spatial orientation

 

50. Which of the following is a function of the skeletal system?

 

A. Regulation of body temperature

B. Production of blood cells

C. Synthesis of hormones

D. Transport of oxygen in the blood

Answer: B. Production of blood cells

 

 

 

51. What is the primary function of the iris in the eye?

 

A. Refract light to focus it on the retina

B. Control the size of the pupil and regulate the amount of light entering the eye

C. Produce tears for lubrication

D. Change the shape of the lens for accommodation

Answer: B. Control the size of the pupil and regulate the amount of light entering the eye

 

52. Which of the following bones is part of the axial skeleton?

 

A. Femur

B. Radius

C. Sternum

D. Clavicle

Answer: C. Sternum

 

53. What is the function of the cochlea in the inner ear?

 

A. Balance and spatial orientation

B. Hearing and detection of sound waves

C. Production of earwax

D. Regulation of body temperature

Answer: B. Hearing and detection of sound waves

 

54. The joint between the atlas and axis vertebrae allows for:

 

A. Flexion and extension

B. Rotation of the head

C. Circumduction

D. Abduction and adduction

Answer: B. Rotation of the head

 

55. What is the primary function of the thymus gland in the immune system?

 

A. Produce antibodies

B. Regulate blood sugar levels

C. Maturation of T lymphocytes

D. Filtration of blood

Answer: C. Maturation of T lymphocytes

 

56. The term "homeostasis" refers to:

 

A. Maintenance of a constant internal environment

B. The study of cells

C. The process of digestion

D. Blood clotting

Answer: A. Maintenance of a constant internal environment

 

57. Which part of the respiratory system is responsible for exchanging gases with the bloodstream?

 

A. Bronchi

B. Alveoli

C. Trachea

D. Pharynx

Answer: B. Alveoli

 

58. What is the purpose of the meninges in the central nervous system?

 

A. Transmit nerve impulses

B. Provide protection and cushioning for the brain and spinal cord

C. Regulate blood pressure

D. Synthesize cerebrospinal fluid

Answer: B. Provide protection and cushioning for the brain and spinal cord

 

59. Which hormone is responsible for regulating water balance in the body?

 

A. Aldosterone

B. Insulin

C. Growth hormone

D. Thyroxine

Answer: A. Aldosterone

 

60. The joint between the skull and the first cervical vertebra (atlas) is known as the:

 

A. Ball and socket joint

B. Hinge joint

C. Pivot joint

D. Suture joint

Answer: C. Pivot joint

 

61. What is the role of the pineal gland in the endocrine system?

 

A. Regulation of blood sugar levels

B. Production of melatonin and regulation of sleep-wake cycle

C. Synthesis of insulin

D. Control of metabolic rate

Answer: B. Production of melatonin and regulation of sleep-wake cycle

 

62. The process by which blood vessels constrict to reduce blood flow is called:

 

A. Vasodilation

B. Hematopoiesis

C. Vasoconstriction

D. Hemostasis

Answer: C. Vasoconstriction

 

63. What is the primary function of the malleus, incus, and stapes in the middle ear?

 

A. Balance and spatial orientation

B. Amplify and transmit sound vibrations to the inner ear

C. Equalize air pressure in the ear

D. Produce earwax

Answer: B. Amplify and transmit sound vibrations to the inner ear

 

64. Which of the following structures is responsible for producing insulin in the pancreas?

 

A. Islets of Langerhans

B. Acini cells

C. Alpha cells

D. Beta cells

Answer: D. Beta cells

 

65. The process of breaking down complex food molecules into simpler forms for absorption is known as:

 

A. Respiration

B. Digestion

C. Circulation

D. Excretion

Answer: B. Digestion

 

66. Which part of the eye is responsible for detecting color and fine detail in vision?

 

A. Retina

B. Cornea

C. Lens

D. Sclera

Answer: A. Retina

 

67. What is the function of the vas deferens in the male reproductive system?

 

A. Production of sperm

B. Storage of sperm

C. Transport of sperm from the testes to the urethra

D. Synthesis of testosterone

Answer: C. Transport of sperm from the testes to the urethra

 

68. The primary function of white blood cells (leukocytes) is:

 

A. Oxygen transport

B. Clotting

C. Immune defense against pathogens

D. Nutrient absorption

Answer: C. Immune defense against pathogens

 

69. Which of the following is NOT a function of the urinary system?

 

A. Filtration of blood

B. Regulation of electrolyte balance

C. Production of insulin

D. Removal of waste products from the body

Answer: C. Production of insulin

 

70. What is the primary function of the adrenal cortex?

 

A. Production of adrenaline

B. Regulation of blood sugar levels

C. Synthesis of cortisol and aldosterone

D. Filtration of blood

Answer: C. Synthesis of cortisol and aldosterone

 

71. Which of the following is a function of the parathyroid glands?

 

A. Regulation of blood sugar levels

B. Control of metabolic rate

C. Regulation of calcium levels in the blood

D. Production of insulin

Answer: C. Regulation of calcium levels in the blood

 

72. The joint between the radius and ulna bones in the forearm is an example of a:

 

A. Pivot joint

B. Hinge joint

C. Ball and socket joint

D. Suture joint

Answer: A. Pivot joint

 

73. Where does the process of sperm maturation occur in the male reproductive system?

 

A. Testes

B. Epididymis

C. Vas deferens

D. Prostate gland

Answer: B. Epididymis

 

74. What is the primary function of the mucus produced by the respiratory system?

 

A. Oxygen transport

B. Protection and moistening of airways

C. Filtration of blood

D. Synthesis of hormones

Answer: B. Protection and moistening of airways

 

75. The process by which the body converts food into energy is known as:

 

A. Respiration

B. Metabolism

C. Digestion

D. Circulation

Answer: B. Metabolism

 

76. Which blood vessels carry oxygenated blood from the lungs to the heart?

 

A. Arteries

B. Veins

C. Capillaries

D. Venules

Answer: B. Veins

 

77. The term "haematopoiesis" refers to:

 

A. Formation of blood cells

B. Regulation of blood pressure

C. Digestion of nutrients

D. Synthesis of hormones

Answer: A. Formation of blood cells

 

78. What is the function of the epiglottis during swallowing?

 

A. Produce sound waves for speech

B. Prevent food from entering the trachea

C. Equalize air pressure in the ears

D. Regulate body temperature

Answer: B. Prevent food from entering the trachea

 

79. Which of the following is a function of the lymphatic system?

 

A. Synthesis of insulin

B. Regulation of body temperature

C. Immune defence and fluid balance

D. Filtration of blood

Answer: C. Immune defence and fluid balance

 

80. What is the primary function of the islets of Langerhans in the pancreas?

 

A. Regulation of blood pressure

B. Synthesis of insulin

C. Production of melatonin

D. Control of metabolic rate

Answer: B. Synthesis of insulin

 

81. Which of the following structures is responsible for producing cerebrospinal fluid in the brain?

 

A. Cerebral cortex

B. Hypothalamus

C. Choroid plexus

D. Medulla oblongata

Answer: C. Choroid plexus

 

82. The joint between the humerus and the ulna is an example of a:

 

A. Ball and socket joint

B. Hinge joint

C. Pivot joint

D. Gliding joint

Answer: B. Hinge joint

 

83. What is the primary function of the gallbladder in the digestive system?

 

A. Production of bile

B. Storage and concentration of bile

C. Digestion of proteins

D. Absorption of nutrients

Answer: B. Storage and concentration of bile

 

84. The process of converting glucose into energy in the presence of oxygen is called:

 

A. Anaerobic respiration

B. Glycolysis

C. Aerobic respiration

D. Fermentation

Answer: C. Aerobic respiration

 

85. Which of the following hormones is responsible for stimulating uterine contractions during childbirth?

 

A. Estrogen

B. Progesterone

C. Oxytocin

D. Prolactin

Answer: C. Oxytocin

 

86. What is the function of the semilunar valves in the heart?

 

A. Prevent backflow of blood into the atria

B. Control the flow of blood between the atria and ventricles

C. Regulate blood pressure

D. Direct blood to the pulmonary and systemic circulations

Answer: D. Direct blood to the pulmonary and systemic circulations

 

87. Where does the process of nutrient absorption primarily occur in the digestive system?

 

A. Stomach

B. Small intestine

C. Large intestine

D. Esophagus

Answer: B. Small intestine

 

88. Which of the following is a function of the spleen in the lymphatic system?

 

A. Filtration of blood and storage of platelets

B. Synthesis of insulin

C. Digestion of fats

D. Regulation of blood pressure

Answer: A. Filtration of blood and storage of platelets

 

89. The process of mitosis is involved in:

 

A. Sperm and egg production

B. Growth and repair of body tissues

C. Digestion of nutrients

D. Blood clotting

Answer: B. Growth and repair of body tissues

 

90. What is the primary function of the vestibulocochlear nerve (cranial nerve VIII)?

 

A. Control of facial muscles

B. Hearing and balance

C. Vision

D. Taste sensation

Answer: B. Hearing and balance

 

91. What is the primary function of the vas deferens in the male reproductive system?

 

A. Production of sperm

B. Storage of sperm

C. Transport of sperm from the testes to the urethra

D. Synthesis of testosterone

Answer: C. Transport of sperm from the testes to the urethra

 

92. The joint between the tibia and fibula bones in the lower leg is an example of a:

 

A. Ball and socket joint

B. Hinge joint

C. Pivot joint

D. Syndesmosis joint

Answer: D. Syndesmosis joint

 

93. What is the function of the ciliary muscle in the eye?

 

A. Control the size of the pupil

B. Refract light to focus it on the retina

C. Change the shape of the lens for accommodation

D. Produce tears for lubrication

Answer: C. Change the shape of the lens for accommodation

 

94. The process of removing waste products from the body is known as:

 

A. Digestion

B. Excretion

C. Respiration

D. Circulation

Answer: B. Excretion

 

95. What is the primary function of the corpus callosum in the brain?

 

A. Regulation of body temperature

B. Coordination of muscle movements

C. Communication between the two cerebral hemispheres

D. Synthesis of neurotransmitters

Answer: C. Communication between the two cerebral hemispheres

 

96. Which of the following hormones is responsible for stimulating the development of female secondary sexual characteristics?

 

A. Testosterone

B. Estrogen

C. Progesterone

D. Prolactin

Answer: B. Estrogen

 

97. The term "osmosis" refers to the movement of:

 

A. Water across a selectively permeable membrane

B. Oxygen in the bloodstream

C. Nutrients in the digestive system

D. Blood cells in the circulatory system

Answer: A. Water across a selectively permeable membrane

98. Which part of the brain is responsible for processing sensory information and relaying it to the cerebral cortex?

 

A. Cerebrum

B. Thalamus

C. Hypothalamus

D. Cerebellum

Answer: B. Thalamus

99. The process of converting nitrogenous wastes into urine is primarily performed by the:

A. Liver

B. Kidneys

C. Bladder

D. Urethra

Answer: B. Kidneys

 

100. What is the main function of the mucus produced by the digestive system?

A. Protection of the stomach lining

B. Lubrication of the oesophagus

C. Digestion of carbohydrates

D. Absorption of nutrients

Answer: A. Protection of the stomach lining

Ø Clinical Pathology 100 MCQS

 

1. What is the primary purpose of a complete blood count (CBC)?

A. Detection of genetic disorders

B. Evaluation of kidney function

C. Assessment of blood clotting

D. Examination of cellular elements in the blood

 

Answer: D. Examination of cellular elements in the blood

 

2. Elevated levels of creatinine in blood tests are often indicative of:

 

A. Liver dysfunction

B. Kidney dysfunction

C. Diabetes mellitus

D. Hyperthyroidism

Answer: B. Kidney dysfunction

 

3. The erythrocyte sedimentation rate (ESR) is a measure of:

 

A. Blood clotting time

B. Inflammation in the body

C. Haemoglobin concentration

D. Oxygen-carrying capacity of red blood cells

Answer: B. Inflammation in the body

 

4. Which enzyme is commonly measured to assess liver function?

 

A. Amylase

B. Creatine kinase

C. Aspartate aminotransferase (AST)

D. Lipase

Answer: C. Aspartate aminotransferase (AST)

 

5. The term "haematuria" refers to:

 

A. Elevated white blood cell count

B. Presence of blood in the urine

C. Abnormal levels of cholesterol

D. Increased platelet count

Answer: B. Presence of blood in the urine

 

6. A high level of troponin in blood tests is indicative of:

 

A. Liver disease

B. Kidney disease

C. Myocardial infarction (heart attack)

D. Respiratory disorders

Answer: C. Myocardial infarction (heart attack)

 

7. The term "polyuria" refers to:

 

A. Excessive sweating

B. Excessive urination

C. Difficulty swallowing

D. Excessive thirst

Answer: B. Excessive urination

 

8. Which blood test is commonly used to assess glucose levels over an extended period (average blood sugar over 2-3 months)?

 

A. Fasting blood sugar (FBS)

B. Oral glucose tolerance test (OGTT)

C. Glycated haemoglobin (HbA1c)

D. Random blood sugar (RBS)

Answer: C. Glycated haemoglobin (HbA1c)

 

9. The Schilling test is used to diagnose deficiencies in which vitamin?

 

A. Vitamin C

B. Vitamin B12

C. Vitamin D

D. Vitamin K

Answer: B. Vitamin B12

 

10. Which of the following is a marker for liver function and is commonly elevated in conditions such as hepatitis or cirrhosis?

 

A. Creatinine

B. Alkaline phosphatase (ALP)

C. Troponin

D. Uric acid

Answer: B. Alkaline phosphatase (ALP)

 

11. The presence of which cells in the peripheral blood is indicative of an allergic reaction or parasitic infection?

 

A. Eosinophils

B. Basophils

C. Neutrophils

D. Lymphocytes

Answer: A. Eosinophils

 

12. In a lipid profile test, which lipid is often referred to as "bad cholesterol"?

 

A. Low-density lipoprotein (LDL)

B. High-density lipoprotein (HDL)

C. Triglycerides

D. VLDL cholesterol

Answer: A. Low-density lipoprotein (LDL)

 

13. What does the Prothrombin Time (PT) measure in blood clotting tests?

 

A. Platelet function

B. Fibrinogen levels

C. Vitamin K levels

D. The extrinsic pathway of coagulation

Answer: D. The extrinsic pathway of coagulation

 

14. Which of the following blood types is considered the universal donor in blood transfusions?

 

A. A

B. B

C. AB

D. O

Answer: D. O

 

15. What is the primary function of the thyroid-stimulating hormone (TSH)?

 

A. Regulation of blood sugar levels

B. Control of metabolic rate

C. Stimulation of red blood cell production

D. Regulation of thyroid gland activity

Answer: D. Regulation of thyroid gland activity

 

16. The presence of which antibody is typically assessed in autoimmune disorders such as rheumatoid arthritis?

 

A. IgG

B. IgM

C. IgA

D. IgE

Answer: A. IgG

 

17. Which laboratory test is commonly used to evaluate kidney function and detect conditions such as glomerulonephritis or kidney failure?

 

A. Alanine aminotransferase (ALT)

B. Blood urea nitrogen (BUN)

C. Serum creatinine

D. C-reactive protein (CRP)

Answer: C. Serum creatinine

 

18. What is the purpose of a sputum culture test?

 

A. Assessing liver function

B. Detecting urinary tract infections

C. Identifying respiratory infections or tuberculosis

D. Evaluating glucose levels

Answer: C. Identifying respiratory infections or tuberculosis

 

19. The term "haemolysis" refers to:

 

A. Formation of blood cells

B. Breakdown of red blood cells

C. Clotting of blood

D. Increase in white blood cell count

Answer: B. Breakdown of red blood cells

 

20. Which of the following is a common marker for inflammation in the body and is measured in conditions such as arthritis or infections?

 

A. Serum electrolytes

B. C-reactive protein (CRP)

C. Thyroid-stimulating hormone (TSH)

D. Lipase

Answer: B. C-reactive protein (CRP)

Feel free to use these questions for educational purposes or to test your knowledge of Clinical Pathology!

 

21. What is the primary function of the D-dimer test?

 

A. Assessing liver function

B. Detecting deep vein thrombosis (DVT) or pulmonary embolism

C. Evaluating kidney function

D. Monitoring blood glucose levels

Answer: B. Detecting deep vein thrombosis (DVT) or pulmonary embolism

 

22. The presence of which antibody is tested in a rapid plasma reagin (RPR) test, which is used to diagnose syphilis?

 

A. IgG

B. IgM

C. IgA

D. VDRL antibody

Answer: D. VDRL antibody

 

23. Which enzyme is commonly elevated in conditions affecting the liver, heart, and skeletal muscles and is used as a general marker of tissue damage?

 

A. Amylase

B. Alanine aminotransferase (ALT)

C. Creatine kinase (CK)

D. Lipase

Answer: C. Creatine kinase (CK)

 

24. What is the purpose of a fecal occult blood test (FOBT)?

 

A. Assessing liver function

B. Detecting blood in the stool

C. Identifying urinary tract infections

D. Monitoring pancreatic enzymes

Answer: B. Detecting blood in the stool

 

25. The HbA1c test is primarily used to monitor and manage individuals with:

 

A. Anaemia

B. Diabetes mellitus

C. Hyperthyroidism

D. Cardiovascular disease

Answer: B. Diabetes mellitus

 

26. Which blood component is primarily responsible for transporting oxygen to tissues and removing carbon dioxide?

 

A. Platelets

B. White blood cells

C. Red blood cells (erythrocytes)

D. Plasma

Answer: C. Red blood cells (erythrocytes)

 

27. The Wright stain is commonly used in the laboratory for:

 

A. Blood smear examination

B. Urinalysis

C. Liver function tests

D. Coagulation studies

Answer: A. Blood smear examination

 

28. A blood gas analysis is often performed to assess:

 

A. Blood clotting factors

B. Acid-base balance and oxygen levels in the blood

C. Kidney function

D. Thyroid function

Answer: B. Acid-base balance and oxygen levels in the blood

 

29. Which electrolyte imbalance is associated with muscle weakness, cardiac arrhythmias, and can result from conditions such as vomiting or excessive sweating?

 

A. Hyperkalemia

B. Hypokalemia

C. Hyponatremia

D. Hypernatremia

Answer: B. Hypokalemia

 

30. The term "leucocytosis" refers to an elevated count of:

 

A. Red blood cells

B. White blood cells

C. Platelets

D. Neutrophils

Answer: B. White blood cells

31. Which of the following is a tumour marker often elevated in prostate cancer and is used for monitoring the disease?

 

A. CA 125

B. PSA (Prostate-Specific Antigen)

C. CEA (Carcinoembryonic Antigen)

D. AFP (Alpha-Fetoprotein)

Answer: B. PSA (Prostate-Specific Antigen)

 

32. A blood test measuring levels of thyroxine (T4) and triiodothyronine (T3) is used to assess the function of which gland?

 

A. Pituitary gland

B. Thyroid gland

C. Adrenal gland

D. Parathyroid gland

Answer: B. Thyroid gland

 

33. In the context of blood clotting, what does the term "INR" stand for?

 

A. International Normalized Ratio

B. Internal Nucleotide Reaction

C. Inflammatory Neutrophil Response

D. Insulin Normalization Rate

Answer: A. International Normalized Ratio

 

34. Which of the following is a test commonly used to evaluate liver function and is elevated in liver diseases such as hepatitis or cirrhosis?

 

A. Amylase

B. Lipase

C. Alkaline phosphatase (ALP)

D. Troponin

Answer: C. Alkaline phosphatase (ALP)

 

35. The MCV (Mean Corpuscular Volume) in a complete blood count (CBC) measures the average volume of:

 

A. Red blood cells

B. White blood cells

C. Platelets

D. Plasma

Answer: A. Red blood cells

 

36. What is the purpose of a bone marrow aspiration and biopsy?

 

A. Evaluate kidney function

B. Assess blood clotting factors

C. Diagnose anaemia and blood disorders

D. Monitor cardiac enzymes

Answer: C. Diagnose anaemia and blood disorders

 

37. Which of the following conditions is associated with an increased level of bilirubin in the blood?

 

A. Diabetes mellitus

B. Haemorrhage

C. Jaundice

D. Hyperthyroidism

Answer: C. Jaundice

 

38. A blood test measuring levels of anti-nuclear antibodies (ANA) is often used in the diagnosis of:

 

A. Rheumatoid arthritis

B. Systemic lupus erythematosus (SLE)

C. Diabetes mellitus

D. Multiple sclerosis

Answer: B. Systemic lupus erythematosus (SLE)

 

39. Which of the following is a marker for inflammation and is often elevated in conditions such as rheumatoid arthritis or inflammatory bowel disease?

 

A. Amylase

B. Erythrocyte sedimentation rate (ESR)

C. Troponin

D. Uric acid

Answer: B. Erythrocyte sedimentation rate (ESR)

 

40. A blood test measuring levels of B-type natriuretic peptide (BNP) is used for the diagnosis and monitoring of:

 

A. Liver function

B. Heart failure

C. Thyroid function

D. Diabetes mellitus

Answer: B. Heart failure

 

41. Which of the following is a common test used to evaluate kidney function and is a measure of the glomerular filtration rate (GFR)?

 

A. Blood urea nitrogen (BUN)

B. Creatinine kinase (CK)

C. Serum creatinine

D. Albumin

Answer: C. Serum creatinine

 

42. The term "haemoptysis" refers to:

 

A. Coughing up blood

B. Blood in the stool

C. Blood in the urine

D. Blood in the sputum

Answer: A. Coughing up blood

 

43. What is the primary function of the International Normalized Ratio (INR) in blood clotting tests?

 

A. Assessing platelet function

B. Evaluating fibrinogen levels

C. Standardizing prothrombin time across different laboratories

D. Monitoring white blood cell activity

Answer: C. Standardizing prothrombin time across different laboratories

 

44. A faecal elastase test is commonly used for the diagnosis of:

 

A. Pancreatitis

B. Gastritis

C. Inflammatory bowel disease (IBD)

D. Hepatitis

Answer: A. Pancreatitis

 

45. What is the primary function of the C-reactive protein (CRP) in blood tests?

 

A. Marking red blood cells for destruction

B. Assessing liver function

C. Indicating inflammation in the body

D. Monitoring blood clotting factors

Answer: C. Indicating inflammation in the body

 

46. Which electrolyte imbalance is associated with muscle cramps, weakness, and cardiac arrhythmias?

 

A. Hyponatremia

B. Hypernatremia

C. Hypokalaemia

D. Hyperkalaemia

Answer: C. Hypokalaemia

 

47. The term "thrombocytopenia" refers to a deficiency of:

 

A. Red blood cells

B. White blood cells

C. Platelets

D. Plasma proteins

Answer: C. Platelets

 

48. A sputum culture and sensitivity test is commonly performed to identify the cause of:

 

A. Urinary tract infections

B. Respiratory infections

C. Gastrointestinal bleeding

D. Liver dysfunction

Answer: B. Respiratory infections

 

49. The Reticulocyte Count measures the percentage of:

 

A. Immature red blood cells

B. Mature red blood cells

C. White blood cells

D. Platelets

Answer: A. Immature red blood cells

 

50. Which of the following is a test used to assess the function of the exocrine pancreas and is often elevated in conditions like cystic fibrosis?

 

A. Lipase

B. Amylase

C. Trypsinogen

D. Faecal elastase

Answer: D. Faecal elastase

 

51. What is the primary purpose of a rheumatoid factor (RF) test in blood analysis?

 

A. Evaluate kidney function

B. Diagnose anemia

C. Identify autoimmune disorders, particularly rheumatoid arthritis

D. Monitor cardiac enzymes

Answer: C. Identify autoimmune disorders, particularly rheumatoid arthritis

 

52. The presence of which antibody is often tested in a serological test for infectious mononucleosis (mono)?

 

A. IgG

B. IgM

C. IgA

D. IgE

Answer: B. IgM

 

53. A CSF (cerebrospinal fluid) analysis is commonly performed to diagnose conditions affecting the:

 

A. Liver

B. Kidneys

C. Central nervous system

D. Lungs

Answer: C. Central nervous system

 

54. The term "hyperglycaemia" refers to elevated levels of:

 

A. Cholesterol in the blood

B. Glucose in the blood

C. Urea in the blood

D. Haemoglobin in the blood

Answer: B. Glucose in the blood

 

55. A test measuring levels of thyroid-stimulating hormone (TSH) is primarily used to assess the function of the:

 

A. Thyroid gland

B. Adrenal gland

C. Pituitary gland

D. Parathyroid gland

Answer: C. Pituitary gland

 

56. The term "dyslipidaemia" refers to abnormalities in the levels of:

 

A. Red blood cells

B. White blood cells

C. Platelets

D. Lipids (cholesterol and triglycerides) in the blood

Answer: D. Lipids (cholesterol and triglycerides) in the blood

 

57. A blood test measuring levels of troponin is often used to diagnose and monitor:

 

A. Liver function

B. Kidney function

C. Myocardial infarction (heart attack)

D. Respiratory disorders

Answer: C. Myocardial infarction (heart attack)

 

58. Which of the following is a common test used to assess coagulation factors and monitor anticoagulant therapy?

 

A. Prothrombin Time (PT)

B. Activated Partial Thromboplastin Time (APTT)

C. Fibrinogen assay

D. D-dimer test

Answer: A. Prothrombin Time (PT)

 

59. The term "erythropoiesis" refers to the production of:

 

A. Red blood cells

B. White blood cells

C. Platelets

D. Plasma proteins

Answer: A. Red blood cells

 

60. A blood test measuring levels of alpha-fetoprotein (AFP) is commonly used for the screening and monitoring of:

 

A. Liver function

B. Pregnancy-associated complications

C. Cardiovascular disease

D. Prostate cancer

Answer: B. Pregnancy-associated complications

 

61. The Schilling test is used to diagnose deficiencies in which vitamin?

 

A. Vitamin C

B. Vitamin B12

C. Vitamin D

D. Vitamin K

Answer: B. Vitamin B12

 

62. A blood test measuring levels of anti-cyclic citrullinated peptide (anti-CCP) antibodies is often associated with the diagnosis of:

 

A. Multiple sclerosis

B. Systemic lupus erythematosus (SLE)

C. Rheumatoid arthritis

D. Crohn's disease

Answer: C. Rheumatoid arthritis

 

63. Which of the following is a tumour marker commonly elevated in ovarian cancer?

 

A. CA 125

B. PSA (Prostate-Specific Antigen)

C. CEA (Carcinoembryonic Antigen)

D. AFP (Alpha-Fetoprotein)

Answer: A. CA 125

 

64. A blood gas analysis is often performed to assess:

 

A. Blood clotting factors

B. Acid-base balance and oxygen levels in the blood

C. Kidney function

D. Thyroid function

Answer: B. Acid-base balance and oxygen levels in the blood

 

65. The term "haemolysis" refers to:

 

A. Formation of blood cells

B. Breakdown of red blood cells

C. Clotting of blood

D. Increase in white blood cell count

Answer: B. Breakdown of red blood cells

 

66. A blood test measuring levels of carcinoembryonic antigen (CEA) is commonly used for monitoring:

 

A. Liver function

B. Thyroid function

C. Cancer recurrence, especially in colorectal cancer

D. Blood glucose levels

Answer: C. Cancer recurrence, especially in colorectal cancer

 

67. The term "oliguria" refers to:

 

A. Excessive sweating

B. Decreased urine output

C. Excessive urination

D. Increased thirst

Answer: B. Decreased urine output

 

68. A blood test measuring levels of lactate dehydrogenase (LDH) is used as a marker for:

 

A. Liver function

B. Kidney function

C. Cardiac injury and tissue damage

D. Pancreatic enzymes

Answer: C. Cardiac injury and tissue damage

 

69. The term "thrombophilia" refers to a tendency for:

 

A. Excessive bleeding

B. Blood clotting disorders

C. Abnormal white blood cell count

D. Anaemia

Answer: B. Blood clotting disorders

 

70. A blood test measuring levels of cortisol is used to assess the function of the:

 

A. Pancreas

B. Adrenal gland

C. Thyroid gland

D. Pituitary gland

Answer: B. Adrenal gland

 

71. The term "thrombocytosis" refers to an elevated count of:

 

A. Red blood cells

B. White blood cells

C. Platelets

D. Neutrophils

Answer: C. Platelets

 

72. A blood test measuring levels of antithyroid antibodies (such as anti-TPO antibodies) is associated with the diagnosis of:

 

A. Type 1 diabetes

B. Hypothyroidism

C. Hashimoto's thyroiditis

D. Cushing's syndrome

Answer: C. Hashimoto's thyroiditis

 

73. The term "thrombus" refers to:

 

A. Excessive bleeding

B. A blood clot that forms and remains in place within a blood vessel

C. Anaemia

D. Decreased platelet count

Answer: B. A blood clot that forms and remains in place within a blood vessel

 

74. A blood test measuring levels of IgE antibodies is often used to diagnose and monitor:

 

A. Autoimmune disorders

B. Allergic conditions

C. Thyroid disorders

D. Cardiac disorders

Answer: B. Allergic conditions

 

75. The term "pyuria" refers to the presence of:

 

A. Blood in the urine

B. Protein in the urine

C. Pus in the urine

D. Glucose in the urine

Answer: C. Pus in the urine

 

76. What is the primary function of a prothrombin time (PT) test?

 

A. Assessing liver function

B. Monitoring blood glucose levels

C. Evaluating blood clotting factors

D. Measuring levels of hemoglobin

Answer: C. Evaluating blood clotting factors

 

77. A blood test measuring levels of amylase and lipase is commonly used for the diagnosis of:

 

A. Liver disease

B. Pancreatitis

C. Kidney disease

D. Diabetes mellitus

Answer: B. Pancreatitis

 

78. The term "hepatomegaly" refers to:

 

A. Enlarged liver

B. Enlarged kidney

C. Enlarged heart

D. Enlarged spleen

Answer: A. Enlarged liver

 

79. A serum iron test is often used to assess:

 

A. Kidney function

B. Thyroid function

C. Iron levels in the blood

D. Pancreatic function

Answer: C. Iron levels in the blood

 

80. Which of the following is a marker for inflammation and is commonly elevated in conditions such as rheumatoid arthritis or inflammatory bowel disease?

 

A. Serum electrolytes

B. C-reactive protein (CRP)

C. Thyroid-stimulating hormone (TSH)

D. Lipase

Answer: B. C-reactive protein (CRP)

 

81. The term "thrombophlebitis" refers to:

 

A. Inflammation of the blood vessels

B. Formation of blood clots in the veins with inflammation

C. Clotting of blood in the arteries

D. Abnormal enlargement of blood vessels

Answer: B. Formation of blood clots in the veins with inflammation

 

82. A blood test measuring levels of IgA antibodies is associated with the diagnosis of:

 

A. Celiac disease

B. Lupus erythematosus

C. Type 2 diabetes

D. Rheumatoid arthritis

Answer: A. Celiac disease

 

83. The term "lymphocytosis" refers to an elevated count of:

 

A. Red blood cells

B. White blood cells, specifically lymphocytes

C. Platelets

D. Neutrophils

Answer: B. White blood cells, specifically lymphocytes

 

84. A blood test measuring levels of CA 19-9 is commonly used as a tumour marker for:

 

A. Colorectal cancer

B. Pancreatic cancer

C. Breast cancer

D. Lung cancer

Answer: B. Pancreatic cancer

 

85. The term "erythema" refers to:

 

A. Yellowing of the skin

B. Redness of the skin due to increased blood flow

C. Bruising of the skin

D. Darkening of the skin

Answer: B. Redness of the skin due to increased blood flow

 

86. Which blood test is commonly used to assess liver function and is elevated in conditions such as hepatitis or cirrhosis?

 

A. Amylase

B. Alanine aminotransferase (ALT)

C. Aspartate aminotransferase (AST)

D. Lipase

Answer: B. Alanine aminotransferase (ALT)

 

87. The term "proteinuria" refers to the presence of:

 

A. Blood in the urine

B. Protein in the urine

C. Pus in the urine

D. Glucose in the urine

Answer: B. Protein in the urine

 

88. A blood test measuring levels of homocysteine is often used as a risk marker for:

 

A. Osteoporosis

B. Cardiovascular disease

C. Type 1 diabetes

D. Chronic kidney disease

Answer: B. Cardiovascular disease

 

89. The term "leukopenia" refers to a deficiency of:

 

A. Red blood cells

B. White blood cells

C. Platelets

D. Neutrophils

Answer: B. White blood cells

 

90. A blood test measuring levels of rheumatoid factor (RF) is commonly used in the diagnosis of:

 

A. Osteoarthritis

B. Rheumatoid arthritis

C. Gout

D. Lupus erythematosus

Answer: B. Rheumatoid arthritis

 

91. The presence of "Ketones" in the urine is often associated with:

 

A. Liver disease

B. Diabetes mellitus

C. Thyroid disorders

D. Kidney disease

Answer: B. Diabetes mellitus

 

92. Which of the following is a test commonly used to evaluate the function of the adrenal glands?

 

A. Thyroid function test

B. Adrenocorticotropic hormone (ACTH) stimulation test

C. C-reactive protein (CRP) test

D. Liver function test

Answer: B. Adrenocorticotropic hormone (ACTH) stimulation test

 

93. The term "thrombocythemia" refers to an elevated count of:

 

A. Red blood cells

B. White blood cells

C. Platelets

D. Lymphocytes

Answer: C. Platelets

 

94. A blood test measuring levels of parathyroid hormone (PTH) is used to assess:

 

A. Thyroid function

B. Pancreatic function

C. Parathyroid gland function

D. Kidney function

Answer: C. Parathyroid gland function

 

95. The term "haematuria" refers to:

 

A. Coughing up blood

B. Blood in the stool

C. Blood in the urine

D. Blood in the sputum

Answer: C. Blood in the urine

 

96. A blood test measuring levels of anti-double-stranded DNA (anti-dsDNA) antibodies is associated with the diagnosis of:

 

A. Multiple sclerosis

B. Systemic lupus erythematosus (SLE)

C. Rheumatoid arthritis

D. Crohn's disease

Answer: B. Systemic lupus erythematosus (SLE)

 

97. The term "polycythaemia" refers to an elevated count of:

 

A. Red blood cells

B. White blood cells

C. Platelets

D. Lymphocytes

Answer: A. Red blood cells

 

98. A blood test measuring levels of creatine kinase-MB (CK-MB) is often used to diagnose and monitor:

 

A. Liver function

B. Kidney function

C. Cardiac injury, particularly myocardial infarction (heart attack)

D. Respiratory disorders

Answer: C. Cardiac injury, particularly myocardial infarction (heart attack)

 

99. The term "hypercalcemia" refers to elevated levels of:

 

A. Calcium in the blood

B. Potassium in the blood

C. Sodium in the blood

D. Magnesium in the blood

Answer: A. Calcium in the blood

 

100. A blood test measuring levels of alpha-1-antitrypsin is often used in the assessment of:

 

A. Liver function

B. Lung function

C. Thyroid function

D. Pancreatic function

Answer: B. Lung function

 

Ø Medical Laboratory Instrumentation

1. What is the primary function of a spectrophotometer in a clinical laboratory?

 

A. Measurement of blood pressure

B. Detection of microorganisms

C. Quantification of chemical substances in a sample

D. Analysis of clotting factors

Answer: C. Quantification of chemical substances in a sample

 

2. Flow cytometry is commonly used in which area of laboratory testing?

 

A. Hematology

B. Microbiology

C. Clinical Chemistry

D. Immunology

Answer: A. Hematology

 

3. Which instrument is used for the automated counting and sizing of blood cells?

 

A. Flow cytometer

B. Coagulation analyzer

C. Hemocytometer

D. Microplate reader

Answer: A. Flow cytometer

 

4. In clinical chemistry, what is the purpose of a chromatograph?

 

A. Separation and identification of components in a mixture

B. Detection of bacterial growth

C. Quantification of white blood cells

D. Measurement of clotting time

Answer: A. Separation and identification of components in a mixture

 

5. What is the primary function of an immunoassay analyzer in the laboratory?

 

A. Identification of bacteria

B. Quantification of blood cells

C. Detection of antibodies and antigens

D. Measurement of electrolytes

Answer: C. Detection of antibodies and antigens

 

6. Which of the following instruments is commonly used for analyzing genetic material in molecular diagnostics?

 

A. Spectrophotometer

B. Flow cytometer

C. Polymerase Chain Reaction (PCR) machine

D. Coagulation analyzer

Answer: C. Polymerase Chain Reaction (PCR) machine

 

7. Point-of-Care Testing (POCT) devices are designed for:

 

A. High-throughput automated testing

B. On-the-spot testing near the patient

C. Analysis of genetic material

D. Microbial identification

Answer: B. On-the-spot testing near the patient

 

8. What does LIS stand for in the context of laboratory instrumentation?

 

A. Laboratory Information System

B. Liquid Ionization Spectrometer

C. Laboratory Instrumentation Standard

D. Local Instrument Storage

Answer: A. Laboratory Information System

 

9. What is the purpose of quality control in medical laboratory instrumentation?

 

A. To increase the cost of testing

B. To ensure accurate and reliable test results

C. To replace faulty instruments

D. To reduce laboratory efficiency

Answer: B. To ensure accurate and reliable test results

 

10. Which technology is often employed for the rapid analysis of urine samples in a clinical laboratory?

- A. Spectrophotometry

- B. Microplate reader

- C. Automated urine analyzer

- D. PCR machine

 

 

**Answer: C. Automated urine analyser**

 

11. What is the primary function of a coagulation analyzer in a clinical laboratory?

 

A. Measurement of blood glucose levels

B. Detection of microorganisms in blood

C. Analysis of blood clotting factors

D. Quantification of white blood cells

Answer: C. Analysis of blood clotting factors

 

12. Which of the following is an example of a point-of-care testing (POCT) device?

 

A. Hematology analyzer

B. Centrifuge

C. Glucometer

D. PCR machine

Answer: C. Glucometer

 

13. The purpose of a microplate reader in the laboratory is to:

 

A. Analyze blood gases

B. Measure absorbance or fluorescence in microplate assays

C. Perform blood typing

D. Separate blood components

Answer: B. Measure absorbance or fluorescence in microplate assays

 

14. What is the primary function of a centrifuge in the clinical laboratory?

 

A. Separation of blood components based on density

B. Measurement of blood pressure

C. Detection of antibodies and antigens

D. Analysis of genetic material

Answer: A. Separation of blood components based on density

 

15. Which instrument is commonly used for the analysis of blood gases?

 

A. Hemocytometer

B. Spectrophotometer

C. Blood gas analyzer

D. Coagulation analyzer

Answer: C. Blood gas analyzer

 

16. What does ELISA stand for in the context of laboratory testing?

 

A. Enzyme-Linked Immunosorbent Assay

B. Extended Laboratory Instrument System Analysis

C. Electron-Linked Instrument Sensitivity Assessment

D. Enzymatic Laboratory Integrated System Analyzer

Answer: A. Enzyme-Linked Immunosorbent Assay

 

17. The technique of PCR (Polymerase Chain Reaction) is primarily used for:

 

A. Separation of blood components

B. Analysis of blood clotting factors

C. Amplification of DNA segments

D. Detection of antibodies

Answer: C. Amplification of DNA segments

 

18. In laboratory automation, what is the aim of integrating automated systems?

 

A. To increase manual workload

B. To decrease overall efficiency

C. To streamline and optimize laboratory workflow

D. To reduce the number of tests performed

Answer: C. To streamline and optimize laboratory workflow

 

19. Which of the following is an example of a pre-analytical factor that can affect laboratory results?

 

A. Instrument calibration

B. Sample collection and handling

C. Quality control procedures

D. LIS implementation

Answer: B. Sample collection and handling

 

20. What is the primary purpose of a LIS (Laboratory Information System) in a clinical laboratory?

- A. Analysis of blood components

- B. Storage of laboratory instruments

- C. Management of laboratory information and data

- D. Maintenance of quality control procedures

 

**Answer: C. Management of laboratory information and data**

Ø Clinical Microscopy

1. Which of the following structures is NOT commonly observed in a routine urinalysis microscopic examination?

 

A. Red blood cells (RBCs)

B. White blood cells (WBCs)

C. Epithelial cells

D. Platelets

Answer: D. Platelets

 

2. A "hyaline cast" observed in urine microscopy is primarily composed of:

 

A. Red blood cells

B. White blood cells

C. Epithelial cells

D. Protein

Answer: D. Protein

 

3. Which of the following conditions is associated with the presence of "dysmorphic red blood cells" in urine microscopy?

 

A. Glomerulonephritis

B. Urinary tract infection (UTI)

C. Renal calculi

D. Diabetes mellitus

Answer: A. Glomerulonephritis

 

4. The presence of "bacteria" in urine microscopy may indicate:

 

A. Diabetes mellitus

B. Renal calculi

C. Urinary tract infection (UTI)

D. Glomerulonephritis

Answer: C. Urinary tract infection (UTI)

 

5. Which of the following crystals is commonly observed in urine microscopy and is associated with ethylene glycol poisoning?

 

A. Calcium oxalate

B. Triple phosphate

C. Uric acid

D. Cystine

Answer: A. Calcium oxalate

 

6. The presence of "oval fat bodies" in urine microscopy is suggestive of:

 

A. Diabetic nephropathy

B. Nephrotic syndrome

C. Glomerulonephritis

D. Urinary tract infection (UTI)

Answer: B. Nephrotic syndrome

 

7. Which of the following structures, when observed in urine microscopy, is indicative of renal tubular injury or inflammation?

 

A. Casts

B. Red blood cells (RBCs)

C. White blood cells (WBCs)

D. Epithelial cells

Answer: D. Epithelial cells

 

8. A "waxy cast" observed in urine microscopy may indicate:

 

A. Acute kidney injury

B. Chronic kidney disease

C. Urinary tract infection (UTI)

D. Renal calculi

Answer: B. Chronic kidney disease

 

9. "Hyaline casts" are primarily composed of:

 

A. Protein

B. Red blood cells (RBCs)

C. White blood cells (WBCs)

D. Epithelial cells

Answer: A. Protein

 

10. The presence of "triple phosphate crystals" in urine microscopy is indicative of:

- A. Urinary tract infection (UTI)

- B. Renal calculi

- C. Diabetic nephropathy

- D. Glomerulonephritis

 

**Answer: A. Urinary tract infection (UTI)**

 

11. "Casts" observed in urine microscopy are formed in the:

- A. Renal tubules

- B. Bladder

- C. Ureters

- D. Urethra

**Answer: A. Renal tubules**

12. Which of the following conditions is associated with the presence of "bilirubin" in urine microscopy?

- A. Hematuria

- B. Liver disease

- C. Glomerulonephritis

- D. Diabetes mellitus

 

**Answer: B. Liver disease**

13. The formation of "uric acid crystals" in urine is influenced by:

- A. Alkaline pH

- B. Neutral pH

- C. Acidic pH

- D. High glucose levels

 

**Answer: C. Acidic pH**

14. The presence of "mucus threads" in urine microscopy may be indicative of:

- A. Normal physiological conditions

- B. Urinary tract infection (UTI)

- C. Diabetes mellitus

- D. Renal calculi

**Answer: A. Normal physiological conditions**

15. "Calcium phosphate crystals" in urine microscopy are more likely to form in an environment with:

- A. Alkaline pH

- B. Neutral pH

- C. Acidic pH

- D. High glucose levels

**Answer: A. Alkaline pH**

 

16. The presence of "fatty casts" in urine microscopy may suggest:

- A. Nephrotic syndrome

- B. Urinary tract infection (UTI)

- C. Glomerulonephritis

- D. Renal calculi

 

**Answer: A. Nephrotic syndrome**

17. "Tyrosine crystals" in urine microscopy may be observed in cases of:

- A. Liver disease

- B. Phenylketonuria (PKU)

- C. Diabetes mellitus

- D. Haematuria

**Answer: B. Phenylketonuria (PKU)**

18. The presence of "hyaline casts" in urine may be seen in conditions such as:

- A. Diabetes mellitus

- B. Dehydration

- C. Urinary tract infection (UTI)

- D. Glomerulonephritis

 

**Answer: B. Dehydration**

19. "Squamous epithelial cells" seen in urine microscopy are typically derived from the:

- A. Renal tubules

- B. Urethra

- C. Bladder

- D. Prostate gland

 

**Answer: B. Urethra**

20. The presence of "crystals with envelope-like shapes" in urine microscopy is characteristic of:

- A. Cystine crystals

- B. Calcium oxalate crystals

- C. Triple phosphate crystals

- D. Uric acid crystals

**Answer: C. Triple phosphate crystals**

 

21. "Crenated red blood cells" in urine microscopy may be caused by:

- A. Dehydration

- B. Diabetes mellitus

- C. Urinary tract infection (UTI)

- D. Renal calculi

 

**Answer: A. Dehydration**

22. The presence of "amorphous urates" in urine microscopy is associated with:

- A. Acidic urine pH

- B. Alkaline urine pH

- C. Neutral urine pH

- D. High glucose levels

 

**Answer: A. Acidic urine pH**

23. "Renal tubular epithelial cells" observed in urine microscopy may indicate:

- A. Normal physiological conditions

- B. Glomerulonephritis

- C. Tubular injury or inflammation

- D. Urinary tract infection (UTI)

 

**Answer: C. Tubular injury or inflammation**

24. The presence of "yeast cells" in urine microscopy may suggest:

- A. Fungal infection

- B. Normal flora

- C. Renal calculi

- D. Glomerulonephritis

 

**Answer: A. Fungal infection**

25. Which of the following is a common cause of "hematuria" in urine microscopy?

- A. Hyperglycemia

- B. Liver disease

- C. Kidney stones

- D. Elevated blood pressure

 

**Answer: C. Kidney stones**

26. "Bilirubinuria" is associated with:

- A. Liver disease

- B. Diabetes mellitus

- C. Hematuria

- D. Dehydration

 

**Answer: A. Liver disease**

27. The term "birefringent crystals" in urine microscopy refers to crystals that exhibit:

- A. No color under polarized light

- B. Two different colors under polarized light

- C. Fluorescence

- D. Phosphorescence

 

**Answer: B. Two different colors under polarized light**

28. "Calcium oxalate crystals" in urine microscopy are commonly associated with:

- A. Acidic urine pH

- B. Alkaline urine pH

- C. Normal urine pH

- D. Elevated blood pressure

 

**Answer: A. Acidic urine pH**

29. The presence of "mucus casts" in urine microscopy may be indicative of:

- A. Normal physiological conditions

- B. Renal calculi

- C. Urinary tract infection (UTI)

- D. Chronic kidney disease

 

**Answer: C. Urinary tract infection (UTI)**

30. "Brown granular casts" in urine microscopy may be seen in cases of:

- A. Dehydration

- B. Diabetes mellitus

- C. Glomerulonephritis

- D. Acute tubular necrosis

 

**Answer: D. Acute tubular necrosis**

Ø Quality Control and Quality Assurance

1. What is the primary goal of Quality Control (QC) in a medical laboratory?

 

A. Minimize costs

B. Maximize workload

C. Ensure accuracy and reliability of test results

D. Increase turnaround time

Answer: C. Ensure accuracy and reliability of test results

 

2. Which of the following is a component of Quality Assurance (QA)?

 

A. Calibration of instruments

B. Routine maintenance

C. Proficiency testing

D. Patient sample analysis

Answer: C. Proficiency testing

 

3. The term "analytical quality" in the laboratory context refers to:

 

A. Precision and accuracy of test results

B. Timeliness of reporting

C. Number of tests performed per day

D. Physical layout of the laboratory

Answer: A. Precision and accuracy of test results

 

4. What is the purpose of an External Quality Assessment (EQA) program in a medical laboratory?

 

A. Monitor employee performance

B. Assess the laboratory's physical infrastructure

C. Evaluate the laboratory's compliance with regulations

D. Assess the laboratory's analytical performance compared to other laboratories

Answer: D. Assess the laboratory's analytical performance compared to other laboratories

 

5. The "mean" in statistical terms represents:

 

A. The middle value of a set of data

B. The most frequently occurring value

C. The average of a set of data

D. The range of values

Answer: C. The average of a set of data

 

6. In the context of Quality Control, a "control chart" is used to monitor:

 

A. Employee attendance

B. Instrument calibration

C. Daily weather conditions

D. Analytical performance over time

Answer: D. Analytical performance over time

 

7. The process of comparing laboratory test results with a reference method or a reference laboratory is known as:

 

A. Calibration

B. Proficiency testing

C. Quality Control

D. External Quality Assessment

Answer: B. Proficiency testing

 

8. "Trends" in Quality Control data may indicate:

 

A. Stability of the analytical process

B. Consistent precision

C. Systematic errors or shifts in performance

D. Random errors

Answer: C. Systematic errors or shifts in performance

 

9. "Random errors" in laboratory testing are best identified by:

 

A. Control charts

B. Proficiency testing

C. Calibration curves

D. Internal audits

Answer: A. Control charts

 

10. The primary purpose of Internal Quality Control (IQC) is to:

- A. Compare laboratory performance with other laboratories

- B. Monitor the stability and precision of analytical methods

- C. Assess employee competency

- D. Validate new testing methods

 

**Answer: B. Monitor the stability and precision of analytical methods**

 

11. What is the purpose of a "blank" in analytical chemistry and Quality Control?

- A. To test the instrument's upper limit

- B. To assess reagent purity and contamination

- C. To calibrate the instrument

- D. To measure the lower limit of detection

 

**Answer: B. To assess reagent purity and contamination**

12. Which statistical measure is used to describe the spread or dispersion of a set of data points?

- A. Mean

- B. Median

- C. Range

- D. Standard deviation

 

**Answer: D. Standard deviation**

13. In Quality Control, the term "precision" refers to:

- A. The accuracy of test results

- B. The reproducibility and consistency of test results

- C. The absence of random errors

- D. The agreement between test results and a reference method

 

**Answer: B. The reproducibility and consistency of test results**

14. "Six Sigma" in Quality Management refers to:

- A. A statistical measure of process capability

- B. Six different quality control methods

- C. Six phases of quality improvement

- D. A type of laboratory accreditation

 

**Answer: A. A statistical measure of process capability**

15. The term "bias" in the context of laboratory testing refers to:

- A. The precision of test results

- B. The accuracy of test results

- C. The variability of test results

- D. The stability of the analytical process

 

**Answer: B. The accuracy of test results**

16. A laboratory that participates in an External Quality Assessment (EQA) program receives samples from an external organization to assess:

- A. Employee performance

- B. Analytical performance compared to other laboratories

- C. Compliance with safety regulations

- D. Patient satisfaction

 

**Answer: B. Analytical performance compared to other laboratories**

17. The process of "calibration" in laboratory instruments involves:

- A. Comparing results with a reference laboratory

- B. Adjusting the instrument to achieve accurate and reliable results

- C. Performing proficiency testing

- D. Monitoring trends in quality control data

 

**Answer: B. Adjusting the instrument to achieve accurate and reliable results**

18. "Verification" of a laboratory test method involves:

- A. Assessing employee competency

- B. Assessing instrument calibration

- C. Confirming the accuracy and reliability of a test method

- D. Participating in proficiency testing

 

**Answer: C. Confirming the accuracy and reliability of a test method**

19. The "Youden Index" in laboratory quality control is used to optimize:

- A. Sensitivity

- B. Specificity

- C. Precision

- D. Accuracy

 

**Answer: C. Precision**

20. In a laboratory, the "reference range" or "normal range" is established based on:

- A. Historical data from the laboratory

- B. Expert opinion

- C. Population-based studies

- D. Regulatory requirements

 

**Answer: C. Population-based studies**

 

21. What is the primary purpose of "reagent blanks" in laboratory testing?

- A. To check the accuracy of instruments

- B. To monitor the stability of reagents

- C. To assess reagent contamination

- D. To calibrate the instrument

 

**Answer: C. To assess reagent contamination**

22. In laboratory quality control, "accuracy" is defined as:

- A. The ability to obtain the same result repeatedly

- B. The closeness of a measured value to the true value

- C. The absence of systematic errors

- D. The precision of test results

 

**Answer: B. The closeness of a measured value to the true value**

23. The term "total allowable error" (TEa) in laboratory quality control refers to:

- A. The maximum acceptable variation from the reference value

- B. The total number of errors allowed in a given period

- C. The total error due to random errors

- D. The allowable errors in proficiency testing

 

**Answer: A. The maximum acceptable variation from the reference value**

24. "Sigma metrics" in laboratory quality control are used to assess:

- A. Precision

- B. Bias

- C. Analytical performance

- D. Proficiency testing results

 

**Answer: C. Analytical performance**

25. In a laboratory, "turnaround time" refers to:

- A. The time taken to perform a single test

- B. The time taken to calibrate instruments

- C. The time taken to report results after sample receipt

- D. The time taken to conduct proficiency testing

 

**Answer: C. The time taken to report results after sample receipt**

26. "Control limits" in quality control charts are established based on:

- A. Regulatory requirements

- B. Laboratory budget constraints

- C. Historical laboratory data

- D. Expert opinion

 

 

**Answer: C. Historical laboratory data**

27. Which of the following is a pre-analytical factor that can impact laboratory test results?

- A. Calibration errors

- B. Instrument precision

- C. Sample collection and handling

- D. Proficiency testing

 

 

**Answer: C. Sample collection and handling**

28. "Levey-Jennings charts" are commonly used in quality control to monitor:

- A. Instrument calibration

- B. Daily laboratory workload

- C. Proficiency testing results

- D. Analytical performance over time

 

**Answer: D. Analytical performance over time**

29. "Risk management" in laboratory quality assurance involves:

- A. Reducing the number of tests performed

- B. Identifying and mitigating potential sources of error

- C. Increasing the laboratory budget

- D. Enhancing employee productivity

 

**Answer: B. Identifying and mitigating potential sources of error**

30. The process of "verification" of a laboratory test method includes:

- A. Routine maintenance of instruments

- B. Confirming the accuracy and reliability of the test method

- C. Employee training programs

- D. Participation in proficiency testing

 

**Answer: B. Confirming the accuracy and reliability of the test method

Ø Clinical Training

 

1. What is the primary goal of clinical training for healthcare professionals?

 

A. Enhancing theoretical knowledge only

B. Developing practical skills and competencies

C. Memorizing medical textbooks

D. Completing coursework assignments

Answer: B. Developing practical skills and competencies

 

2. In clinical training, what does the term "bedside manner" refer to?

 

A. The cleanliness of hospital beds

B. The ability to communicate effectively with patients

C. The arrangement of furniture in patient rooms

D. The time spent at the patient's bedside

Answer: B. The ability to communicate effectively with patients

 

3. Which of the following is a crucial aspect of clinical training for medical professionals?

 

A. Learning only from textbooks

B. Developing empathy and compassion

C. Focusing solely on individual tasks

D. Avoiding patient interaction

Answer: B. Developing empathy and compassion

 

4. What role does clinical simulation play in healthcare training?

 

A. Providing theoretical knowledge only

B. Replacing hands-on clinical experience

C. Simulating real-world scenarios for practice

D. Allowing trainees to skip practical training

Answer: C. Simulating real-world scenarios for practice

 

5. In clinical training, what is the purpose of case-based learning?

 

A. Memorizing medical facts

B. Applying theoretical knowledge to real cases

C. Avoiding patient interaction

D. Ignoring real-life scenarios

Answer: B. Applying theoretical knowledge to real cases

 

6. What is the significance of interprofessional education in clinical training?

 

A. Isolating healthcare professionals from each other

B. Enhancing teamwork and collaboration

C. Reducing communication skills

D. Promoting individualistic approaches

Answer: B. Enhancing teamwork and collaboration

 

7. How does clinical training contribute to continuous professional development in healthcare?

 

A. By limiting exposure to practical experiences

B. By promoting a static approach to learning

C. By encouraging lifelong learning and adaptation

D. By relying solely on initial education

Answer: C. By encouraging lifelong learning and adaptation

 

8. What is the role of preceptorship in clinical training?

 

A. Excluding experienced professionals from training programs

B. Providing guidance and mentorship to learners

C. Ignoring the importance of practical skills

D. Focusing solely on theoretical knowledge

Answer: B. Providing guidance and mentorship to learners

 

9. In clinical training, what does the term "continuing education" refer to?

 

A. Completing one-time training programs

B. Pursuing education throughout one's career

C. Relying solely on initial education

D. Ignoring the need for ongoing learning

Answer: B. Pursuing education throughout one's career

 

10. What is the importance of reflective practice in clinical training?

- A. Avoiding self-assessment and improvement

- B. Focusing solely on technical skills

- C. Promoting self-awareness and critical thinking

- D. Disregarding the impact of training on patient care

 

**Answer: C. Promoting self-awareness and critical thinking**

 

11. What does the term "hands-on training" imply in the context of clinical education?

- A. Learning solely from textbooks

- B. Emphasizing theoretical knowledge

- C. Actively engaging in practical experiences

- D. Avoiding practical skills development

 

**Answer: C. Actively engaging in practical experiences**

12. In clinical training, what is the significance of cultural competence?

- A. Ignoring diversity in patient populations

- B. Understanding and respecting diverse patient cultures

- C. Exclusively focusing on medical knowledge

- D. Neglecting communication skills

 

**Answer: B. Understanding and respecting diverse patient cultures**

13. What role does feedback play in the clinical training process?

- A. Discouraging trainees from improvement

- B. Providing guidance and assessment for improvement

- C. Being irrelevant to the learning process

- D. Undermining the importance of practical skills

 

**Answer: B. Providing guidance and assessment for improvement**

14. How does clinical training contribute to the development of critical thinking skills in healthcare professionals?

- A. By discouraging independent thought

- B. By promoting rote memorization

- C. By encouraging problem-solving and analysis

- D. By relying solely on theoretical knowledge

 

**Answer: C. By encouraging problem-solving and analysis**

15. The term "preceptor" refers to:

- A. A trainee in a clinical program

- B. A seasoned professional who provides guidance to learners

- C. A theoretical concept in medical education

- D. A type of clinical simulation tool

 

**Answer: B. A seasoned professional who provides guidance to learners**

16. What is the primary focus of clinical training evaluations?

- A. Assessing only theoretical knowledge

- B. Identifying trainees' weaknesses for punitive measures

- C. Providing constructive feedback for improvement

- D. Avoiding performance assessments

 

**Answer: C. Providing constructive feedback for improvement**

17. What is the role of mentorship in clinical training?

- A. Isolating learners from experienced professionals

- B. Promoting a competitive environment

- C. Providing guidance, support, and career advice

- D. Ignoring the importance of interpersonal skills

 

 

**Answer: C. Providing guidance, support, and career advice**

18. In clinical education, what does the term "patient-centred care" emphasize?

- A. Focusing solely on medical procedures

- B. Prioritizing administrative tasks

- C. Placing the patient's needs and preferences at the forefront

- D. Avoiding communication with patients

 

**Answer: C. Placing the patient's needs and preferences at the forefront**

19. What is the purpose of "clinical rounds" in medical education?

- A. Isolating healthcare professionals from each other

- B. Providing an opportunity for interdisciplinary collaboration

- C. Ignoring patient interactions

- D. Relying solely on individualized learning

 

 

**Answer: B. Providing an opportunity for interdisciplinary collaboration**

20. How does clinical training contribute to the development of professionalism in healthcare practitioners?

- A. By emphasizing technical skills only

- B. By promoting ethical behaviour, communication, and teamwork

- C. By avoiding interactions with patients

- D. By relying solely on theoretical knowledge

 

**Answer: B. By promoting ethical behaviour, communication, and teamwork*

Ø SEROLOGY  60 MCQ

 

1. What is the primary focus of serology in a medical laboratory?

a. Study of microorganisms
b. Study of blood serum
c. Study of tissues
d. Study of urine

Answer: b. Study of blood serum

2. Which of the following is a common serological test used for the detection of antibodies in the blood?

a. Polymerase Chain Reaction (PCR)
b. ELISA (Enzyme-Linked Immunosorbent Assay)
c. Western Blot
d. Flow cytometry

Answer: b. ELISA (Enzyme-Linked Immunosorbent Assay)

3. What is the main purpose of the VDRL (Venereal Disease Research Laboratory) test?

a. Detection of HIV
b. Detection of syphilis
c. Detection of hepatitis B
d. Detection of malaria

Answer: b. Detection of syphilis

4. Which immunoglobulin class is primarily involved in serological reactions?

a. IgA
b. IgE
c. IgG
d. IgM

Answer: c. IgG

5. In blood typing, the presence of which antigens determines the ABO blood group?

a. Rh antigen
b. A and B antigens
c. H antigen
d. K antigen

Answer: b. A and B antigens

6. The rapid plasma regain (RPR) test is commonly used for the diagnosis of which disease?

a. HIV
b. Syphilis
c. Tuberculosis
d. Hepatitis C

Answer: b. Syphilis

7. Which of the following is a confirmatory test for HIV infection?

a. ELISA
b. PCR
c. Western Blot
d. Rapid Test

Answer: c. Western Blot

8. What is the purpose of the Coombs test in serology?

a. Detection of bacterial infections
b. Detection of autoimmune haemolytic anaemia
c. Identification of viral infections
d. Measurement of blood glucose levels

Answer: b. Detection of autoimmune haemolytic anaemia

9. Which of the following is a common marker used in serological tests to indicate recent streptococcal infection?

a. ASO (Anti-Streptolysin O)
b. CRP (C-Reactive Protein)
c. RF (Rheumatoid Factor)
d. ANA (Antinuclear Antibody)

Answer: a. ASO (Anti-Streptolysin O)

10. The presence of which antibody is typically associated with allergic reactions in serological testing?

a. IgA
b. IgE
c. IgG
d. IgM

Answer: b. IgE

11. Which serological test is commonly used for the diagnosis of infectious mononucleosis?

a. Widal test
b. Monospot test
c. Mantoux test
d. Coombs test

Answer: b. Monospot test

12. What is the primary purpose of the Rheumatoid Factor (RF) test in serology?

a. Detection of rheumatic fever
b. Diagnosis of rheumatoid arthritis
c. Identification of viral infections
d. Monitoring glucose levels in diabetes

Answer: b. Diagnosis of rheumatoid arthritis

13. Which of the following serological tests is used for the diagnosis of autoimmune diseases such as systemic lupus erythematosus (SLE)?

a. ANA (Antinuclear Antibody) test
b. VDRL test
c. TPHA (Treponema Pallidum Hemagglutination Assay)
d. CRP (C-Reactive Protein) test

Answer: a. ANA (Antinuclear Antibody) test

14. In blood banking, which test is performed to determine compatibility between the donor's blood and the recipient's blood before a transfusion?

a. ELISA
b. Crossmatch
c. Western Blot
d. Coombs test

Answer: b. Crossmatch

15. The Quellung reaction is a serological test used for the identification of which type of microorganisms?

a. Bacteria
b. Viruses
c. Fungi
d. Parasites

Answer: a. Bacteria

16. What is the primary target of the RPR (Rapid Plasma Reagin) test for syphilis?

a. Treponema pallidum antibodies
b. Treponema pallidum antigens
c. HIV antibodies
d. Hepatitis B antigens

Answer: b. Treponema pallidum antigens

17. Which serological test is commonly used for the diagnosis of hepatitis B infection?

a. Western Blot
b. ELISA
c. HBsAg (Hepatitis B Surface Antigen) test
d. Monospot test

Answer: c. HBsAg (Hepatitis B Surface Antigen) test

18. What is the significance of the VDRL test titer in the diagnosis of syphilis?

a. It indicates the severity of the infection
b. It determines the stage of syphilis
c. It assesses the response to treatment
d. It identifies the causative microorganism

Answer: c. It assesses the response to treatment

19. Which of the following is a confirmatory test for hepatitis C infection?

a. HCV RNA PCR
b. HBsAg test
c. HCV ELISA
d. Monospot test

Answer: a. HCV RNA PCR

20. In blood grouping, which blood type is considered the universal donor?

a. A positive
b. B negative
c. AB positive
d. O negative

Answer: d. O negative

21. What does a positive Rheumatoid Factor (RF) test indicate?

a. Recent streptococcal infection
b. Rheumatoid arthritis
c. Systemic lupus erythematosus (SLE)
d. Infectious mononucleosis

Answer: b. Rheumatoid arthritis

22. Which serological marker is associated with the autoimmune disorder known as celiac disease?

a. Anti-CCP (Anti-Cyclic Citrullinated Peptide)
b. ANA (Antinuclear Antibody)
c. Anti-TTG (Anti-Tissue Transglutaminase)
d. ASO (Anti-Streptolysin O)

Answer: c. Anti-TTG (Anti-Tissue Transglutaminase)

23. What is the primary purpose of the C-reactive protein (CRP) test in serology?

a. Diagnosis of autoimmune diseases
b. Monitoring inflammation and infection
c. Detection of viral infections
d. Blood typing

Answer: b. Monitoring inflammation and infection

24. Which serological test is commonly used for the detection of antibodies in the diagnosis of autoimmune thyroid disorders such as Hashimoto's thyroiditis?

a. TSH (Thyroid Stimulating Hormone) test
b. Anti-TPO (Antithyroid Peroxidase) test
c. Free T4 test
d. Thyroglobulin test

Answer: b. Anti-TPO (Antithyroid Peroxidase) test

25. The Mantoux test is a type of serological test used for the diagnosis of:

a. Tuberculosis
b. HIV/AIDS
c. Syphilis
d. Malaria

Answer: a. Tuberculosis

26. Which serological test is commonly used for the detection of antibodies in the diagnosis of autoimmune disorders like systemic lupus erythematosus (SLE)?

a. ASO (Anti-Streptolysin O) test
b. VDRL (Venereal Disease Research Laboratory) test
c. ANA (Antinuclear Antibody) test
d. Monospot test

Answer: c. ANA (Antinuclear Antibody) test

27. What is the primary purpose of the Rapid Test for HIV?

a. Initial screening for HIV antibodies
b. Confirmation of HIV infection
c. Monitoring HIV viral load
d. Detection of hepatitis B antigens

Answer: a. Initial screening for HIV antibodies

28. Which of the following is a serological test used for the diagnosis of autoimmune hemolytic anemia?

a. Coombs test
b. Widal test
c. Monospot test
d. TPHA (Treponema Pallidum Hemagglutination Assay)

Answer: a. Coombs test

29. Which serological test is commonly used for the diagnosis of rheumatic fever and glomerulonephritis?

a. ASO (Anti-Streptolysin O) test
b. CRP (C-Reactive Protein) test
c. RF (Rheumatoid Factor) test
d. ANA (Antinuclear Antibody) test

Answer: a. ASO (Anti-Streptolysin O) test

30. The QuantiFERON-TB Gold test is used for the diagnosis of:

a. Malaria
b. Tuberculosis
c. HIV/AIDS
d. Hepatitis C

Answer: b. Tuberculosis

31. Which serological test is commonly used for the diagnosis of autoimmune disorders such as multiple sclerosis?

a. ANA (Antinuclear Antibody) test
b. ESR (Erythrocyte Sedimentation Rate) test
c. Anti-MOG (Myelin Oligodendrocyte Glycoprotein) test
d. Widal test

Answer: c. Anti-MOG (Myelin Oligodendrocyte Glycoprotein) test

32. The HCG (Human Chorionic Gonadotropin) test is commonly used for:

a. Detection of thyroid disorders
b. Pregnancy testing
c. Monitoring diabetes
d. Detection of autoimmune diseases

Answer: b. Pregnancy testing

33. Which of the following is a serological test used for the diagnosis of autoimmune blistering diseases such as pemphigus vulgaris?

a. TPHA (Treponema Pallidum Hemagglutination Assay)
b. Bullous Pemphigoid Antibody test
c. VDRL (Venereal Disease Research Laboratory) test
d. Monospot test

Answer: b. Bullous Pemphigoid Antibody test

34. The Widal test is commonly employed for the diagnosis of:

a. Tuberculosis
b. Typhoid fever
c. Hepatitis B
d. Dengue fever

Answer: b. Typhoid fever

35. Which serological test is commonly used for the diagnosis of autoimmune diseases like systemic sclerosis (scleroderma)?

a. Anti-Sm (Anti-Smith) test
b. Anti-dsDNA (Anti-double-stranded DNA) test
c. Anti-CCP (Anti-Cyclic Citrullinated Peptide) test
d. Anti-TPO (Antithyroid Peroxidase) test

Answer: a. Anti-Sm (Anti-Smith) test

36. The Rapid Antigen Test (RAT) is frequently used for the rapid detection of:

a. HIV
b. Influenza
c. Hepatitis C
d. Syphilis

Answer: b. Influenza

37. Which serological marker is associated with autoimmune liver diseases, such as autoimmune hepatitis?

a. Anti-TPO (Antithyroid Peroxidase)
b. Anti-MOG (Myelin Oligodendrocyte Glycoprotein)
c. Anti-LKM (Liver Kidney Microsomal)
d. ANA (Antinuclear Antibody)

Answer: c. Anti-LKM (Liver Kidney Microsomal)

38. The C3 and C4 complement levels are often measured in serological tests to assess:

a. Liver function
b. Renal function
c. Immune system activity
d. Cardiovascular health

Answer: c. Immune system activity

39. The HbA1c test is not a typical serological test but is commonly used to monitor:

a. Liver function
b. Blood glucose levels over time
c. Autoimmune diseases
d. Hormone levels in pregnancy

Answer: b. Blood glucose levels over time

40. The TPHA (Treponema Pallidum Hemagglutination Assay) test is commonly used as a confirmatory test for:

a. HIV
b. Syphilis
c. Tuberculosis
d. Hepatitis B

Answer: b. Syphilis

41. The Anti-CCP (Anti-Cyclic Citrullinated Peptide) test is primarily associated with the diagnosis of:

a. Rheumatoid arthritis
b. Tuberculosis
c. Lupus erythematosus
d. Multiple sclerosis

Answer: a. Rheumatoid arthritis

42. The RAST (Radio-allergo-sorbent) test is used for the diagnosis of:

a. Autoimmune diseases
b. Allergies
c. Viral infections
d. Thyroid disorders

Answer: b. Allergies

43. What is the primary purpose of the D-dimer test in serology?

a. Detection of liver function
b. Monitoring blood glucose levels
c. Evaluation of coagulation and fibrinolysis
d. Identification of autoimmune diseases

Answer: c. Evaluation of coagulation and fibrinolysis

44. The Anti-dsDNA (Anti-double-stranded DNA) test is associated with the diagnosis of:

a. Lupus erythematosus
b. Rheumatoid arthritis
c. Multiple sclerosis
d. Hashimoto's thyroiditis

Answer: a. Lupus erythematosus

45. The Tzanck smear is a diagnostic test used for the detection of:

a. Fungal infections
b. Bacterial infections
c. Viral infections
d. Parasitic infections

Answer: c. Viral infections

46. Which serological test is commonly used to diagnose and monitor autoimmune blistering diseases such as pemphigus vulgaris?

a. TPHA (Treponema Pallidum Hemagglutination Assay)
b. Coombs test
c. ANA (Antinuclear Antibody) test
d. Bullous Pemphigoid Antibody test

Answer: d. Bullous Pemphigoid Antibody test

47. The AFP (Alpha-fetoprotein) test is often used for the screening and monitoring of:

a. Liver cancer
b. Lung cancer
c. Breast cancer
d. Colorectal cancer

Answer: a. Liver cancer

48. The TSH (Thyroid Stimulating Hormone) test is commonly used to assess:

a. Liver function
b. Kidney function
c. Thyroid function
d. Pancreatic function

Answer: c. Thyroid function

49. Which serological test is commonly used for the detection of antibodies in the diagnosis of autoimmune disorders such as myasthenia gravis?

a. Anti-TPO (Antithyroid Peroxidase) test
b. Anti-AChR (Anti-Acetylcholine Receptor) test
c. CRP (C-Reactive Protein) test
d. Monospot test

Answer: b. Anti-AChR (Anti-Acetylcholine Receptor) test

50. The Schilling test is used to assess the absorption of:

a. Iron
b. Vitamin B12
c. Calcium
d. Vitamin D

Answer: b. Vitamin B12

51. The MPO-ANCA (Myeloperoxidase-Antineutrophil Cytoplasmic Antibody) test is associated with the diagnosis of:

a. Multiple sclerosis
b. Inflammatory bowel disease
c. Wegener's granulomatosis
d. Psoriasis

Answer: c. Wegener's granulomatosis

52. The CA-125 (Cancer Antigen 125) test is commonly used in the diagnosis and monitoring of:

a. Ovarian cancer
b. Prostate cancer
c. Lung cancer
d. Breast cancer

Answer: a. Ovarian cancer

53. The HLA-B27 test is associated with the diagnosis of:

a. Rheumatoid arthritis
b. Systemic lupus erythematosus (SLE)
c. Ankylosing spondylitis
d. Multiple sclerosis

Answer: c. Ankylosing spondylitis

54. The CA 19-9 (Cancer Antigen 19-9) test is commonly used in the diagnosis and monitoring of:

a. Colorectal cancer
b. Pancreatic cancer
c. Breast cancer
d. Lung cancer

Answer: b. Pancreatic cancer

55. The Anti-Jo-1 antibody is associated with:

a. Rheumatoid arthritis
b. Systemic sclerosis (scleroderma)
c. Polymyositis
d. Ankylosing spondylitis

Answer: c. Polymyositis

56. The T-SPOT.TB test is used for the diagnosis of:

a. Tuberculosis
b. Hepatitis B
c. HIV/AIDS
d. Malaria

Answer: a. Tuberculosis

57. The MMR (Measles, Mumps, Rubella) titer is measured in serology to assess:

a. Immunity to common childhood infections
b. Thyroid function
c. Liver function
d. Autoimmune diseases

Answer: a. Immunity to common childhood infections

58. The presence of anti-Smooth Muscle Antibody (ASMA) is associated with:

a. Hepatitis B infection
b. Autoimmune hepatitis
c. Rheumatoid arthritis
d. Lupus erythematosus

Answer: b. Autoimmune hepatitis

59. The ESR (Erythrocyte Sedimentation Rate) is a non-specific marker of:

a. Inflammation
b. Viral infections
c. Autoimmune diseases
d. Diabetes

Answer: a. Inflammation

60. The CEA (Carcinoembryonic Antigen) test is commonly used for monitoring:

a. Liver function
b. Pancreatic function
c. Tumour marker for colorectal cancer
d. Thyroid function

Answer: c. Tumour marker for colorectal cancer

Ø PATHOLOGY 100 MCQS

 

1. What is the primary focus of Clinical Pathology?

a. Study of blood
b. Study of tissues and cells
c. Study of bacteria and viruses
d. Study of hormones

Answer: b. Study of tissues and cells

2. The term "hematology" refers to the study of:

a. Hormones
b. Blood
c. Tissues
d. Urine

Answer: b. Blood

3. Which laboratory test is commonly used to assess kidney function?

a. Complete Blood Count (CBC)
b. Liver Function Tests (LFTs)
c. Blood Urea Nitrogen (BUN)
d. Serum Amylase

Answer: c. Blood Urea Nitrogen (BUN)

4. The presence of which cell type in the blood is indicative of an allergic or parasitic condition?

a. Neutrophils
b. Basophils
c. Lymphocytes
d. Monocytes

Answer: b. Basophils

5. Which of the following is not a component of a complete blood count (CBC)?

a. Hemoglobin
b. Platelet count
c. Serum creatinine
d. Red blood cell count

Answer: c. Serum creatinine

6. The laboratory test that measures the average blood glucose levels over the past 2-3 months is called:

a. Fasting Blood Sugar (FBS)
b. Random Blood Sugar (RBS)
c. Oral Glucose Tolerance Test (OGTT)
d. Hemoglobin A1c

Answer: d. Hemoglobin A1c

7. A differential white blood cell count provides information about:

a. Total number of white blood cells
b. Percentage of different types of white blood cells
c. Red blood cell count
d. Platelet count

Answer: b. Percentage of different types of white blood cells

8. The erythrocyte sedimentation rate (ESR) is a non-specific indicator of:

a. Inflammation
b. Blood clotting
c. Oxygen-carrying capacity of blood
d. Hemoglobin concentration

Answer: a. Inflammation

9. Which laboratory test is commonly used to assess liver function and detect liver disorders?

a. Serum Amylase
b. Alanine Aminotransferase (ALT)
c. Troponin
d. Creatine Kinase (CK)

Answer: b. Alanine Aminotransferase (ALT)

10. The presence of which antibody is typically tested in autoimmune disorders such as rheumatoid arthritis?

a. ANA (Antinuclear Antibody)
b. Anti-TPO (Antithyroid Peroxidase)
c. ASO (Anti-Streptolysin O)
d. RF (Rheumatoid Factor)

Answer: d. RF (Rheumatoid Factor)

11. Which of the following is a marker used in the diagnosis and monitoring of cardiac muscle damage?

a. Troponin
b. Amylase
c. Creatinine
d. Lipase

Answer: a. Troponin

12. The measurement of activated partial thromboplastin time (APTT) is associated with the assessment of:

a. Blood glucose levels
b. Coagulation pathways
c. Liver function
d. Red blood cell morphology

Answer: b. Coagulation pathways

13. Which of the following is a test used to evaluate the function of the thyroid gland?

a. Serum Creatinine
b. Thyroid Stimulating Hormone (TSH)
c. Alanine Aminotransferase (ALT)
d. Lipid Profile

Answer: b. Thyroid Stimulating Hormone (TSH)

14. The prothrombin time (PT) is a measure of:

a. Red blood cell count
b. Blood clotting time
c. Oxygen-carrying capacity of blood
d. White blood cell differential count

Answer: b. Blood clotting time

15. Which enzyme is commonly elevated in the blood during a myocardial infarction (heart attack)?

a. Alanine Aminotransferase (ALT)
b. Creatine Kinase (CK)
c. Amylase
d. Lipase

Answer: b. Creatine Kinase (CK)

16. The CA 15-3 and CA 27-29 tumor markers are associated with the diagnosis and monitoring of:

a. Prostate cancer
b. Ovarian cancer
c. Colorectal cancer
d. Breast cancer

Answer: d. Breast cancer

17. The C-reactive protein (CRP) test is used as a marker of:

a. Liver function
b. Kidney function
c. Inflammation
d. Thyroid function

Answer: c. Inflammation

18. The CSF analysis is a diagnostic test that involves the examination of:

a. Blood cells in cerebrospinal fluid
b. Liver enzymes in cerebrospinal fluid
c. Glucose levels in cerebrospinal fluid
d. Red blood cells in urine

Answer: a. Blood cells in cerebrospinal fluid

19. The anti-TPO (Antithyroid Peroxidase) test is used in the diagnosis of:

a. Diabetes mellitus
b. Thyroid disorders
c. Autoimmune hepatitis
d. Lupus erythematosus

Answer: b. Thyroid disorders

20. The von Willebrand factor (vWF) assay is used to assess:

a. Liver function
b. Platelet function
c. Kidney function
d. Coagulation pathways

Answer: b. Platelet function

21. Which of the following blood cell types is primarily responsible for immune responses and antibody production?

a. Neutrophils
b. Lymphocytes
c. Eosinophils
d. Basophils

Answer: b. Lymphocytes

22. The APTT (Activated Partial Thromboplastin Time) test is commonly used to monitor the effectiveness of:

a. Anticoagulant therapy
b. Antibiotic treatment
c. Antihypertensive medications
d. Antidiabetic medications

Answer: a. Anticoagulant therapy

23. The presence of microorganisms in the blood is detected through which laboratory test?

a. Blood Urea Nitrogen (BUN)
b. Blood Culture
c. Erythrocyte Sedimentation Rate (ESR)
d. Complete Blood Count (CBC)

Answer: b. Blood Culture

24. Which of the following tests is commonly used to assess pancreatic function?

a. Lipase
b. Creatinine
c. Amylase
d. Troponin

Answer: a. Lipase

25. The CA 125 tumor marker is associated with the diagnosis and monitoring of:

a. Colorectal cancer
b. Ovarian cancer
c. Prostate cancer
d. Lung cancer

Answer: b. Ovarian cancer

26. The MCHC (Mean Corpuscular Hemoglobin Concentration) is a measure of:

a. Hemoglobin levels in the blood
b. Red blood cell count
c. Oxygen-carrying capacity of hemoglobin
d. Iron levels in the blood

Answer: c. Oxygen-carrying capacity of hemoglobin

27. The D-dimer test is often used in the diagnosis of:

a. Liver disorders
b. Thrombotic disorders
c. Pancreatic disorders
d. Renal disorders

Answer: b. Thrombotic disorders

28. Which of the following is a marker used in the diagnosis and monitoring of prostate cancer?

a. CA 19-9
b. PSA (Prostate-Specific Antigen)
c. AFP (Alpha-fetoprotein)
d. CEA (Carcinoembryonic Antigen)

Answer: b. PSA (Prostate-Specific Antigen)

29. The Coombs test is used to diagnose:

a. Thyroid disorders
b. Anemia
c. Autoimmune hemolytic anemia
d. Kidney disorders

Answer: c. Autoimmune hemolytic anemia

30. The Tzanck smear is utilized for the diagnosis of:

a. Skin infections
b. Viral infections
c. Urinary tract infections
d. Respiratory infections

Answer: b. Viral infections

31. The MCH (Mean Corpuscular Hemoglobin) is a measure of:

a. Red blood cell count
b. Hemoglobin levels in the blood
c. Oxygen-carrying capacity of hemoglobin
d. Platelet count

Answer: b. Hemoglobin levels in the blood

32. The anti-dsDNA (Anti-double-stranded DNA) test is associated with the diagnosis of:

a. Lupus erythematosus
b. Rheumatoid arthritis
c. Multiple sclerosis
d. Hashimoto's thyroiditis

Answer: a. Lupus erythematosus

33. The Wright stain is commonly used for the microscopic examination of:

a. Blood cells
b. Urine crystals
c. Tissue sections
d. Bacteria

Answer: a. Blood cells

34. The LDH (Lactate Dehydrogenase) test is often used in the diagnosis of:

a. Liver disorders
b. Kidney disorders
c. Cardiac disorders
d. Hematologic disorders

Answer: c. Cardiac disorders

35. The T3 and T4 tests are used to evaluate the function of which organ?

a. Liver
b. Kidneys
c. Thyroid
d. Pancreas

Answer: c. Thyroid

36. The HbA1c test is primarily used to monitor:

a. Liver function
b. Blood glucose levels over time
c. Thyroid function
d. Kidney function

Answer: b. Blood glucose levels over time

37. The Schilling test is utilized for the diagnosis of:

a. Anemia
b. Vitamin B12 deficiency
c. Liver disorders
d. Diabetes mellitus

Answer: b. Vitamin B12 deficiency

38. The VLDL (Very Low-Density Lipoprotein) test measures levels of:

a. Triglycerides
b. Cholesterol
c. High-density lipoprotein (HDL)
d. Low-density lipoprotein (LDL)

Answer: a. Triglycerides

39. The HCV RNA PCR test is used for the diagnosis of:

a. Hepatitis B
b. Hepatitis C
c. HIV
d. Tuberculosis

Answer: b. Hepatitis C

40. The WBC (White Blood Cell) count is a measure of:

a. Total number of white blood cells in a sample
b. Oxygen-carrying capacity of red blood cells
c. Platelet count
d. Hemoglobin concentration

Answer: a. Total number of white blood cells in a sample

41. The Haptoglobin test is commonly used to assess:

a. Liver function
b. Renal function
c. Pancreatic function
d. Hemolytic disorders

Answer: d. Hemolytic disorders

42. The Ferritin test is used to evaluate:

a. Iron levels in the blood
b. Blood clotting time
c. Thyroid function
d. Kidney function

Answer: a. Iron levels in the blood

43. The TdT (Terminal Deoxynucleotidyl Transferase) test is associated with the diagnosis of:

a. Leukemia
b. Multiple myeloma
c. Lymphoma
d. Thrombocytopenia

Answer: a. Leukemia

44. The presence of atypical lymphocytes is often associated with which viral infection?

a. Influenza
b. Hepatitis B
c. HIV
d. Herpes simplex

Answer: c. HIV

45. The PTT (Partial Thromboplastin Time) test is commonly used to evaluate:

a. Platelet function
b. Coagulation pathways
c. Red blood cell count
d. White blood cell differential count

Answer: b. Coagulation pathways

46. The PSA (Prostate-Specific Antigen) test is used for the screening and monitoring of:

a. Ovarian cancer
b. Prostate cancer
c. Colorectal cancer
d. Lung cancer

Answer: b. Prostate cancer

47. The MCV (Mean Corpuscular Volume) is a measure of:

a. Red blood cell count
b. Hemoglobin levels in the blood
c. Oxygen-carrying capacity of hemoglobin
d. Average volume of red blood cells

Answer: d. Average volume of red blood cells

48. The presence of anti-HCV antibodies is indicative of infection with which virus?

a. Hepatitis A
b. Hepatitis B
c. Hepatitis C
d. Hepatitis D

Answer: c. Hepatitis C

49. The sweat chloride test is used in the diagnosis of:

a. Cystic fibrosis
b. Diabetes mellitus
c. Chronic kidney disease
d. Hemochromatosis

Answer: a. Cystic fibrosis

50. The CA 19-9 (Cancer Antigen 19-9) test is commonly used in the diagnosis and monitoring of:

a. Colorectal cancer
b. Pancreatic cancer
c. Breast cancer
d. Lung cancer

Answer: b. Pancreatic cancer

51. The JAK2 mutation test is associated with the diagnosis of:

a. Leukaemia
b. Multiple myeloma
c. Polycythaemia vera
d. Lymphoma

Answer: c. Polycythaemia vera

52. The Reticulocyte Count is a measure of:

a. Immature red blood cells in the blood
b. Platelet count
c. Mature red blood cells in the blood
d. White blood cell count

Answer: a. Immature red blood cells in the blood

53. The anti-CCP (Anti-Cyclic Citrullinated Peptide) test is associated with the diagnosis of:

a. Rheumatoid arthritis
b. Systemic lupus erythematosus (SLE)
c. Osteoarthritis
d. Gout

Answer: a. Rheumatoid arthritis

54. The Methylmalonic Acid (MMA) test is used to assess:

a. Liver function
b. Renal function
c. Vitamin B12 deficiency
d. Thyroid function

Answer: c. Vitamin B12 deficiency

55. The TSH (Thyroid Stimulating Hormone) test is commonly used to assess:

a. Liver function
b. Kidney function
c. Thyroid function
d. Pancreatic function

Answer: c. Thyroid function

56. The Heparin-induced Thrombocytopenia (HIT) test is used to detect an adverse reaction to:

a. Antibiotics
b. Anticoagulant medication
c. Pain relievers
d. Antihypertensive medications

Answer: b. Anticoagulant medication

57. The C3 and C4 complement levels are often measured in the diagnosis of:

a. Hepatitis
b. Rheumatoid arthritis
c. Lupus erythematosus
d. Diabetes mellitus

Answer: c. Lupus erythematosus

58. The CA 72-4 tumor marker is associated with the diagnosis and monitoring of:

a. Colorectal cancer
b. Ovarian cancer
c. Lung cancer
d. Prostate cancer

Answer: b. Ovarian cancer

59. The D-dimer test is commonly used to assess:

a. Liver function
b. Kidney function
c. Coagulation and fibrinolysis
d. Thyroid function

Answer: c. Coagulation and fibrinolysis

60. The Papanicolaou (Pap) smear is a screening test for:

a. Breast cancer
b. Ovarian cancer
c. Cervical cancer
d. Prostate cancer

Answer: c. Cervical cancer

61. The Tumor Marker CA 15-3 is associated with:

a. Ovarian cancer
b. Breast cancer
c. Lung cancer
d. Colorectal cancer

Answer: b. Breast cancer

62. The Mantoux test is used for the diagnosis of:

a. Tuberculosis
b. Malaria
c. HIV/AIDS
d. Hepatitis B

Answer: a. Tuberculosis

63. The BUN (Blood Urea Nitrogen) test is commonly used to assess:

a. Liver function
b. Renal function
c. Thyroid function
d. Pancreatic function

Answer: b. Renal function

64. The Anti-Jo-1 antibody is associated with:

a. Rheumatoid arthritis
b. Systemic sclerosis (scleroderma)
c. Polymyositis
d. Ankylosing spondylitis

Answer: c. Polymyositis

65. The von Willebrand factor (vWF) assay is used to assess:

a. Liver function
b. Platelet function
c. Kidney function
d. Coagulation pathways

Answer: b. Platelet function

66. The HLA-B27 test is associated with the diagnosis of:

a. Rheumatoid arthritis
b. Systemic lupus erythematosus (SLE)
c. Ankylosing spondylitis
d. Multiple sclerosis

Answer: c. Ankylosing spondylitis

67. The CEA (Carcinoembryonic Antigen) test is a tumor marker primarily associated with:

a. Colorectal cancer
b. Ovarian cancer
c. Lung cancer
d. Breast cancer

Answer: a. Colorectal cancer

68. The MPO-ANCA (Myeloperoxidase-Antineutrophil Cytoplasmic Antibody) test is associated with the diagnosis of:

a. Multiple sclerosis
b. Inflammatory bowel disease
c. Wegener's granulomatosis
d. Psoriasis

Answer: c. Wegener's granulomatosis

69. The Tzanck smear is a diagnostic test used for the detection of:

a. Fungal infections
b. Bacterial infections
c. Viral infections
d. Parasitic infections

Answer: c. Viral infections

70. The ESR (Erythrocyte Sedimentation Rate) is a non-specific marker of:

a. Inflammation
b. Viral infections
c. Autoimmune diseases
d. Diabetes

Answer: a. Inflammation

71. The Widal test is used for the diagnosis of:

a. Tuberculosis
b. Malaria
c. Typhoid fever
d. Hepatitis B

Answer: c. Typhoid fever

72. The TSH (Thyroid Stimulating Hormone) test is commonly used to assess:

a. Liver function
b. Kidney function
c. Thyroid function
d. Pancreatic function

Answer: c. Thyroid function

73. The Anti-Smooth Muscle Antibody (ASMA) test is associated with the diagnosis of:

a. Hepatitis B infection
b. Autoimmune hepatitis
c. Rheumatoid arthritis
d. Lupus erythematosus

Answer: b. Autoimmune hepatitis

74. The serum uric acid test is used to assess:

a. Kidney function
b. Liver function
c. Thyroid function
d. Pancreatic function

Answer: a. Kidney function

75. The MMR (Measles, Mumps, Rubella) titer is measured in serology to assess:

a. Immunity to common childhood infections
b. Thyroid function
c. Liver function
d. Autoimmune diseases

Answer: a. Immunity to common childhood infections

76. The Rheumatoid Factor (RF) test is associated with the diagnosis of:

a. Rheumatoid arthritis
b. Systemic lupus erythematosus (SLE)
c. Osteoarthritis
d. Gout

Answer: a. Rheumatoid arthritis

77. The Prothrombin Time (PT) test assesses the function of:

a. Platelets
b. Coagulation pathways
c. Red blood cells
d. White blood cells

Answer: b. Coagulation pathways

78. The Anti-AChR (Anti-Acetylcholine Receptor) test is used for the diagnosis of:

a. Multiple sclerosis
b. Myasthenia gravis
c. Lupus erythematosus
d. Psoriasis

Answer: b. Myasthenia gravis

79. The ANCA (Antineutrophil Cytoplasmic Antibody) test is associated with the diagnosis of:

a. Multiple sclerosis
b. Inflammatory bowel disease
c. Wegener's granulomatosis
d. Psoriasis

Answer: c. Wegener's granulomatosis

80. The Creatinine Clearance test is used to assess:

a. Liver function
b. Renal function
c. Thyroid function
d. Pancreatic function

Answer: b. Renal function

81. The HbA1c test is primarily used for monitoring:

a. Liver function
b. Blood glucose levels over time
c. Thyroid function
d. Kidney function

Answer: b. Blood glucose levels over time

82. The anti-dsDNA (Anti-double-stranded DNA) test is associated with the diagnosis of:

a. Lupus erythematosus
b. Rheumatoid arthritis
c. Multiple sclerosis
d. Hashimoto's thyroiditis

Answer: a. Lupus erythematosus

83. The Fibrinogen level is commonly assessed to evaluate:

a. Liver function
b. Coagulation pathways
c. Thyroid function
d. Pancreatic function

Answer: b. Coagulation pathways

84. The Anti-TPO (Antithyroid Peroxidase) test is used in the diagnosis of:

a. Diabetes mellitus
b. Thyroid disorders
c. Autoimmune hepatitis
d. Lupus erythematosus

Answer: b. Thyroid disorders

85. The C-reactive protein (CRP) test is used as a marker of:

a. Liver function
b. Kidney function
c. Inflammation
d. Thyroid function

Answer: c. Inflammation

86. The Ammonia test is often used in the assessment of:

a. Liver function
b. Kidney function
c. Coagulation pathways
d. Thyroid function

Answer: a. Liver function

87. The HLA-B27 test is associated with the diagnosis of:

a. Rheumatoid arthritis
b. Systemic lupus erythematosus (SLE)
c. Ankylosing spondylitis
d. Multiple sclerosis

Answer: c. Ankylosing spondylitis

88. The CA 27-29 tumor marker is commonly used in the diagnosis and monitoring of:

a. Colorectal cancer
b. Ovarian cancer
c. Breast cancer
d. Pancreatic cancer

Answer: c. Breast cancer

89. The APTT (Activated Partial Thromboplastin Time) test is commonly used to monitor the effectiveness of:

a. Anticoagulant therapy
b. Antibiotic treatment
c. Antihypertensive medications
d. Antidiabetic medications

Answer: a. Anticoagulant therapy

90. The Anti-Smooth Muscle Antibody (ASMA) test is associated with the diagnosis of:

a. Hepatitis B infection
b. Autoimmune hepatitis
c. Rheumatoid arthritis
d. Lupus erythematosus

Answer: b. Autoimmune hepatitis

91. The anti-centromere antibody is associated with:

a. Systemic lupus erythematosus (SLE)
b. Scleroderma (systemic sclerosis)
c. Rheumatoid arthritis
d. Myasthenia gravis

Answer: b. Scleroderma (systemic sclerosis)

92. The aldolase enzyme is elevated in conditions affecting:

a. Liver
b. Heart
c. Muscles
d. Kidneys

Answer: c. Muscles

93. The MUGA scan is a diagnostic test for assessing:

a. Liver function
b. Cardiac function
c. Kidney function
d. Lung function

Answer: b. Cardiac function

94. The Anti-Jo-1 antibody is associated with which group of autoimmune disorders?

a. Connective tissue diseases
b. Gastrointestinal diseases
c. Endocrine disorders
d. Neurological disorders

Answer: a. Connective tissue diseases

95. The Schilling test is used for the diagnosis of:

a. Anemia
b. Vitamin B12 deficiency
c. Liver disorders
d. Diabetes mellitus

Answer: b. Vitamin B12 deficiency

96. The Russell viper venom time (RVVT) test is used to evaluate:

a. Liver function
b. Coagulation pathways
c. Renal function
d. Thyroid function

Answer: b. Coagulation pathways

97. The presence of anti-Sm antibodies is associated with:

a. Systemic lupus erythematosus (SLE)
b. Rheumatoid arthritis
c. Multiple sclerosis
d. Psoriasis

Answer: a. Systemic lupus erythematosus (SLE)

98. The sweat chloride test is used in the diagnosis of:

a. Cystic fibrosis
b. Diabetes mellitus
c. Chronic kidney disease
d. Hemochromatosis

Answer: a. Cystic fibrosis

99. The C3 and C4 complement levels are often measured in the diagnosis of:

a. Hepatitis
b. Rheumatoid arthritis
c. Lupus erythematosus
d. Diabetes mellitus

Answer: c. Lupus erythematosus

100. The RPR (Rapid Plasma Reagin) test is commonly used for the screening of:

a. Syphilis
b. HIV/AIDS
c. Hepatitis B
d. Tuberculosis

Answer: a. Syphilis

Ø PART- BLOOD BANK 50 MCQS

Ø PART- SICKLE CELL 40 MCQS

Ø MALARIA DISEASE 40 MCQS

Ø BACTERIAL DISEASE 50 MCQS

Ø VIROLOGY MCQS 80 MCQS

Ø IMMUNOLOGY MCQS 100

Ø CYTOLOGY 80 MCQS

Ø HISTOPATHOLOGY 100 MCQS

Ø ANATOMY & PHYSIOLOGY 100 MCQS

Ø Clinical Pathology 100 MCQS

Ø Medical Laboratory Instrumentation 20 MCQS

Ø Clinical Microscopy 30 MCQS

Ø Quality Control and Quality Assurance 30 MCQS

Ø Clinical Training 20 MCQS

Ø SEROLOGY  60 MCQS

Ø PATHOLOGY 100 MCQS

         PDF PASSWORD- MLT2001TO3000

P

Ø TOTAL- 1000

"ALL UPLOADS"

MY ALL UPLOADS (SARKARI RESULTS FAST) CLICK HERE

  1-   AMERICAN LAB TECHNOLOGIST EXAM QUESTION ANSWER 2-  Clinical Chemistry mcqs for Medical Lab Technologist, 3-   Lab Technician Previous...

Popular Post